RN Maternity HESI Review Questions

¡Supera tus tareas y exámenes ahora con Quizwiz!

The mother of a 5-week-old tells the nurse that her baby has acne and asks if she can use her teenage son's acne cream, benzoyl peroxide, on the baby's face. Which answer should the nurse to provide?

" Your baby may be showing signs of a systemic disease and needs to be seen by a healthcare provider"

The newborn nursery admission protocol includes a prescption for phytonadione (Vitamin K1, AquaMEPHYTON) 0.5 mg IM to newborns upon admission. The ampoule provides 2 mg/ml. How many ml should the nurse administer?

0.3

A 4-day postpartum client calls the clinic and reports that her nipples are so sore that she does not know if she can continue to breastfeed her infant. What instruction is best for the nurse to provide?

Apply hot packs just before each feeding.

The cervix of a client in labor is dilated to 8 cm. She tells the nurse that she has a desire to push and is becoming increasingly uncomfortable. The client requests pain medication. How should the nurse respond at this time?

Help her take panting breaths

A 16-year-old gravida 1, para 0 client has just been admitted to the hospital with a diagnosis of eclampsia. She is not presently convulsing. Which intervention should the nurse plan to include in this client's nursing care plan?

Monitor Blood pressure, pulse, and respirations q4h.

The nurse teaches a client who is about to undergo an amniocentesis that ultrasonography will be performed just before the procedure to determine what?

Position of the fetus and the placenta

A client at 32 weeks' gestation visits the prenatal clinic because she is experiencing uterine contractions. She is to be treated at home with restricted activity and long periods of bed rest. Which instructions should the teaching plan include when the client is advised to remain in bed?

"Assume a side position, with the head raised on a small pillow."

A client with a history of endometriosis gives birth to a healthy infant. She expresses concern that the problems associated with endometriosis will return now that her pregnancy is over. What is the best response by the nurse?

"Breast-feeding will delay the return of the endometriosis."

The nurse weighs a 6-month-old infant during a well-baby check-up and determines that the baby's weight has triple compared to the birth weight of 7 pounds 8 ounces. The mother asks if the baby is gaining enough weight. What response should the nurse offer?

"What food does your baby usually eat in a normal day?"

During labor, the nurse determines that a full term client is demonstrating late decelerations. In which sequence should the nurse implement these nursing actions? (Arrange in order) a. Provide oxygen via face mack b. Reposition the client c. Increase IV fluid d. Call the healthcare provider

1. Reposition the Client 2. Provide oxygen via face mask 3. Increase IV fluid 4. Call the healthcare provider To stabilize the fetus, intrauterine resuscitation is the first priority, and to enhance the fetal blood supply, the laboring client should be repositioned (1) to displace the gravid uterus and improve fetal perfusion. Secondly, to optimize oxygenation of the circulatory blood volume, oxygen via face mask (2) should be applied to the mother. Next, the IV fluids should be increased (3) to expand the maternal circulating blood volume. Then, the primary healthcare provider should be notified (4) for additional interventions to resolve the fetal stress.

Put the following actions in order to prevent hypotension in the pregnant client: 1. reposition the client 2. provide oxygen via face mask 3. increase IV fluid 4. call the healthcare provider

1. reposition the client 2. increase the IV fluid 3. provide oxygen via face mask 4. call the healthcare provider.

The healthcare provider prescribes zidovudine 100mg po 5x daily for a pregnant woman who is HIV positive. How much do you administer? (?)

10

A multigravida client in labor is receiving oxytocin 4mu/minute to help promote an effective contraction pattern. The available solution is Lactacted Ringer's 1,000 mL with oxytocin 20 units. The nurse should program the infusion pump to deliver how many mL/hr?

12

The healthcare provider prescribe Pitocin 2milliunits / min to induce labor for a client at 41 weeks gestation. The nurse initiates an infusion of Lactated Ringer 1000 ml with Pitocin 10 units. How many ml/ hr should the nurse program the infusion pump?

12

A loading dose of terbutaline (Bretine) 250 mcg IV is prescribed for a client in preterm labor. Brethine 20 mg is added to 1000 ml D5W. How many ml of the solution should the nurse administer? (Enter numeric value only)

13

At ten weeks gestation, a high risk multiparous client with a Fhx of downs syndrome is admitted for observation following a CVS. What assessment findings requires immediate action? A) Uterine Cramping B) Systolic BP <100 C) Abdominal tenderness D) Intermittent Nausea

A

A client at 39 weeks is admitted into the labor unit. Her OB history includes 3 live births at 39 weeks, 34 weeks, 35 weeks. Using the GTPAL system which designation is the most accurate summary of this client's obstetrical history? A) 4-1-2-0-3 B) 3-1-1-1-3 C) 4-3-1-0-2 D) 3-0-3-0-3

A

A macrosomic infant is in stable condition after a difficult forceps assisted delivery. After obtaining the infant's weight at 9lb 6oz, what is the priority nursing action? A) Obtain serum glucose levels frequently while observing closely for signs of hypoglycemia B) Assess newborn reflexes for signs of neurological impairment C) Perform a gestational age assessment to determine if the infant is large for gestational age D) Leave the infant in the room with the mother to foster attachment

A

A newborn infant who is 24 hours old is on a 4 hour feeding schedule of formula. To meet daily caloric needs, how many oz are recommended each feeding? A) 3.5 oz B) 2 oz C) 4 oz D) 1.5 oz

A

A woman whose pregnancy is confirmed asks the nurse what the function of the placenta is in early pregnancy. What information supports the explanation that the nurse should provide? A) Secretes both estrogen and progesterone B) Excretes prolactin and insulin C) Forms a protective impenetrable barrier D) Produces nutrients for fetal nutrition.

A

An infant in respiratory distress is placed on pulse ox. The O2 sat is 85%. What is the priority nursing intervention? A) Begin humidified oxygen B) Place the infant under a radiant warmer C) Evaluate the blood pH D) Stimulate infant to cry

A

An infant who weighs 3.8 kg is delivered vaginally at 39 weeks with a nuchal cord after a 30 minute second stage of labor. The nurse identifies petechiae over the face and upper back of the newborn. What information should the nurse provide the parents about this finding? A) The pinpoint spots are benign and disappear within 48 hours B) An increased blood volume causes broken blood vessels C) Further assessment is needed D) Petechiae occurs with forceps delivery

A

The nurse is giving discharge instructions for a client following a suction curettage for a hydatidiform mole. THe client asks why oral contraceptives are being recommended for the next 12 months. What information should the nurse provide? A) Diagnostic testing for hCG levels are elevated by pregnancy B) Molar recurrences are higher is conception occurs within 1 year after an initial mutation C) Pregnancy within 1 year decreases the chances of future successful pregnancy D) Oral contraceptives prevent a reoccurance of a molar pregnancy

A

The nurse is preparing to gavage feed a preterm infant who is receiving IV antibiotics. The infant expels a bloody stool. Which action should the nurse implement? A) Assess for abdominal distension B) Institute contact precautions C) Decrease the amount of feeding D) Obtain a rectal temp

A

What action should the nurse implement to prevent conductive heat loss in a newborn? A) put a blanket on the scale while weighing the infant B) Place the crib under a radiant warming system C) Position the crib away from the windows D) Dry the newborn in a warmed blanket

A

The nurse is counseling a couple who has sought information about conceiving. The couple asks the nurse to explain when ovulation usually occurs. Which statement by the nurse is correct? A.Two weeks before menstruation B.Immediately after menstruation C.Immediately before menstruation D.Three weeks before menstruation

A.Two weeks before menstruation Rationale:Ovulation occurs 14 days before the first day of the menstrual period. Although ovulation can occur in the middle of the cycle or 2 weeks after menstruation, this is only true for a woman who has a perfect 28-day cycle. For many women, the length of the menstrual cycle varies. Options B, C, and D are incorrect.

A pregnant client was exposed to a teratogen in her tenth week of gestation. Which nursing intervention should be provided?

Assure the client that the risk of malformation is minimal

Which statement is true for attachment in the newborn?

Attachment is the interaction between parent and child.

A client at 35 week visits the clinic for a prenatal check up. Which complaint by the client warrants further assessment? A) Backache with prolonged standing B) Periodic abdominal pain C) Ankle edema in the afternoon D) Shortness of breath when climbing stairs

B

During a prenatal visit, the nurse discusses the effects of smoking on the fetus with a client. Which statement is most characteristic of an infant whose mother smoked during pregnancy compared with the infant of a nonsmoking mother? A.Lower Apgar score recorded at delivery B.Lower initial weight documented at birth C.Higher oxygen use to stimulate breathing D.Higher prevalence of congenital anomalies

B.Lower initial weight documented at birth Rationale:Smoking is associated with low-birth-weight infants. Therefore, mothers are encouraged not to smoke during pregnancy. Options A, C, and D have not been clearly associated with smoking during pregnancy, but there is a strong correlation between smoking and lower birth weights.

The nurse is assessing a full-term newborn's breathing pattern. Which findings should the nurse assess further? (Select all) A) Shallow with an irregular rhythm B) Diaphragmatic with chest retraction C) Rate of 58 per minute D) Abdominal with synchronous chest movements E) Grunting heard with a stethoscope F) Chest breathing with nasal flaring

BEF

A mother expresses fear about changing the infant's diaper after circumcision. What information should the nurse include in the teaching plan? A.Cleanse the penis with prepackaged diaper wipes every 3 to 4 hours. B.Wash off the yellow exudate on the glans once every day to prevent infection. C.Place petroleum ointment around the glans with each diaper change and cleansing. D.Apply pressure by squeezing the penis with the fingers for 5 minutes if bleeding occurs.

C.Place petroleum ointment around the glans with each diaper change and cleansing. Rationale: With each diaper change, the glans penis should be washed with warm water to remove any urine or feces, and petroleum ointment should be applied to prevent the diaper from sticking to the healing surface. Prepackaged wipes often contain other products that may irritate the site. The yellow exudate, which covers the glans penis as the area heals and epithelializes, is not an infective process and should not be removed. If bleeding occurs at home, the client should be instructed to apply gentle pressure to the site of the bleeding with sterile gauze squares and call the health care provider.

The nurse is assessing a 38-week gestation newborn infant immediately following a vaginal birth. Which assessment finding best indicates that the infant is transitioning well to extrauterine life?

Cries vigorously when stimulated.

A client at 25 weeks tells the nurse that she dropped a cooking utensil last week and her baby jumped in response to the noise. What information should the nurse provide? A) This is a demonstration of the fetus's acoustical reflex B) It is a coincidence the fetus responded at the same time C) Report the behavior to the Dr. D) The fetus can respond to sound by 24 weeks

D

A client at 28 weeks gestation arrives at the labor and delivery unit with a complaint of bright red, painless vaginal bleeding. For which diagnostic procedure should the nurse prepare the client? A) Internal fetal monitoring B) Lecithin-sphingomylein ratio C) Contraction stress test D) Abdominal ultrasound

D

In developing a teaching plan for expectant parents, the nurse decides to include information about when the parents can expect the infant's fontanels to close. Which statement is accurate regarding the timing of closure of an infant's fontanels that should be included in this teaching plan? A.The anterior fontanel closes at 2 to 4 months and the posterior fontanel by the end of the first week. B.The anterior fontanel closes at 5 to 7 months and the posterior fontanel by the end of the second week. C.The anterior fontanel closes at 8 to 11 months and the posterior fontanel by the end of the first month. D.The anterior fontanel closes at 12 to 18 months and the posterior fontanel by the end of the second month

D.The anterior fontanel closes at 12 to 18 months and the posterior fontanel by the end of the second month Rationale: In the normal infant, the anterior fontanel closes at 12 to 18 months of age and the posterior fontanel closes by the end of the second month. These growth and development milestones are frequently included in questions on the licensure examination. Options A, B, and C are incorrect.

Which drug may cause gynecologic malignancies in females?

Diethylstilbestrol

At 6-weeks gestation, the rubella titer of a client indicates she is non-immune. When is the best time to administer a rubella vaccine to this client?

Early postpartum, within 72 hours of delivery.

During a 26-week gestation prenatal exam, a client reports occasional dizziness and lightheadness when she is lying down. What intervention is best for the nurse to recommend to this client.

Elevate the head with two pillows while sleeping.

During a childbirth preparation class, the nurse teacher discusses the importance of the "spurt" of energy that occurs before labor. Why is it important to conserve this energy?

Fatigue may influence pain medication requirements.

A client tells the nurse that the first day of her last menstrual period was July 22. What is the estimated date of birth (EDB)?

Her EDB is April 29. The Nägele rule is an indirect, noninvasive method for estimating the date of birth: EDB = last menstrual period + 1 year - 3 months + 7 days. May 7 is beyond the expected date of birth. April 22 and March 6 are both before the EDB.

A female client is undergoing treatment for infertility. After therapy with clomiphene the client comes for follow-up visits and no results are seen. What further treatment does the nurse anticipate administering?

Human chorionic gonadotropin

A client's labor has progressed to the point where she is 6 cm dilated; however, the fetal head is not engaged. An amniotomy is performed. After this procedure, the nurse checks the fetal heart rate. What other nursing action should be performed at this time?

Inspect the perinuem

A client expresses a desire to postpone her first pregnancy for at least 5 years. She smokes 1.5 packs of cigarettes a day, has never been pregnant, and does not want to use a barrier method. Which method does the nurse anticipate the primary healthcare provider will recommend?

Medroxyprogesterone long-acting progestin-only contraceptive that is less likely to cause cardiovascular problems in women who smoke than contraceptives containing estrogen might. Vaginal rings, combined oral contraceptive pills, and the birth control patch all contain estrogen and are not recommended for women who smoke.

Which drug is used to induce abortion?

Mifepristone

Which tocolytic agent can be used safely without combining with other agents to inhibit labor and maintain a pregnancy?

Nifedipine

During a routine prenatal office visit at 26 weeks' gestation, a client states that she is getting fat all over and that she even needed to buy bigger shoes. What is the next nursing action?

Obtaining the client's weight and blood pressure

The nurse is planning discharge teaching for a client who had an evacuation of gestational trophoblastic disease (GTD) two days ago. Which information is most important for the nurse to include in this client's teaching plan?

Oral contraceptive use for at least one year.

A pregnant client is prescribed heparin to prevent the risk of thromboembolism. Which adverse effects should the nurse anticipate with this medication?

Osteoporosis Compression fractures of the spine

A client who delivered an infant an hour ago tells the nurse that she feels wet underneath her buttock. The nurse notes that both perineal pads are completely saturated and the client is lying in a 6 inch diameter pool of blood. Which action should the nurse implement next? A. cleanse the perineum B. obtain a blood pressure C. palpate the firmness of the fundus D. inspect the perineum for lacerations.

Palpate the firmness of the fundus. A firm fundus is needed to control bleeding from the placental site of attachment on the uterine wall. The nurse should first assess for firmness and massage the fundus as indicated.

The nurse is caring for a client in the transition phase of labor. Which breathing pattern should the nurse instruct the client to use when there is an urge to push at 9 cm of dilation?

Panting-blowing pattern

Immediately after a client has completed the third stage of labor, the nurse adds the prescribed 10 units of oxytocin to the intravenous line. What is the desired response to this medication?

Prevention of the occurrence of profuse bleeding after placental separation

What goal is most important for the nurse to include in the plan of care for a client with gestational diabetes?

Restrict carbohydrate intake.

Which criteria may prove that a drug is a teratogen?

The drug may cause fetal malformations. The drug acts in the gestation period. The drug-related malformations increase with an increase in the dose and exposure.

A client had a cesarean birth 3 days ago. Where should the nurse, while palpating the client's fundus, expect to locate the fundus if each line represents 1 cm?

Three days after birth, the fundus should be located 3 cm below the umbilicus; 12 hours after birth it should be located about 1 cm above the umbilicus; each following day it drops 1 to 2 cm. Location a is too high; the fundal height is 1 cm below the umbilicus on day 1. Location b is too high; the fundal height is 2 cm below the umbilicus on day 2. Location d is too low; the fundal height is 4 cm below the umbilicus on day 4.

Which GI finding should the nurse be concerned about in a client at 28 weeks? A)Pyrosis B) PICA C) Ptyalism D) Decreased peristalsis

B

Which gestational period is appropriate for the administration of corticosteroids during preterm labor?

24-34 weeks

THe nurse is planning for the care of the 30 year old primigravida with pre-gestational diabetes. What is the most important factor affecting this client's pregnancy outcome? A) Degree of glycemic control during pregnancy B) Mother's age C) Amount of insulin required prenatally D) Number of years since diabetes was diagnosed

A

The nurse assess a high risk neonate under a radiant warmer who has an umbilical catheter and identifies that the neonates feet are blanched. What nursing should be implemented? A) Report findings to the Dr. B) Wrap feet loosely in a prewarmed blanket C) Elevate feet 15 degrees D) Place socks on infant

A

The nurse is teaching a primigravida at 10 weeks about the need to increase her folic acid intake. Which explanation should the nurse provide that supports preventative perinatal care? A) Adequate folic acid during embryogenesis reduces the incidence of neural tube defects B) The incidence of congenital heart defects is related to folic acid intake deficiencies C) Folic acid can significantly reduce the incidence of mental retardation D) The risk for neonatal cerebral palsy increases with folic acid deficiency during pregnancy

A

What nursing action should the nurse be implemented when intermittently gavage feeding a preterm a preterm infant? A) Allow formula to flow by gravity B) Apply steady pressure to the syringe C) Insert feeding tube through the nares D) Avoid letting infant suck on the tube

A

Which prescription should the nurse administer to the newborn to reduce complications related to birth trauma? A) Vitamin K B) Silver nitrate C) Ceftriaxone (Rocephin) D) Erythromycin

A

A postpartal client who is bottle feeding develops engorgement . What is the best recommendation for nurse to provide this client ? A. Avoid stimulation to breast and wear a tight bra B. Express small amount of breast milk in your hand C. Take a prescribed analgesic and express the breast to air D. Place warm packs on both sides of breast

A. Avoid stimulation to breast and wear tight bra

Which findings are most critical for the nurse to report to the primary health care provider when caring for the client during the last trimester of her pregnancy? (Select all that apply.) A. Increased heartburn that is not relieved with doses of antacids B. Increase of the fetal heart rate from 126 to 156 beats/min from the last visit C. Shoes and rings that are too tight because of peripheral edema in extremities D. Decrease in ability for the client to sleep for more than 2 hours at a time E. Chronic headache that has been lingering for a week behind the client's eyes

A & E (Possible signs of Preeclampsia) A. Increased heartburn that is not relieved with doses of antacids E. Chronic headache that has been lingering for a week behind the client's eyes

The nurse is assessing a full term newborn's breathing pattern. Which findings should the nurse assess further? (select all) A) Diaphragmatic with chest retractions B) Grunting heard with stethescope C) Chest breathing with nasal flaring D) HR of 158 E) Abdominal with synchronous chest movements F) Shallow with an irregular rhythm

A B C

A client at 43 weeks' gestation has just given birth to an infant with typical postmaturity characteristics. Which signs of postmaturity does the nurse identify?

A client at 43 weeks' gestation has just given birth to an infant with typical postmaturity characteristics. Which postmature signs does the nurse identify? (Select all that apply.)

Which findings are of most concern to the nurse when caring for a woman in the first trimester of pregnancy? (Select all that apply.) A. Cramping with bright red spotting B. Extreme tenderness of the breast C. Lack of tenderness of the breast D. Increased amounts of discharge E. Increased right-side flank pain

A, C, E A. Cramping with bright red spotting C. Lack of tenderness of the breast E. Increased right-side flank pain

An ambulatory client at 39-weeks gestation presents to the emergency center with an obvious injury to her arm that occurred as the result of a fall. Which concurrent symptom is a priority for the nurse to assess. A. Ecchymotic knees. B. Dribbling urine. C. 1+ pedal edema. D. Pain in the forearm.

A. Ecchymotic knees.

A one-day-old neonate develops a cephalohematoma. The nurse should closely assess this neonate for which common complication? A. jaundice B. poor appetite C. brain damage D. hypoglycemia

A. jaundice

A client in active labor is admitted with preeclampsia. Which assessment finding is most significant in planning this client's care? A. patellar reflex 4+ B. blood pressure 158/80 C. four-hour urine output 240 mL D. respiration 12/minute

A. patellar reflex 4+. a 4+ reflex in a client with pregnancy-induced hypertension indicates hyperreflexia, which is an indication of impending seizure.

Which findings are of most concern to the nurse when caring for a woman in the first trimester of pregnancy? (Select all that apply.) A.Cramping with bright red spotting B.Extreme tenderness of the breast C.Lack of tenderness of the breast D.Increased amounts of discharge E.Increased right-side flank pain

A.Cramping with bright red spotting C.Lack of tenderness of the breast E.Increased right-side flank pain Rationale: Options A and C are signs of a possible miscarriage. Cramping with bright red bleeding is a sign that the client's menstrual cycle is about to begin. A decrease of tenderness in the breast is a sign that hormone levels have declined and that a miscarriage is imminent. Option E could be a sign of an ectopic pregnancy, which could be fatal if not discovered in time before rupture. Options B and D are normal signs during the first trimester of a pregnancy.

Client teaching is an important part of the perinatal nurse's role. Which factor has the greatest influence on successful teaching of the pregnant client? A.The client's investment in what is being taught B.The couple's highest levels of education C.The order in which the information is presented D.The extent to which the pregnancy was planned

A.The client's investment in what is being taught Rationale: When teaching any client, readiness to learn is related to how much the client has invested in what is being taught or how important the materials are to the client's particular life. For example, the client with severe morning sickness in the first trimester may not be ready to learn about labor and delivery but is probably very ready to learn about ways to relieve morning sickness. Options B and C are factors that may influence learning but are not as influential as option A. Even if a pregnancy is planned and very desirable, the client must be ready to learn the content presented.

A pregnant woman who is in the third trimester arrives in the emergency department with vaginal bleeding. She states that she snorted cocaine approximately 2 hours ago. Which complication does the nurse suspect as the cause of the bleeding?

Abruptio placenta

A 16-year-old primigravida who appears to be at or close to term arrives at the emergency department stating that she is in labor and complaining of pain continuing between contractions. The nurse palpates the abdomen, which is firm and shows no sign of relaxation. What problem does the nurse conclude that the client is experiencing?

Abruptio placentae

A mental health nurse is admitting a client with anorexia nervosa. When obtaining the history and physical assessment, the nurse expects the client's condition to reveal which symptom?

Amenorrhea

1. Wrap the infant's foot with a heel warmer for 5 minutes 2. Collect a spring-loaded automatic puncture device 3. Restrain the newborn's foot with your free hand 4. Cleanse puncture site on the lateral aspect of the heel

An infant born at 37 weeks gestation, weighing 4.1 kg is 2 hours old and appears large for gestational age, flushed, and tremulous. What procedure should the nurse follow to implement? (4 steps)

A client at 8 weeks gestation asks the nurse about the risk for congenital heart defects in her baby. Which response best explains why these defects occur? A) they usually occur in the first trimester B) The heart develops in the third - fifth weeks after conception C) it depends on what the causative factors are for the defect D) We really don't know why they occur.

B

An infant with hyperbilirubinemia is receiving phototherapy. What intervention should the nurse implement? A) Change t-shirt every 3 hours B) Monitor temperature C) Maintain NPO status D) Apply skin lotion as prescribed

B

During a prenatal counseling session for women trying to get pregnant in 3-6 months, what information should the nurse provide? A) Lose weight so more weight is gained during pregnancy B) Make sure to include adequate folic acid in the diet C) Discontinue all forms of contraception D) Continue to take any medications that are taken regularly

B

The mother of a neonate asks the nurse why it is so important to keep the infant warm. What information should the nurse provide? A) The kidneys and renal function are not fully developed B) A large body surface area favors heat loss to the enviroment C) Warmth promotes sleep so the infant will grow quickly D) The thick layer of subQ fat is inadequate for insulin

B

A 41-week multigravida is receiving oxytocin (Pitocin) to augment labor. Contractions are firm and occurring every 5 minutes, with a 30- to 40-second duration. The fetal heart rate increases with each contraction and returns to baseline after the contraction. Which action should the nurse implement? A.Place a wedge under the client's left side. B.Determine cervical dilation and effacement. C.Administer 10 L of oxygen via facemask. D.Increase the rate of the oxytocin (Pitocin) infusion.

B.Determine cervical dilation and effacement. Rationale: The goal of labor augmentation is to produce firm contractions that occur every 2 to 3 minutes, with a duration of 60 to 70 seconds, and without evidence of fetal stress. FHR accelerations are a normal response to contractions, so the oxytocin (Pitocin) infusion should be increased per protocol to stimulate the frequency and intensity of contractions. Options A and C are indicated for fetal stress. A sterile vaginal examination places the client at risk for infection and should be performed when the client exhibits signs of progressing labor, which is not indicated at this time.

A newborn who was a breech presentation is admitted to the nursery. Which assessment procedure is a priority for the nurse to perform?

Babinski's reflex.

A 31-year-old woman uses an over-the-counter pregnancy test that is positive one week after a missed period. At the clinic the client tells the nurse she takes epilepsy medication, has a hx of irregular periods, is under stress at work, and has not been sleeping well. The client's physical exam and ultrasound do not indicate that she is pregnant. How should the nurse explain the most likely cause for obtaining a false-positive? A) being under too much stress at work B)Having an irregular menstrual cycle C) Using an anticonvulsant for epilepsy D) Taking the pregnancy test too early

C

A client at 28 weeks is concerned about her weight gain of 17 lbs. What information should the nurse provide to the client? A) Try to exercise more because too much weight has been gained B) Increase the calories in your diet to gain more weight per week C) The weight gain is acceptable for the number of weeks pregnant D) It is not necessary to keep such a close watch on weight gain

C

A client at 29 weeks gestation with possible placental insufficiency is being prepared for prenatal testing. Information about which diagnostic study should the nurse provide information to the client? A) Aminocentisis B) Maternal serum alpha fetoprotein C) Ultrasonography D) Chronic villus sampling

C

A neonate who is receiving an exchange transfusion for hemolytic disease develops respiratory distress, tachycardia, and a cutaneous rash. What nursing intervention should be implement first? A) Monitor vital signs electronically B) Inform the Dr. C) Stop the transfusion D) Administer calcium gluconate

C

A newborn infant is jaundiced due to Rh incompatibility. Which finding is most important for the nurse to report to the healthcare provider? A) Bruising B) Oral intake C) Bilirubin D) Hemoglobin

C

A woman who is bottle feeding her newborn infant calls the clinic 72 hours after delivery and tells the nurse that both of her breasts are swollen, warm, and tender. What instructions should the nurse give? A) Wear a loose fitting bra B) Run warm water on the breasts during a shower C) Apply ice to the breasts D) express small amounts of milk from the breast

C

The apnea monitor alarm sounds for the third time during one shift for a neonate who was delivered at 37 weeks gestation. What nursing action should be implemented first? A) Administer 100% oxygen B) Provide tactile stimulation C) Evaluate the newborns color and respiration D) Assess the functionality of the monitoring device

C

The nurse is assessing a 12 hour old infant with maternal history of frequent alcohol consumption during pregnancy. Which finding should the nurse report that is most suggestive of fetal alcohol syndrome? A) An extra digit on the left hand B) Asymmetrical bulging fontanels C) Flat nasal bridge D) Corneal clouding

C

The nurse notes an irregular bluish hue on the sacral area of a 1 day old hispanic infant. How should the nurse document this finding? A) Harlequin sign B) Acrocyanosis C) Mongolian spots D) Erythema toxicum

C

A client who delivered a healthy infant 5 days ago calls the clinic nurse and reports that her lochia is getting lighter in color. Which action should the nurse take? A.Instruct the client to go to the emergency room. B.Recommend vaginal douching. C.Explain this is a normal finding. D.Determine if ovulation has occurred.

C.Explain this is a normal finding. Rationale:The client is describing lochia serosa, a normal change in the lochial flow. Options A, B, and D are not recommended for this normal finding.

The nurse instructs a laboring client to use accelerated blow breathing. The client begins to complain of tingling fingers and dizziness. Which action should the nurse take? A.Administer oxygen by facemask. B.Notify the health care provider of the client's symptoms. C.Have the client breathe into her cupped hands. D.Check the client's blood pressure and fetal heart rate.

C.Have the client breathe into her cupped hands. Rationale: Tingling fingers and dizziness are signs of hyperventilation (blowing off too much carbon dioxide). Hyperventilation is treated by retaining carbon dioxide. This can be facilitated by breathing into a paper bag or cupped hands. Option A is inappropriate because the carbon dioxide level is low, not the oxygen level. Options B and D are not specific for this situation.

When preparing a class on newborn care for expectant parents, which is correct for the nurse to teach concerning the newborn infant born at term gestation? A.Milia are red marks made by forceps and will disappear within 7 to 10 days. B.Meconium is the first stool and is usually yellow gold in color. C.Vernix is a white cheesy substance, predominantly located in the skin folds. D.Pseudostrabismus found in newborns is treated by minor surgery.

C.Vernix is a white cheesy substance, predominantly located in the skin folds. Rationale: Vernix, found in the folds of the skin, is a characteristic of term infants. Milia are not red marks made by forceps but are white pinpoint spots usually found over the nose and chin that represent blockage of the sebaceous glands. Meconium is the first stool, but it is tarry black, not yellow. Pseudostrabismus (crossed eyes) is normal at birth through the third or fourth month and does not require surgery.

A primigravida arrives at the observation unit of the maternity unit because thinks is in labor. The nurse applies the external fetal heart monitor and determines that the fetal heart rate is 140 beats/minute and the contractions are occurring irregularly every 10 to 15 minutes. What assessment finding confirms to the nurse that the client is not labor at this time?

Contractions decrease with walking.

A multigravida woman at 35 weeks is diagnosed with pregnancy induced hypertension. Which symptom should the nurse instruct the client to report immediately? A) Increased urine output B) Constipation C) Backache D) Blurred vision

D

A primigravida at 12 weeks tells the nurse that she does not like dairy products. Which food should the nurse recommend to increase the client's calcium intake? A) Canned clams B) Fresh Apricots C) Spaghetti with meat sauce D) Canned sardines

D

26. Which action should the nurse implement when caring for a newborn immediately after birth? A. Administer eye prophylaxis and vitamin K. B. Foster parent-newborn attachment. C. Dry the newborn and wrapping in a blanket. D. Keep the newborn's airway clear.

D. Keep the newborn's airway clear.

A woman is being seen in the prenatal clinic at 36 weeks' gestation. The nurse is reviewing signs and symptoms that should be reported to the primary healthcare provider with the mother. Which signs and symptoms require further evaluation by the primary healthcare provider?

Decreased urine output Blurred vision with spots Contractions that are regular and 5 minutes apart

An ultrasound scan of a 7-month-pregnant woman indicates fetal death. Which drug should be given to the client to evacuate the contents of the uterus?

Dinoprostone

Which medication is indicated for evacuation in case of a miscarriage?

Dinoprostone

The current vital signs for a primipara who delivered vaginally during the previous shift are: temperature 100.4 F, heart rate 58 beats/minute, respiratory rate 16 breaths/minute, and blood pressure 130/74. What action should the nurse implement?

Document the vital signs in the record.

The nurse is providing care to a client with preeclampsia who is receiving magnesium sulfate 2 g/hr. The nurse receives a call from the laboratory technician indicating that the client has a magnesium level of 6.4 mEq/L (0.30 mmol/L). What is the next nursing action?

Documenting the level in the client's electronic medical record

A primigravida at 8 weeks' gestation is visiting the prenatal clinic for the first time. Which finding would an assessment reveal at this time?

Goodell's sign

Artificial rupture of the membranes of a laboring client reveals meconium-stained fluid. What intervention has the greatest priority?

Have a meconium aspirator available at delivery.

A nurse is caring for a pregnant client with thrombophlebitis. Which anticoagulant medication may be prescribed?

Heparin and Enoxaprin

The nurse is caring for a client in active labor with a history of T5 spinal cord injury. Which of the following findings indicates to the nurse that the client is experiencing a complication of the labor process?

Increased blood pressure

When assessing a newly admitted primigravida in labor, a nurse determines that the fetal heartbeat is loudest in the upper left quadrant. What fetal position does the nurse identify?

Left Sacral anterior

An expectant couple asks the nurse about the cause of low back pain in labor. The nurse replies that this pain occurs most often when the fetus is in what position?

Occiput posterior

A newborn with myelomeningocele is admitted to the neonatal intensive care unit. Which preoperative nursing intervention should the nurse implement first?

Place the infant on the abdomen to protect the sac.

A multiparous client presents to the labor and delivery area in active labor. The initial vaginal examination reveals that the cervix is dilated 4 cm and 100% effaced. Two hours later the client experiences rectal pressure, followed by delivery 5 minutes later. How is this delivery best documented?

Precipitous vaginal delivery

While a client is being given intravenous magnesium sulfate therapy for preeclampsia, it is essential for the nurse to monitor the client's deep tendon reflexes. What reason does the nurse give to the client to explain why this is done?

Reveals the potential for respiratory depression

A pregnant client's history reveals opioid abuse. What is the nurse's initial plan for providing pain relief measures during labor?

Scheduling pain medication at regular intervals

The nurse is providing discharge instructions to the parents of a child who has undergone surgical correction of hypospadias. What is the priority information for the nurse to include?

Teaching parents how to care for the catheterization system

What issues are associated with the difficulty in identification of teratogens?

Teratogenic effects may be delayed. Prolonged drug exposure may be required. Animal test results may not be applicable to humans.

The postpartum nurse is providing care to four maternal/infant couplets who have all delivered within the past 24 hours. After receiving the handoff report from the off-going nurse, which client is a priority for the nurse to see first?

The term infant with a transcutaneous bilirubin reading of 8.6 mg/dL 12 hours after birth

The nurse is teaching a woman how to use her basal body temperature (BBT) pattern as a tool to assist her in conceiving a child. Which temperature pattern indicates the occurrence of ovulation, and therefor, the best time for intercourse to ensure conception? a. Between the time the temperature falls and rises b. Between 36 and 48 hours after the temperature rises c. When the temperature falls and remains low for 36 hours d. Within 72 hours before the temperature falls

a. Between the time the temperature falls and rises In most women, the BBT drops slightly 24 to 36 hours before ovulation and rises 24 to 72 hours after ovulation, when the corpus luteum of the ruptured ovary produces progesterone. Therefore, intercourse between the time of the temperature fall and rise (A) is the best time for conception.

A multigravida client arrives at the labor and delivery unit and tells the nurse that her bag of water has broken. The nurse identifies the presence of meconium fluid on the perineum and determines the fetal heart rate is between 140 to 150 beats/minute. What action should the nurse implement next? a. Ccmplete a sterile vaginal exam b. Take maternal temperature every 2 hours c. Prepare for an immediate cesarean birth d. Obtain sterile suction equipment

a. Complete a sterile vaginal exam A vaginal exam (A) should be performed after the rupture of membranes to determine the presence of a prolapsed cord.

A pregnant client with mitral stenosis Class III is prescribed complete bedrest. The client asks the nurse, "Why must I stay in bed all the time?" Which response is best for the nurse to provide this client? a. Complete bedrest decreases oxygen needs and demands on the heart muscle b. We want your baby to be healthy, and this is the only way we can make sure that will happen again c. I know you're upset. Would you like to talk about somethings you could so while in bed? d. Labor is difficult and you need to use this time to rest before you have to assume all child-caring duties

a. Complete bedrest decreases oxygen needs and demands on the heart muscle tissue To help preserve cardiac reserves, the woman may need to restrict her activities and complete bedrest is often prescribes (A).

A healthcare provider informs the charge nurse of a labor and delivery unit that a client is coming to the unit with suspected abruptio placentae. What findings should the charge nurse expect the client to demonstrate? (Select all that apply) a. Dark, red vaginal bleeding b. Lower back pain c. Premature rupture of membranes d. Increased uterine irritability e. Bilateral pitting edema f. A rigid abdomen

a. Dark, red vaginal bleeding d. Increased uterine irritability f. A rigid abdomen The symptoms of abruptio placentae include dark red vaginal bleeding (A), increased uterine irritability (D), and a rigid abdomen (F).

A client in active labor complains of cramps in her leg. What intervention should the nurse implement? a. Ask the client if she takes a daily calcium tablet b. Extend the leg and dorsiflex the foot c. Lower the leg off the side of the bed d. Elevate the leg above the heart

b. Extend the leg and dorsiflex the foot Dorsiflexing the foot by puching the sole of the foot forward or by stnading (if the client is capable) (B), and putting the heel of the foot on the floor is the best means of relieving leg cramps.

Which assessment finding should the nursery nurse report to the pediatric healthcare provider? a. Blood glucose level of 45 mg/dl b. Blood pressure of 82/45 mmHg c. Non-bulging anterior fontanel d. Central cyanosis when crying

d. Central cyanosis when crying An infant who demonstrates central cyanosis when crying (D) is manifesting poor adaptation to extrauterine life which should be reported to the healthcare provider for determination of a possible underlying cardiovascular problem.

A client is admitted in active labor at 39 weeks' gestation. During the initial examination the nurse identifies multiple red blisterlike lesions on the edges of the client's vaginal orifice. Once the nurse has spoken to the primary healthcare provider and received prescriptions, what is the priority nursing action?

preparing for a ceserean

A pregnant client tells the nurse that she thinks she has developed an allergy because her nose is often very congested and she has difficulty breathing. How should the nurse reply?

"That is an expected occurrence; the increased hormones are responsible for the congestion."

The client comes to the hospital assuming she is in labor. Which assessment findings by the nurse would indicate that the client is in true labor? (Select all that apply.) A. Pain in the lower back that radiates to abdomen B. Contractions decreased in frequency with ambulation C. Progressive cervical dilation and effacement D. Discomfort localized in the abdomen E. Regular and rhythmic painful contractions

A,C,E A. Pain in the lower back that radiates to abdomen C. Progressive cervical dilation and effacement E. Regular and rhythmic painful contractions

A woman who thinks she could be pregnant calls her neighbor, a nurse, to ask when she could use a home pregnancy test to diagnose pregnancy. Which response is appropriate? A. "A home pregnancy test can be used right after your first missed period." B. "These tests are most accurate after you have missed your second period." C. "Home pregnancy tests often give false positives and should not be trusted." D. "The test can provide accurate information when used right after ovulation."

A. "A home pregnancy test can be used right after your first missed period."

Which statement made by the client indicates that the mother understands the limitations of breastfeeding her newborn? A. "Breastfeeding my infant consistently every 3 to 4 hours stops ovulation and my period." B. "Breastfeeding my baby immediately after drinking alcohol is safer than waiting for the alcohol to clear my breast milk." C. "I can start smoking cigarettes while breastfeeding because it will not affect my breast milk." D. "When I take a warm shower after I breastfeed, it relieves the pain from being engorged between breastfeedings."

A. "Breastfeeding my infant consistently every 3 to 4 hours stops ovulation and my period."

Which statement made by the client indicates that the mother understands the limitations of breastfeeding her newborn? A. "Breastfeeding my infant consistently every 3 to 4 hours stops ovulation and my period." B. "Breastfeeding my baby immediately after drinking alcohol is safer than waiting for the alcohol to clear my breast milk." C. "I can start smoking cigarettes while breastfeeding because it will not affect my breast milk." D. "When I take a warm shower after I breastfeed, it relieves the pain from being engorged between breastfeedings."

A. "Breastfeeding my infant consistently every 3 to 4 hours stops ovulation and my period."

A pregnant client with mitral stenosis Class III is prescribed complete bedrest. The client asks the nurse, "Why must I stay in bed all the time?" Which response is best for the nurse to provide the client? A. "Complete bedrest decreases oxygen needs and demands on the heart muscle tissue." B. "We want your baby to be healthy, and this is the only way we can make sure that will happen." C. "I know you're upset. Would you like to talk about some things you could do while in bed." D. "Labor is difficult and you need to use this time to rest before you have to assume all child-caring duties."

A. "Complete bedrest decreases oxygen needs and demands on the heart muscle tissue."

42. The father of a newborn tells the nurse, "My son just died." How should the nurse respond? A. "I am sorry for your loss." B. "I understand how you feel." C. "There is an angel in heaven." D. "You can have other children."

A. "I am sorry for your loss."

A new mother asks the nurse about an area of swelling on her baby's head near the posterior fontanel that lies across the suture line. How should the nurse respond? A. "This is called caput succedaneum. It will absorb and cause no problems." B. "This is called caput succedaneum. It will have to be drained." C. "This is called a cephalhematoma. It will cause no problems." D. "This is called cephalhematome. It can cause jaundice as it is absorbed."

A. "This is called caput succedaneum. It will absorb and cause no problems."

A client at 32 weeks gestation is diagnosed with preeclampsia. Which assessment finding is most indicative of an impending convulsion? A. 3+ deep tendon reflexes and hyperclonus B. periorbital edema, flashing lights, and aura C. epigastric pain in the third trimester D. recent decreased urinary output

A. 3+ deep tendon reflexes and hyperclonus

60. A client at 39-weeks gestation is admitted to the labor and delivery unit. Her obstetrical history includes 3 live births at 39-weeks, 34-weeks, and 35-weeks gestation. Using the GTPAL system, which designation is the most accurate summary of the clients obstetrical history? A. 4-1-2-0-3. B. 3-1-1-1-3. C. 4-3-1-0-2. D. 3-0-3-0-3.

A. 4-1-2-0-3.

40. The nurse on the postpartum unit receives a report for 4 clients during change of shift. Which client should the nurse assessed for risk of postpartum hemorrhage (PPH)? A. A multiparous client receiving magnesium sulfate during induction for severe preeclampsia. B. A primiparous client who had an emergency cesarean birth due to fetal distress. C. A multigravida who delivered an 8 pound 2 ounce infant after an 8-hour labor. D. A primigravida who had a spontaneous birth of preterm twins.

A. A multiparous client receiving magnesium sulfate during induction for severe preeclampsia.

A multigravida full term , laboring client complains of back labor. Vaginal examine reveals that the client is 3cm with 50% effacement , and the fetal head is at -1 station. What action should the nurse implement first? A. Apply counter - pressure to the sacral area B. Turn the client lateral position C. Notify the scrub nurse to prepare the OR D. Ambulate the client between contractions

A. Apply counter - pressure to the sacral area

The nurse's assessment of a preterm infant reveals decreased muscle tone, signs of respiratory difficulty, irritability, and mottled, cool skin. Which intervention should the nurse implement first? A. Assess the infant's blood glucose level. B. Nipple feed 1oz 5% glucose in water. C. Place the infant in a side-lying position. D. Position a radiant warmer over the crib.

A. Assess the infant's blood glucose level.

The nurse is teaching a new mother about diet and breastfeeding. Which instruction is most important to include in the teaching plan? A. Avoid alcohol because it is excreted in breast milk. B. Eat a high-roughage diet to help prevent constipation. C. Increase caloric intake by approximately 500 cal/day. D. Increase fluid intake to at least 3 quarts each day.

A. Avoid alcohol because it is excreted in breast milk.

The nurse is teaching a new mother about diet and breastfeeding. Which instruction is most important to include in the teaching plan? A. Avoid alcohol because it is excreted in breast milk. B. Eat a high-roughage diet to help prevent constipation. C. Increase caloric intake by approximately 500 cal/day. D. Increase fluid intake to at least 3 quarts each day.

A. Avoid alcohol because it is excreted in breast milk.

The nurse is interacting with a female client who is diagnosed with postpartum depression. Which finding should the nurse document as an objective signs of depression? (Select all that apply.) A. Avoids eye contact. B. Interacts with a flat affect. C. Reports feeling sad. D. Expresses suicidal thoughts. E. Has a disheveled appearance.

A. Avoids eye contact. B. Interacts with a flat affect. C. Reports feeling sad. D. Expresses suicidal thoughts.

The nurse is planning care for a client at 30-weeks gestation who is experiencing preterm labor. What maternal prescription is most important in preventing this fetus from developing respiratory distress syndrome? A. Betamethasone (Celestone) 12 mg deep IM B. Butorphanol 1 mg IV push q2h PRN pain C. Ampicillin 1 Gram IV push q8h D. Terbutaline (Brethine) 0.25 mg subcutaneously q15 minutes x3

A. Betamethasone (Celestone) 12 mg deep IM

A breastfeeding postpartum client is diagnosed with mastitis, and antibiotic therapy is prescribed. Which instruction should the nurse provide to this client? A. Breastfeed the infant, ensuring that both breasts are completely emptied. B. Feed expressed breast milk to avoid the pain of the infant latching onto the infected breast. C. Breastfeed on the unaffected breast only until the mastitis subsides. D. Dilute expressed breast milk with sterile water to reduce the antibiotic effect on the infant.

A. Breastfeed the infant, ensuring that both breasts are completely emptied.

A breastfeeding postpartum client is diagnosed with mastitis, and antibiotic therapy is prescribed. Which instruction should the nurse provide to this client? A. Breastfeed the infant, ensuring that both breasts are completely emptied. B. Feed expressed breast milk to avoid the pain of the infant latching onto the infected breast. C.Breastfeed on the unaffected breast only until the mastitis subsides. D. Dilute expressed breast milk with sterile water to reduce the antibiotic effect on the infant.

A. Breastfeed the infant, ensuring that both breasts are completely emptied.

72. A primigravida at 12-weeks gestation tells the nurse that she does not like dairy products. Which food should the nurse recommend to increase the clients calcium intake? A. Canned sardines. B. Spaghetti with meat sauce. C. Canned clams. D. Fresh apricots.

A. Canned sardines.

Twenty-four hours after admission to the newborn nursery, a full-term male infant develops localized swelling on the right side of his head. In a newborn, what is the most likely cause of this accumulation of blood between the periosteum and skull that does not cross the suture line? A. Cephalhematoma, which is caused by forceps trauma B. Subarachnoid hematoma, which requires immediate drainage C. Molding, which is caused by pressure during labor D. Subdural hematoma, which can result in lifelong damage

A. Cephalhematoma, which is caused by forceps trauma

36. A client at 28-weeks gestation experiences blunt abdominal trauma. Which parameter should the nurse assess first for signs of internal hemorrhage? A. Changes in fetal heart rate patterns. B. Alteration and maternal blood pressure. C. Complains of abdominal pain. D. Vaginal bleeding.

A. Changes in fetal heart rate patterns.

One day after a vaginal delivery of a full-term baby, a postpartum client's white blood cell count is 15,000/mm2. What action should the nurse take first? A. Check he differential, since the WBC is normal for this client. B. Assess the clients temperature, pulse, and respirations q4h. C. Notify the healthcare provider, since this finding is indicative of infection D. Assess the clients perineal area for signs of a perineal hematoma.

A. Check he differential, since the WBC is normal for this client.

A client at 28 weeks of gestation calls the antepartal clinic and states that she has just experienced a small amount of vaginal bleeding, which she describes as bright red. The bleeding has subsided. She further states that she is not experiencing any uterine contractions or abdominal pain. What instruction should the nurse provide? A. Come to the clinic today for an ultrasound. B. Go immediately to the emergency department. C. Lie on your left side for about 1 hour and see if the bleeding stops. D. Take a urine specimen to the laboratory to see if you have a urinary tract infection (UTI).

A. Come to the clinic today for an ultrasound.

68. A client who is breastfeeding engorged breasts on the third postpartum day. Which action should the nurse recommend to relieve breast engorgement? A. Continue breastfeeding every 2 hours. B. Skip a feeding to rest the breasts. C. Avoid pumping her breasts. D. Decreased fluid intake for at least 24 hours.

A. Continue breastfeeding every 2 hours.

During a routine first trimester prenatal exam, a pregnant client tells the nurse that she has noticed an increase in vaginal discharge that is white, thin, and watery. What action should the nurse implement? A. Inform her that this is a normal physiological change. B. Notify the healthcare provider of the complaint. C. Recommend an over-the-counter yeast medication. D. Prepare the client for a sterile speculum exam.

A. Inform her that this is a normal physiological change.

A primigravida arrives at the observation unit of the maternity unit because she thinks she is in labor. The nurse applies the external fetal heart monitor and determines that the fetal heart rate is 140 beats/minute and contractions are occurring irregularly every 10-15 minutes. Which assessment finding confirms to the nurse that the client is not in labor at this time? A. Contractions decrease with walking. B. 2+ pitting edema in lower extremities. C. Cervical dilations is 1cm. D. Membranes are intact.

A. Contractions decrease with walking.

5. When assessing a newborn infant's heart rate, which technique is most important for the nurse to use? A. Count the heart rate for at least one full minute. B. Quiet the infant before counting the heart rate. C. Palpate the umbilical cord. D. Listen at the apex of the heart.

A. Count the heart rate for at least one full minute.

Which finding(s) is (are) of most concern to the nurse when caring for a woman in the first trimester of pregnancy? (Select all that apply.) A. Cramping with bright red spotting B. Extreme tenderness of the breast C. Lack of tenderness of the breast D. Increased amounts of discharge E. Increased right-side flank pain

A. Cramping with bright red spotting C. Lack of tenderness of the breast E. Increased right-side flank pain

33. A nurse is planning for the care of a 30-year-old primigravida with pre-gestational diabetes. What is the most important factor affecting the clients pregnancy outcome? A. Degree of glycemic control during pregnancy. B. Mother's age. C. Amount of insulin required prenatally. D. Number of years since diabetes was diagnosed.

A. Degree of glycemic control during pregnancy.

At 6 weeks gestation, the rubella titer of a client indicates she is non-immune. When is the best time to administer a rubella vaccine to this client? A. Early postpartum, within 72hrs of delivery. B. Immediately, at 6-weeks gestation, to protect this fetus. C. After the client reaches 20-weeks gestation. D. After the client stops breastfeeding.

A. Early postpartum, within 72 hours of delivery.

A full term infant is admitted to the newborn nursery 2 hours after delivery. The delivery record reports that the mother is positive for HIV and received AZT intravenously during labor. What action should the nurse implement first? A. Ensure that AZT is given within 6 hours after birth B. Collect venous specimen for serum glucose level C. Asses for the presence of the Moro reflex D. Obtain consent for the Hep B vaccine

A. Ensure that AZT is given within 6 hours after birth

57. The apnea monitor alarm sounds for the third time during one shift for a neonate who was delivered at 37-weeks gestation. What nursing action should be implemented first? A. Evaluate the newborn's color and respirations. B. Assess the functionality of the monitoring device. C. Provide tactile stimulation. D. Administer flow by 100% oxygen.

A. Evaluate the newborn's color and respirations.

A mother of a 3-year-old boy has just given birth to a new baby girl. The little boy asks the nurse, "Why is my baby sister eating my mommy's breast?" How should the nurse respond? (Select all that apply) A. Explain that newborns get milk from their mothers in this way B. Reassure the older brother that it does not hurt his mother C. Remind him that his mother breastfed him too D. Suggest that the baby can also drink from a bottle E. Clarify that breastfeeding is his mother's choice

A. Explain that newborns get milk from their mothers in this way B. Reassure the older brother that it does not hurt his mother C. Remind him that his mother breastfed him too

70. The nurse is assessing a 12-hour-old infant with a maternal history of frequent alcohol consumption during pregnancy. Which findings should the nurse report that is most suggestive of fetal alcohol syndrome (FAS)? A. Flat nasal bridge. B. An extra digit on the left hand. C. Asymmetrical bulging fontanels. D. Corneal clouding.

A. Flat nasal bridge.

A newborn's assessment reveals spina bifida occulta. Which maternal factor should the nurse identify as having the greatest impact on the development of this newborn complication? A. Folic acid deficiency B. Preeclampsia C. Tobacco use D. Short interval pregnancy

A. Folic acid deficiency

A 30-year-old primigravida delivers a 9-pound (4082 gram) infant vaginally after a 30-hour labor. What is the priority nursing action for this client? A. Gently massage the fundus every 4 hours. B. Observe for signs of uterine hemorrhage. C. Encourage direct contact with the infant. D. Assess the blood pressure for hypertension.

A. Gently massage the fundus every 4 hours.

The nurse is counseling a patient who is at 6 weeks gestation and is experiencing morning sickness , but does not want to take any drugs for this discomfort. Which herbal supplement is likely to help this client with the nausea she is experiencing ? A. Ginger B. Chamomile C. Peppermint D. Ginko

A. Ginger

28. The nurse is assessing a full-term newborn's breathing pattern. Which findings should the nurse assess further? *(Select all that apply.)* A. Grunting heard with a stethoscope. B. Diaphragmatic with chest retraction. C. Abdominal with synchronous chest movements. D. Shallow with an irregular rhythm. E. Chest breathing with nasal flaring. F. Rate of 58 breaths per minute.

A. Grunting heard with a stethoscope. B. Diaphragmatic with chest retraction. E. Chest breathing with nasal flaring.

A pregnant client presents to the antepartum clinic complaining of brownish vaginal bleeding. The nurse notes that she has a greatly enlarges uterus and is complaining of severe nausea. The client reports that her period was "about 2 and a half months ago". Vital signs are: temperature 98.7F, pulse rate 70bpm, rr 18, and bp 190/110 mmHg. Based on these findings, what laboratory value should the nurse review? A. HcG values. B. Hematocrit. C. Vaginal secretions culture. D. Glucose in the urine.

A. HcG values.

A new mother who is breastfeeding her 4-week-old infant and has type 1 diabetes, reports that her insulin needs have decreased since the birth of her child. Which action should the nurse implement? A. Inform her that a decreased need for insulin occurs while breastfeeding. B. Counsel her to increase her caloric intake. C. Advise the client to breastfeed more frequently. D. Schedule an appointment for the client with the diabetic nurse educator.

A. Inform her that a decreased need for insulin occurs while breastfeeding.

61. A client delivers her first infant and asks the nurse if her skin changes from pregnancy are permanent. Which change should the nurse tell the client that will remain after pregnancy? A. Striae gravidarum. B. Chloasma. C. Vascular spiders. D. Pruritus.

A. Striae gravidarum.

While assessing a 40-week gestation primigravida in active labor, the client's membranes rupture spontaneously and the nurse notices that the amniotic fluid is meconium stained. Which additional finding is most important for the nurse to report to the healthcare provider? A. Maternal blood pressure of 130/85 mmHg. B. Fetal heart rate of 100 to 110 bpm. C. Vaginal exam reveals a cervix 6cm dilated. D. Contractions occurring every 2-3 minutes.

A. Maternal blood pressure of 130/85 mmHg.

A client delivers a viable infant , but begins to have excessive uncontrolled vaginal .. notifying the healthcare provider of the clients condition ,What information is most important A. Maternal blood pressure B. Maternal apical pulse C. Time pitocin infusion completed D. Total amount of pitocin infused

A. Maternal blood pressure.

20. When discussing birth in a home setting with a group of pregnant women, which situation should the nurse include about the safety of a home birth? A. Medical back up should be available quickly in case of complications. B. The women's extended family should be allowed to attend the home birth. C. Only the woman and her midwife should be present during the delivery. D. The woman should live no more than 15 minutes from the hospital.

A. Medical back up should be available quickly in case of complications.

62. The nurse administers meperidine (Demerol) 25 mg IV push to a laboring client, who delivers the infant 90 minutes later. What medication should the nurse anticipate administering to the infant? A. Naloxone (Narcan). B. Nalbuphine (Nubain). C. Promethazine (Phenergan). D. Fentanyl (Sublimaze).

A. Naloxone (Narcan).

An unlicensed assistive personnel (UAP) reports to the charge nurse that a client who delivers a 7-pound infant 12 hours ago is reporting a severe headache. The client blood pressure is 110/70 mmHg, respiratory rate is 18 breaths/minute, heart rate is 74 bpm, and temperature is 98.6F. The client's fundus is firm and one fingerbreadth above the umbilicus. Which action should the charge nurse implement first? A. Notify the healthcare provider of the assessment findings. B. Obtain a STAT hemoglobin and hematocrit. C. Assign a practical nurse (PN) to reassess the client's vital signs. D. Determine if the client received anesthesia during delivery.

A. Notify the healthcare provider of the assessment findings.

On admission to the prenatal clinic, a 23-year old woman tells the nurse that her last menstrual period began on February 15, and that previously her periods were regular. Her pregnancy test is positive. This client's expected date of delivery (EDD) would be A. November 22 B. November 8 C. December 22 D. October 22

A. November 22

On admission to the prenatal clinic, a client tells the nurse that her last menstrual period began on February 15 and that previously her periods were regular (28-day cycle). Her pregnancy test is positive. What is this client's expected date of birth (EDB)? A. November 22 B. November 8 C. December 22 D. October 22

A. November 22

A school nurse is preparing a seminar to address the concerns of pregnant adolescent. What information is most important for the nurse to include in this program? A. Nutritional requirements during pregnancy B. Pain management options for labor C. Comparison of infant feeding method D. Symptoms to report to the healthcare provider

A. Nutritional requirements during pregnancy

50. The nurse observes a new mother avoiding eye contact with her newborn. Which action should the nurse take? A. Observe the mother for other attachment behaviors. B. Ask the mother why she won't look at the infant. C. Examine the newborn's eyes for the ability to focus. D. Recognize this as a common reaction in new mothers.

A. Observe the mother for other attachment behaviors.

A newborn infant, diagnosed with developmental dysplasia of the hip (DDH), is being prepared for discharge. Which nursing intervention should be included in this infant's discharge teaching plan? A. Observe the parents applying a Pavlik harness. B. Provide a referral for an orthopedic surgeon. C. Schedule a physical therapy follow-up home visit. D. Teach the parents to check for hip joint mobility.

A. Observe the parents applying a Pavlik harness.

A toddler with a history of acyanotic defect is admitted to the pediatric intensive care. Respiration rate 60 beats / min and heart 150 beats/ min.What action should the nurse take first? A. Obtain a pulse ox reading B. Assess childs blood pressure C. Perform a neurological assessment D. Initiate peripheral intravenous access

A. Obtain a pulse ox reading

34. A client with asthma who is 8-hours post-delivery is experiencing postpartum hemorrhage. Which prescription should the nurse administer? A. Oxytocin (Pitocin). B. Ibuprofen (Motrin). C. Fentanyl (Sublimaze). D. Hemabate (Carboprost).

A. Oxytocin (Pitocin).

The client comes to the hospital assuming she is in labor. Which assessment finding(s) by the nurse would indicate that the client is in true labor? (Select all that apply.) A. Pain in the lower back that radiates to abdomen B. Contractions decreased in frequency with ambulation C. Progressive cervical dilation and effacement D. Discomfort localized in the abdomen E. Regular and rhythmic painful contractions

A. Pain in the lower back that radiates to abdomen C. Progressive cervical dilation and effacement E. Regular and rhythmic painful contractions

The nurses assessment on a preterm infant reveals decreased muscle tone , sign of respiratory distress , irritability , mottled cool skin.Which intervention should the nurse implement first ? A. Position a radiant warmer on the crib B. Asses infant blood glucose level C. Place infant in side lying position D. Nipple feed 1 ounce of 5%glucose in water

A. Position a radiant warmer on the crib

A primigravida is 36 weeks gestation, is Rh negative is experienced abdominal trauma in a motor vehicle collision. Which assessment finding is most important for the nurse to report to the healthcare provider? A. Positive fetal hemoglobin test B. Fetal heart rate is 162 beats / min C. Trace of protein in urine D. Mild contractions every 10 mins

A. Positive fetal hemoglobin test

In preparing a gravid client for a triple screen analysis, which action should the nurse take? A. Prepare to draw blood for analysis. B. Encourage the client to drink 8 oz of water. C. Assist the client to left lateral tilt position. D. Apply an external fetal monitor to the abdomen.

A. Prepare to draw blood for analysis.

The nurse is planning care for a client at 30-weeks gestation who is experiencing preterm labor. A. Terbutaline (Brethine) 0.25 mg subcutaneously q15 minutes x 3 B. Ampicillin 1 gram IV push q8h. C. Betamethasone (Celestone) 12 mg deep IM D. Butorphanol (Stadol) 1 mg IV push q2h PRN pain

A. Terbutaline (Brethine) 0.25 mg subcutaneously q15 minutes x 3

46. A primigravida at 12-weeks gestation who just move to United States indicates she has not received any immunizations. Which immunization(s) should the nurse administer at this time? *(Select all that apply.)* A. Tetanus. B. Rubella. C. Hepatitis B. D. Chickenpox. E. Diphtheria.

A. Tetanus. C. Hepatitis B. E. Diphtheria.

Client teaching is an important part of the perinatal nurse's role. Which factor has the greatest influence on successful teaching of the pregnant client? A. The client's investment in what is being taught B. The couple's highest levels of education C. The order in which the information is presented D. The extent to which the pregnancy was planned

A. The client's investment in what is being taught

Client teaching is an important part of the maternity nurse's role. Which factor has the greatest influence on successful teaching of the gravid client? A. The client's readiness to learn B. The client's educational background C. The order in which the information is presented D. The extent to which the pregnancy is planned

A. The client's readiness to learn

39. The nurse is caring for a client whose labor is being augmented with oxytocin (Pitocin). Which finding indicates that the nurse should discontinue the oxytocin infusion? A. The fetal heart rate is 180 bpm without variability. B. Amniotic membranes rupture. C. The client needs to void. D. Uterine contractions occur every 8 to 10 minutes.

A. The fetal heart rate is 180 bpm without variability.

Following a minor vehicle collision , a client 36 weeks gestation is brought to the emergency center. She is lying supine on a backboard , is awake , denies any complaints. Her blood pressure is 80/50 mm Hg and heart rate is 130 beats per min. What action should the nurse implement first? A. Turn the board sideways to displace the uterus lateral B. Palpate the abdomen for contractions C. Infuse 1,000 normal saline using a large bore IV D. Obtain blood sample for a complete blood count

A. Turn the board sideways to displace the uterus lateral

The nurse is counseling a couple who has sought information about conceiving. The couple asks the nurse to explain when ovulation usually occurs. Which statement by the nurse is correct? A. Two weeks before menstruation B. Immediately after menstruation C. Immediately before menstruation D. Three weeks before menstruation

A. Two weeks before menstruation

The nurse is counseling a couple who has sought information about conceiving. The couple asks the nurse to explain when ovulation usually occurs. Which statement by the nurse is correct? A. Two weeks before menstruation B. Immediately after menstruation C. Immediately before menstruation D. Three weeks before menstruation

A. Two weeks before menstruation

The nurse is assessing a client at 29 weeks gestation. Which assessment measure would provide the most accurate determination of fetal position? A. Ultrasound B. Vaginal examination C. Leopolds maneuver D. Doppler

A. Ultrasound

1. At 10 weeks gestation, a high-risk multiparous client with a family history of Down syndrome is admitted for observation following a chorionic villavilla sampling (CVS) procedure. What assessment finding requires immediate intervention? A. Uterine cramping. B. Intermittent nausea. C. Systolic blood pressure < 100 mmHg. D. Abdominal tenderness.

A. Uterine cramping.

A woman who is trying to get pregnant tells the nurse that she was very disappointed several months ago when she was informed that her positive pregnancy test was a false positive. Which method of testing provides the greatest degree of accuracy? A. Visualization of implantation by vaginal ultrasound. B. Presence of amenorrhea for 2 months. C. Maternal blood serum tests positive for alpha-fetoprotein. D. Complaints of feeling tired all of the time.

A. Visualization of implantation by vaginal ultrasound.

A 6-week-old infant diagnosed with pyloric stenosis has recently developed projectile vomiting. Which assessment finding indicates to the nurse that the infant is becoming dehydrated? A. Weak cry without any tears B. Bulging fontanel C. Visible peristaltic wave. D. Palpable mass in the right upper quadrant

A. Weak cry without any tears

Twenty four hours after admission to the newborn nursery, a full-term male infant develops localized edema on the right side of his head. The nurse knows that, in the newborn, an accumulation of blood between the periosteum and skull which does not cross the suture line in a newborn variation known as A. a cephalhematoma, caused by forceps trauma and may last up to 8 weeks B. a subarachnoid hematoma, which requires immediate drainage to prevent further complications C. molding, caused by pressure during labor and will disappear within 2 to 3 days D. a subdural hematoma which can result in lifelong damage

A. a cephalhematoma, caused by forceps trauma and may last up to 8 weeks

The nurse is performing a gestational age assessment on a full-term newborn during the first hour of transition using the Ballard (Dubowitz) scale. Based on this assessment, the nurse determines that the neonate has a maturity rating of 40 weeks. What findings should the nurse identify to determine if the neonate is small for gestational age (SGA)? Select all that apply A. admission weight of 4 pounds, 15 ounces (2244 grams) B. head to heel length of 17 inches (42.5 cm) C. Frontal occipital circumference of 12.5 in (31.25 cm) D. Skin smooth with visible veins and abundant vernix E. Anterior plantar crease and smooth heel surfaces F. Full flexion of all extremities in resting supine position

A. admission weight of 4 pounds, 15 ounces (2244 grams) B. head to heel length of 17 inches (42.5 cm) C. Frontal occipital circumference of 12.5 in (31.25 cm). The normal full-term appropriate for gestational age (AGA) newborn should fall between the measurement ranges of weight 6-9 pounds, length 19-21 inches, FOC 13-14 inches. This neonate's parameters plot below the 10% percentile, which indicate that the infant is SGA.

A woman who gave birth 48 hours ago is bottle-feeding her infant. During assessment, the nurse determines that both breasts are swollen, warm, and tender upon palpation. What action should the nurse take? A. apply cold compresses to both breasts for comfort. B. instruct the client to run warm water on her breasts C. wear a loose-fitting bra to prevent nipple irritation D. express small amounts of milk to relieve pressure

A. apply cold compresses to both breasts for comfort.

A vaginally delivered infant of an HIV positive mother is admitted to the newborn nursery. What intervention should the nurse perform first? A. bathe the infant with an antimicrobial soap B. measure the head and chest circumference C. obtain the infant's footprints D. administer vitamin K (AquaMEPHYTON)

A. bathe the infant with an antimicrobial soap. To reduce direct contact with the Human immuno-virus in blood and body fluids on the newborn's skin, a bath with an antimicrobial soap should be administered first.

The nurse is teaching a woman how to use her basal body temperature (BBT) pattern as a tool to assist her in conceiving a child. Which temperature pattern indicates the occurrence of ovulation, and therefore, the best time for intercourse to ensure conception? A. between the time the temperature falls and rises B. between 36 and 48 hours after the temperature rises C. when the temperature falls and remains low for 36 hours D. within 72 hours before the temperature falls

A. between the time the temperature falls and rises

A multigravida client at 41 weeks gestation presents in the labor and delivery unit after a non-stress test indicated that the fetus is experiencing some difficulties in utero. Which diagnostic test should the nurse prepare the client for additional information about fetal status? A. biophysical profile (BPP) B. ultrasound for fetal anomalies C. maternal serum alpha-fetoprotein (AF) screening D. percutaneous umbilical blood sampling (PUBS)

A. biophysical profile (BPP). This test provides data regarding fetal risk surveillance by examining 5 areas: fetal breathing movements, fetal movements, amniotic fluid volume, and fetal tone and heart rate. The client's gestation has progressed past the estimated date of confinement, so the major concern is fetal well-being related to the aging placenta.

What is the priority nursing assessment immediately following the birth of an infant with esophageal atresia and a tracheoesophageal (the) fistula ? A. body temperature B. level of pain C. time of first void D. number of vessels in the cord

A. body temperature

A full term infant is admitted to the newborn nursery. After careful assessment, the nurse suspects that the infant may have an esophageal atresia. Which symptoms are this newborn likely to exhibit? A. choking, coughing, and cyanosis B. projectile vomiting and cyanosis C. apneic spells and grunting D. scaphoid abdomen and anorexia

A. choking, coughing and cyanosis

A client at 28 weeks gestation calls the antepartal clinical and states that she is experiencing a small amount of vaginal bleeding which she describes as bright red. She further states that she is not experiencing any uterine contractions or abdominal pain. What instruction should the nurse provide? A. come to the clinic today for an ultrasound B. go immediately to the emergency room C. lie on your left side for about one hour and see if the bleeding stops D. bring a urine specimen to the lab tomorrow to determine if you have a urinary tract infection

A. come to the clinic today for an ultrasound. Third trimester painless bleeding is characteristic of a placenta previa. Bright red bleeding may be intermittent, occur in gushes, or be continuous. Rarely is the first incidence life-threatening, nor cause for hypovolemic shock. Diagnosis is confirmed by transabdominal ultrasound.

A multigravida client arrives at the labor and delivery unit and tells the nurse that her "bag of water" has broken. The nurse identifies the presence of meconium fluid on the perineum and determines the fetal heart rate is between 140-150 bpm. What action should the nurse implement next? A. complete a sterile vaginal exam B. take maternal temperature every 2 hours C. prepare for an immediate cesarean birth D. obtain sterile suction equipment

A. complete a sterile vaginal exam. This is done to determine the presence of a prolapsed umbilical cord.

A healthcare provider informs the charge nurse of a labor and delivery unit that a client is coming to the unit with suspected abruptio placentae. What findings should the charge nurse expect the client to demonstrate? (Select all that apply) A. dark, red vaginal bleeding B. lower back pain C. premature rupture of the membranes D. increased uterine irritability E. bilateral pitting edema F. a rigid abdomen

A. dark, red vaginal bleeding D. increased uterine irritability

A 35-year old primigravida client with severe preeclampsia is receiving magnesium sulfate via continuous IV infusion. Which assessment data would indicate to the nurse that the client is experiencing magnesium sulfate toxicity? A. deep tendon reflexes 2+ B. blood pressure 140/90 C. respiratory rate 18/min D. urine output 90 mL/4 hours

A. deep tendon reflexes 2+

A woman with type 2 diabetes mellitus becomes pregnant, and her oral hypoglycemic agents are discontinued. Which intervention is most important for the nurse to implement? A. Describe diet changes that can improve the management of her diabetes B. inform the client that oral hypoglycemic agents are teratogenic during pregnancy C. Demonstrate self-administration of insulin D. evaluate the client's ability to do glucose monitoring

A. describe diet changes that can improve the management of her diabetes

The nurse is caring for a woman with a previously diagnosed heart disease who is in the second stage of labor. Which assessment findings are of the greatest concern? A. edema, basilar rates, and an irregular pulse B. increased urinary output and tachycardia C. shortness of breath, bradycardia, and hypertension D. regular heart rate and hypertension

A. edema, basilar rates, and an irregular pulse

The healthcare provider prescribes terbutalne (Brethine) for a client in preterm labor. Before initiating this prescription, it is most important for the nurse to assess the client for which condition? A. gestational diabetes B. elevated blood pressure C. urinary tract infection D. swelling in lower extremities

A. gestational diabetes

The nurse attempts to help an unmarried teenager deal with her feelings following a spontaneous abortion at 8 weeks gestation. What type of emotional response should the nurse anticipate? A. grief related to her perceptions about the loss of this child B. Relief of ambivalent feelings experienced with this pregnancy C. Shock because she may not have realized that she was pregnant D. guilt because she had not followed her healthcare provider's instructions

A. grief related to her perceptions about the loss of this child

Which action should the nurse implement when preparing to measure the fundal height of a pregnant client? A. have the client empty her bladder B. request the client lie on her left side C. Perform Leopold's maneuvers first D. Give the client some cold juice to drink

A. have the client empty her bladder

Immediately after birth, a newborn infant is suctioned, dried, and placed under a radiant warmer. The infant has spontaneous respirations and the nurse assesses an apical heart rate of 80 bpm and respirations of 20 breaths/min. What action should the nurse perform next? A. initiate positive pressure ventilation B. intervene after the one minute Apgar is assessed C. initiate CPR on the infant D. assess the infant's blood glucose level

A. initiate positive pressure ventilation because the infant's vital signs are not within the normal range and oxygen deprivation leads to cardiac depression in infants. The normal newborn pulse is 100-160 bpm and respirations are 40-60 breaths/minute.

Which nurse intervention would be most helpful in relieving postpartum uterine contractions or "afterpains?" A. lying prone with a pillow on the abdomen B. using a breast pump C. massaging the abdomen D. giving oxytocic medications

A. lying prone with a pillow on the abdomen

When explaining "postpartum blues" to a client who is 1 day postpartum, which symptoms should the nurse include in the teaching plan? (Select all that apply) A. mood swings B. panic attacks C. tearfulness D. decreased need for sleep E. disinterest in the infant

A. mood swings C. tearfulness

A client receiving epidural anesthesia begins to experience nausea and becomes pale and clammy. What intervention should the nurse implement first? A. raise the foot of the bed B. assess for vaginal bleeding C. evaluate the fetal heart rate D. take the client's blood pressure

A. raise the foot of the bed. These symptoms are suggestive of hypotension which is a side effect of epidural anesthesia. Raising the foot of the bed (Trendelenburg position) will increase venous return and provide blood to the vital areas.

The nurse is providing discharge for a client who is 24 hours postpartum. The nurse explains to the client that her vaginal discharge will change from red to pink to white. The client asks, "What if I start having red bleeding after it changes?" What should the nurse instruct the client to do? A. reduce activity level and notify the healthcare provider B. go to bed and assume a knee-chest position C. massage the uterus and go to the emergency room d. do not worry as this is a normal occurance

A. reduce activity level and notify the healthcare provider

A 4 week old premature infant has been receiving epoetin alfa (Epogen) for the last 3 weeks. Which assessment finding indicates to the nurse that the drug is effective? A. slowly increasing urinary output over the last week B. respiratory rate changes from the 40s to the 60s. C. changes in apical heart rate from the 180s to the 140s D. change in indirect bilirubin from 12 mg/dl to 8 mg/dl

A. slowly increasing urinary output over the last week

A 42 week gestational client is receiving an intraenous infusion of oxytocin (pitocin) to augment early labor. The nurse should discontinue the oxytocin infusion for which pattern of contractions? A. transition labor with contractions every 2 minutes, lasting 90 seconds each. B. early labor with contractions every 5 minutes, lasting 40 seconds each C. Active labor with contractions every 31 minutes, lasting 60 seconds each D. Active labor with contractions every 2-3 minutes, lasting 70-80 seconds each

A. transition labor with contractions every 2 minutes, lasting 90 seconds each. When oxytocin causes uterine hyperstimulation as evidence by inadequate resting time between contractions, the oxytocin infusion should be discontinued because placental perfusion is impeded.

The nurse is counseling a couple who has sought information about conceiving. For teaching purposes, the nurse should know that ovulation usually occurs A. two weeks before menstruation B. immediately after menstruation C. immediately before menstruation D. three weeks before menstruation

A. two weeks before menstruation

The nurse is assessing a 3 day old infant with a cephaloheatoma in the newborn nursery. Which assessment finding should the nurse report to the healthcare provider? A. Yellowish tinge to the skin B. Babinski reflex present bilaterally C. pink papular rash on the face D. Moro reflex noted after a loud noise

A. yellowish tinge to the skin. Cephalohematomas are characterized by bleeding between the bone and its covering, the periosteum. Due to the breakdown of the red blood cells within a hematoma, the infant is at a greater risk for jaundice so it should be reported.

Which statement made by the client indicates that the mother understands the limitations of breastfeeding her newborn? A."Breastfeeding my infant consistently every 3 to 4 hours stops ovulation and my period." B."Breastfeeding my baby immediately after drinking alcohol is safer than waiting for the alcohol to clear my breast milk." C."I can start smoking cigarettes while breastfeeding because it will not affect my breast milk." D."When I take a warm shower after I breastfeed, it relieves the pain from being engorged between breastfeedings.

A."Breastfeeding my infant consistently every 3 to 4 hours stops ovulation and my period." Rationale: Continuous breastfeeding on a 3- to 4-hour schedule during the day will cause a release of prolactin, which will suppress ovulation and menses, but is not completely effective as a birth control method. Option B is incorrect because alcohol can immediately enter the breast milk. Nicotine is transferred to the infant in breast milk. Taking a warm shower will stimulate the production of milk, which will be more painful after breastfeedings.

The nurse is teaching a new mother about diet and breastfeeding. Which instruction is most important to include in the teaching plan? A.Avoid alcohol because it is excreted in breast milk. B.Eat a high-roughage diet to help prevent constipation. C.Increase caloric intake by approximately 500 cal/day. D.Increase fluid intake to at least 3 quarts each day.

A.Avoid alcohol because it is excreted in breast milk. Rationale: Alcohol should be avoided while breastfeeding because it is excreted in breast milk and may cause a variety of problems, including slower growth and cognitive impairment for the infant. Options B, C, and D should also be included in diet teaching for a breastfeeding mother; however, because these do not involve safety of the infant, they do not have the same degree of importance as option A.

A breastfeeding postpartum client is diagnosed with mastitis, and antibiotic therapy is prescribed. Which instruction should the nurse provide to this client? A.Breastfeed the infant, ensuring that both breasts are completely emptied. B.Feed expressed breast milk to avoid the pain of the infant latching onto the infected breast. C.Breastfeed on the unaffected breast only until the mastitis subsides. D.Dilute expressed breast milk with sterile water to reduce the antibiotic effect on the infant.

A.Breastfeed the infant, ensuring that both breasts are completely emptied. Rationale:Mastitis, caused by plugged milk ducts, is related to breast engorgement, and breastfeeding during mastitis facilitates the complete emptying of engorged breasts, eliminating the pressure on the inflamed breast tissue. Option B is less painful but does not facilitate complete emptying of the breast tissue. Option C will not relieve the engorgement on the affected side. Option D will not decrease antibiotic effects on the infant.

Twenty-four hours after admission to the newborn nursery, a full-term male infant develops localized swelling on the right side of his head. In a newborn, what is the most likely cause of this accumulation of blood between the periosteum and skull that does not cross the suture line? A.Cephalhematoma, which is caused by forceps trauma B.Subarachnoid hematoma, which requires immediate drainage C.Molding, which is caused by pressure during labor D.Subdural hematoma, which can result in lifelong damage

A.Cephalhematoma, which is caused by forceps trauma Rationale: Cephalhematoma, a slight abnormal variation of the newborn, usually arises within the first 24 hours after delivery. Trauma from delivery causes capillary bleeding between the periosteum and skull. Option C is a cranial distortion lasting 5 to 7 days, caused by pressure on the cranium during vaginal delivery, and is a common variation of the newborn. Options B and D both involve intracranial bleeding and could not be detected by physical assessment alone.

A client at 28 weeks of gestation calls the antepartal clinic and states that she has just experienced a small amount of vaginal bleeding, which she describes as bright red. The bleeding has subsided. She further states that she is not experiencing any uterine contractions or abdominal pain. What instruction should the nurse provide? A.Come to the clinic today for an ultrasound. B.Go immediately to the emergency department. C.Lie on your left side for about 1 hour and see if the bleeding stops. D.Take a urine specimen to the laboratory to see if you have a urinary tract infection (UTI).

A.Come to the clinic today for an ultrasound. Rationale:Third-trimester painless bleeding is characteristic of a placenta previa. Bright red bleeding may be intermittent, occur in gushes, or be continuous. Rarely is the first incident life threatening or cause for hypovolemic shock. Diagnosis is confirmed by transabdominal ultrasound. Bleeding that has a sudden onset and is accompanied by intense uterine pain indicates abruptio placenta, which is life threatening to the mother and fetus. If those symptoms were described, option B would be appropriate. Option C does not address the cause of the symptoms. The client is not describing symptoms of a UTI.

Which findings are most critical for the nurse to report to the primary health care provider when caring for the client during the last trimester of her pregnancy? (Select all that apply.) A.Increased heartburn that is not relieved with doses of antacids B.Increase of the fetal heart rate from 126 to 156 beats/min from the last visit C.Shoes and rings that are too tight because of peripheral edema in extremities D.Decrease in ability for the client to sleep for more than 2 hours at a time E.Chronic headache that has been lingering for a week behind the client's eyes

A.Increased heartburn that is not relieved with doses of antacids E.Chronic headache that has been lingering for a week behind the client's eyes Rationale: Options A and E are possible signs of preeclampsia or eclampsia but can also be normal signs of pregnancy. These signs should be reported to the health care provider for further evaluation for the safety of the client and the fetus. Options B, C, and D are all normal signs during the last trimester of pregnancy.

On admission to the prenatal clinic, a client tells the nurse that her last menstrual period began on February 15 and that previously her periods were regular (28-day cycle). Her pregnancy test is positive. What is this client's expected date of birth (EDB)? A.November 22 B.November 8 C.December 22 D.October 22

A.November 22 Rationale: Option A correctly applies the Nägele rule for estimating the due date by counting back 3 months from the first day of the last menstrual period (January, December, November) and adding 7 days (15 + 7 = 22). Options B, C, and D are not calculated correctly.

A newborn infant, diagnosed with developmental dysplasia of the hip (DDH), is being prepared for discharge. Which nursing intervention should be included in this infant's discharge teaching plan? A.Observe the parents applying a Pavlik harness. B.Provide a referral for an orthopedic surgeon. C.Schedule a physical therapy follow-up home visit. D.Teach the parents to check for hip joint mobility.

A.Observe the parents applying a Pavlik harness. Rationale: It is important that the hips of infants with hip dysplasia are maintained in an abducted position, which can be accomplished by using the Pavlik harness; this keeps the hips and knees flexed, the hips abducted, and the femoral head in the acetabulum. Early treatment often negates the need for surgery, and option B is not indicated until approximately 6 months of age. Option C is not indicated for hip dysplasia. It is best for the pediatrician to monitor hip joint mobility, and teaching the parents to perform this technique is likely to increase their anxiety.

The client comes to the hospital assuming she is in labor. Which assessment findings by the nurse would indicate that the client is in true labor? (Select all that apply.) A.Pain in the lower back that radiates to abdomen B.Contractions decreased in frequency with ambulation C.Progressive cervical dilation and effacement D.Discomfort localized in the abdomen E.Regular and rhythmic painful contractions

A.Pain in the lower back that radiates to abdomen C.Progressive cervical dilation and effacement E.Regular and rhythmic painful contractions Rationale: These are all signs of true labor. Options B and D are signs of false labor.

The nurse is assisting with the insertion of a pulmonary artery catheter (PAC) for a client at 32 weeks who has severe preeclampsia with pulmonary edema. As the PAC enters the right ventricle, what is the priority nursing assessment? A) Observe for maternal BP changes B) Monitor for premature ventricular contractions C) Assess fetal response to the procedure D) Note the complaint of sudden chest pain

B

The nurse is providing discharge teaching for a gravid client who is being released from the hospital after placement of a cerclage. Which instruction is the most important for the client to understand? A) Plan for a possible C-section B) Report uterine cramping or low backache C) Arrange for home uterine monitoring D) Make arrangements for care at home

B

The nurse is teaching a new mother about diet and breastfeeding. Which instruction is most important to include in the teaching plan? A)Increase caloric intake by approximately 500kcal/day B)Avoid alcohol because it is excreted in breast milk C) avoid spicy foods to prevent infant colic D) Double prenatal milk intake to improve Vit D transfer to infant

B

The nurse tells a client in her first trimester that she should increase her daily intake of calcium. The client doesn't like milk. What dietary adjustment should the nurse recommend? A) Choose more fresh citrus and other fruits daily B) Eat more green leafy veggies C) Increase organ meats in the diet D) Add molasses and whole grain breads to the diet

B

What assessment finding should the nurse report to the healthcare provider that is consistent with concealed hemorrhage in an abruptio placenta? A) decrease in abdominal pain B) Hard board like abdomen C) Decrease in fundal height D) Maternal bradycardia

B

What nursing action should be included in the plan of care for a newborn experiencing symptoms of drug withdrawal? A) Feed every 4-6 hours to allow extra rest B) Swaddle the infant snugly and hold tightly C) play soft music and talk to soothe the infant D) Administer chloral hydrate for sedation

B

When assessing a newborn infant's HR, what technique is most important for the nurse to use? A) palpate the umbilical cord B) Count the HR for at least one full minute C) Quiet the infant before counting HR D) Listen at the apex of the heart

B

Which client finding should the nurse document as a positive sign of pregnancy? A) Last menstrual cycle occurred 2 months ago B) FHTs heard with a doppler C) Presence of braxton hicks contractions D) A urine sample with a positive pregnancy test

B

Which statement by a client who is pregnant indicates to the nurse an understanding of the role of protein during pregnancy? A) Anemia is averted by consuming enough protein B) Protein helps the fetus grow while I am pregnant C) My baby will develop strong teeth after he is born D) Gestational diabetes is prevented by eating protein

B

While inspecting the newborn's head the nurse identifies a swelling of the scalp that does not cross the suture line. Which finding does the nurse document? A) Molding B) Cephalohematoma C) Caput succedaneum D) Bulging fontanel

B

A female client with insulin-dependent diabetes arrives at the clinic seeking a plan to get pregnant in approximately 6 months. She tells the nurse that she wants to have an uncomplicated pregnancy and a healthy baby. What information should the nurse share with the client? A. "Your current dose of insulin should be maintained throughout your pregnancy." B. "Maintain blood sugar levels in a constant range within normal limits during pregnancy." C. "The course and outcome of your pregnancy is not an achievable goal with diabetes." D. "Expect an increase in insulin dosages by 5 units/wk during the first trimester."

B. "Maintain blood sugar levels in a constant range within normal limits during pregnancy."

17. Which statement by a client who is pregnant indicates to the nurse an understanding of the role of protein during pregnancy? A. "Gestational diabetes is prevented by eating protein." B. "Protein helps the fetus grow while I am pregnant." C. "My baby will develop strong teeth after he is born." D. "Anemia is averted by consuming enough protein."

B. "Protein helps the fetus grow while I am pregnant."

A new mother asks the nurse about an area of swelling on her baby's head near the posterior fontanel that lies across the suture line. How should the nurse respond? A. "That is called caput succedaneum. It will have to be drained." B. "That is called caput succedaneum. It will absorb and cause no problems." C. "That is called a cephalhematoma. It will cause no problems." D. "That is called a cephalhematoma. It can cause jaundice as it is absorbed."

B. "That is called caput succedaneum. It will absorb and cause no problems."

A new mother asks the nurse, "How do I know that my daughter is getting enough breast milk?" Which explanation is appropriate? A. "Weigh the baby daily, and if she is gaining weight, she is getting enough to eat." B. "Your milk is sufficient if the baby is voiding pale, straw-colored urine six to ten times a day." C. "Offer the baby extra bottled milk after her feeding and see if she still seems hungry." D "If you're concerned, you might consider bottle feeding so that you can monitor intake."

B. "Your milk is sufficient if the baby is voiding pale, straw-colored urine six to ten times a day."

A new mother asks the nurse, "How do I know that my daughter is getting enough breast milk?" Which explanation will the nurse provide? A. "weigh the baby daily, and if she is gaining weight, she is eating enough." B. "your milk is sufficient if the baby is voiding pale straw-colored urine 6-10 times/day." C. "Offer the baby extra bottle milk after her feeding and see if she is still hungry." D. "If you're concerned, you might consider bottle feeding so that you can monitor her intake."

B. "your milk is sufficient if the baby is voiding pale straw-colored urine 6-10 times/day."

The nurse is planning discharge teaching for 4 mothers. Which postpartum client is at highest risk for psychological difficulties during the postpartum period? A. A multiparous client who lives with her husband and his family members. B. A primiparous woman who has recently immigrated to the U.S. with her spouse. C. A multiparous female with a large family living in the community. D. A primiparous adolescent living at home with her parents and significant other.

B. A primiparous woman who has recently immigrated to the U.S. with her spouse.

The nurse caring for a laboring client encourages her to void at least q2h, and records each time the client empties her bladder. What is the primary reason for implementing this nursing intervention? A. emptying the bladder during delivery is difficult because of the position of the presenting fetal part. B. An over-distending bladder could be traumatized during labor, as well as prolong the progress of labor C. urine specimens for glucose and protein must be obtained at certain intervals throughout labor. D. frequent voiding minimizes the need for catheterization which increases the chance of bladder infection

B. An over-distending bladder could be traumatized during labor, as well as prolong the progress of labor

66. A woman who is bottle-feeding her newborn infant calls the clinic 72 hours after delivery and tells the nurse that both of her breasts are swollen, warm, and tender. What instructions should the nurse give? A. Wear a loose-fitting bra. B. Apply ice to the breasts. C. Run warm water on the breasts during a shower. D. Express small amounts of milk from the breasts.

B. Apply ice to the breasts.

4. An infant in respiratory distress is placed on pulse oximetry. The oxygen saturation indicates 85%. What is the priority nursing intervention? A. Evaluate the blood pH. B. Begin humidified oxygen via hood. C. Place the infant under a radiant warmer. D. Stimulate infant crying.

B. Begin humidified oxygen via hood.

A client who is 3 days postpartum and breastfeeding asks the nurse how to reduce breast engorgement. Which instruction should the nurse provide? A. Avoid using the breast pump. B. Breastfeed the infant every 2 hours. C. Reduce fluid intake for 24 hours. D. Skip feedings to let the sore breasts rest.

B. Breastfeed the infant every 2 hours.

A postpartal client complains that she has the urge to urinate every hour but is only able to void a small amount. What interventions provides the nurse with the most useful information? A. Initiate a perineal pad count B. Catheterize for residual urine after next voiding C. Assess for perineal hematoma D. Determine the clients usual voiding pattern

B. Catheterize for residual urine after next voiding

A pregnant woman in the first trimester of pregnancy has a hemoglobin of 8.6 mg/dl and a hematocrit of 25.1%. What food should the nurse encourage this client to include in her diet? A. Carrots B. Chicken C. Yogurt D. Cheese

B. Chicken

A client at 35-weeks gestation complains of a "pain whenever the baby moves." On assessment, the nurse notes the client's temperature to be 101.2F, with severe abdominal or uterine tenderness on palpation. The nurse knows that these findings are indicative of what condition? A. Round ligament strain B. Chorioamnionitis C. Abruptio placenta D. Viral infection.

B. Chorioamnionitis

The nurse is conducting postpartum teaching with a mother who is breastfeeding here infant. When discussing birth control, which method should the nurse recommend to this client as beneficial for her to use in preventing an unwanted pregnancy? A. Breastfeed exclusively at least every 3-4 hours B. Condoms and contraceptive foam or gel C. Rhythm method (natural family planning) D. Combined estrogen progesterone oral contraceptives.

B. Condoms and contraceptive foam or gel

A client whose labor is being augmented with an oxytocin (Pitocin) infusion requests an epidural for pain control. Findings of the last vaginal exam, performed 1 hour ago, were 3 cm cervical dilatation, 60% effacement, and a -2 station. What action should the nurse implement first? A. Decrease the oxytocin infusion rate B. Determine current cervical dilation C. Request placement of the epidural D. Give a bolus of intravenous fluids

B. Determine current cervical dilation

73. A multiparous client is experiencing bleeding 2 hours after a vaginal delivery. What action should the nurse implement next? A. Inform the healthcare provider of the bleeding. B. Determine the firmness of the fundus. C. Give oxytocin (Pitocin) intravenously. D. Assess the vital signs for indicators of shock.

B. Determine the firmness of the fundus.

The nurse is caring for a client whose fetus died in utero at 32 weeks gestation. After the fetus is delivered vaginally, the nurse implements routine fetal demise protocol and identification procedures. Which action is important for the nurse to take? A. Explain reasons consent for an infant autopsy is needed. B. Encourage the mother to hold and spend time with her baby. C. Determine if the mother desires a visit from her clergy. D. Create a memory box of baby's footprints and photographs.

B. Encourage the mother to hold and spend time with her baby.

A client at 18-weeks gestation was informed this morning that she has an elevated alpha-fetoprotein(AFP) level. After the healthcare provider leaves the room, the client asks what she should do next. What information should the nurse provide? A. Reassure the client that the AFP results are likely to be a false reading. B. Explain that a sonogram should be scheduled for definitive results. C. Discuss options for intrauterine surgical correction of congenital defects. D. Inform her that a repeat alpha-fetoprotein(AFP) should be elevated

B. Explain that a sonogram should be scheduled for definitive results.

6. The nurse prepares to administer an injection of vitamin K to a newborn infant. The mother tells the nurse, "Wait! I don't want my baby to have a shot." Which response would be best for the nurse to make? A. Inform the mother that the injection was prescribed by the healthcare provider. B. Explore the mother's concern about the infant receiving an injection of vitamin K. C. Remind the mother that all babies receive the shot and it is relatively painless. D. Explain that vitamin K is required by state law and compliance is mandatory.

B. Explore the mother's concern about the infant receiving an injection of vitamin K.

A primipara at 20-weeks gestation is scheduled for an ultrasound. In preparing the client for the procedure, the nurse should explain that the primary reason for conducting this diagnostic study is to obtain which information? A. Sex and size of the infant. B. Fetal growth and gestational age. C. Chromosomal abnormalities. D. Lecithin-sphingomyelin ration.

B. Fetal growth and gestational age.

On the first postpartum day, the nurse examines the breasts of the new mother. Which condition is the nurse most likely to. A. Slightly firm with immediate let down response B. Filing and secreting colostrum C. Soft, with no change from before delivery D. Firm, larger very tender to touch

B. Filling and secreting colostrum

At 20-weeks gestation, a client who has gained 20 pounds during this pregnancy tells the nurse that she is feeling fetal movement. Fundal height measurement is 20 cm, and the clients only complaint is that her breasts are leaking clear fluid. Which assessment finding warrants further evaluation? A. Presence of fetal movements. B. Gestational weight gain C. Fundal height measurement D. Leakage from breasts

B. Gestational weight gain

A primigravida at 40 weeks gestation is contraction q2 minutes her cervix is 9cm dilated and 100% effaced. The fetus heart rate is 120 beats per minute. The client is screaming and her husband is alarmed. What intervention should the nurse do?A. Notify rapid response B. Have delivery table set up C. Ask husband to step out D. Administer a PRN narcotic

B. Have delivery table set up

Which maternal behavior is the nurse most likely to see when a new mother receives her infant for the first time? A. she eagerly reaches for the infant, undresses the infant, and examines the infant completely. B. Her arms and hands receive the infant and she then traces the infant's profile with her fingertips C. Her arms and hands receive the infant and she then cuddles the infant to her own body. D. She eagerly reaches for the infant and then holds the infant close to her own body

B. Her arms and hands receive the infant and she then traces the infant's profile with her fingertips

Which maternal behavior is the nurse most likely to see when a new mother receives her infant for the first time? A. She eagerly reaches for the infant, undresses the infant, and examines the infant completely. B. Her arms and hands receive the infant and she then traces the infant's profile with her fingertips. C. Her arms and hands receive the infant and she then cuddles the infant to her own body. D. She eagerly reaches for the infant and then holds the infant close to her own body.

B. Her arms and hands receive the infant and she then traces the infant's profile with her fingertips.

Which maternal behavior is the nurse most likely to see when a new mother receives her infant for the first time? A. She eagerly reaches for the infant, undresses the infant, and examines the infant completely. B. Her arms and hands receive the infant and she then traces the infant's profile with her fingertips. C. Her arms and hands receive the infant and she then cuddles the infant to her own body. D. She eagerly reaches for the infant and then holds the infant close to her own body.

B. Her arms and hands receive the infant and she then traces the infant's profile with her fingertips.

A primigravida, when returning for the results of her multiple marker screening (triple screen), asks the nurse how problems with her baby can be detected by the test. What information will the nurse give to the client to describe best how the test is interpreted? A. If MSAFP (maternal serum alpha-fetoprotein) and estriol levels are high and the human chorionic gonadotropin (hCG) level is low, results are positive for a possible chromosomal defect. B. If MSAFP and estriol levels are low and the hCG level is high, results are positive for a possible chromosomal defect. C. If MSAFP and estriol levels are within normal limits, there is a guarantee that the baby is free of all structural anomalies. D. If MSAFP, estriol, and hCG are absent in the blood, the results are interpreted as normal findings.

B. If MSAFP and estriol levels are low and the hCG level is high, results are positive for a possible chromosomal defect.

47. A gravid client develops maternal hypotension following regional anesthesia. What intervention(s) should the nurse implement? *(Select all that apply.)* A. Perform a vaginal examination. B. Increase IV fluids. C. Administer oxygen. D. Monitor fetal status. E. Place the client in a lateral position. F. Assist client to a sitting position.

B. Increase IV fluids. C. Administer oxygen. D. Monitor fetal status. E. Place the client in a lateral position.

At 0600 while admitting a woman for a scheduled repeat cesarean section, the client tells the nurse that she drank a cup of coffee at 0400 because she wanted to avoid getting a headache. What action would the nurse take first? A. Ensure preoperative lab results are available. B. Inform the anesthesia care provider. C. Start prescribed IV with Lactated Ringer's. D. Contact the client's obstetrician.

B. Inform the anesthesia care provider.

A male infant with a 2-day history of fever and diarrhea is brought to a clinic by his mother who tells the nurse that the child refuses to drink anything. The nurse determines that the child has a weak cry with no tears. Which prescription is most important to implement? A. Provide a bottle of electrolyte solution B. Infuse normal saline intravenously C. Administer an antipyretic rectally D. Apply external cooling blanket

B. Infuse normal saline intravenously

The nurse is counseling a woman who wants to become pregnant. The woman tells the nurse that she has a 36-day menstrual cycle and the first day of her last menstrual period was January 8. The nurse correctly calculates that the woman's next fertile period will be A. January 14-15 B. January 22-23 C. January 30-31 D. February 6-7

B. January 30-31. This woman can expect her next period to begin 36 days from the first day of her last menstrual period - the cycle begins at the first day of the cycle and continues to the first day of the next cycle. Her next period would, therefore, began on February 13. Ovulation occurs 14 days before the first day of the menstrual period. Therefore, ovulation for this woman would occur January 31.

Following a precipitous labor, a postpartum client has a continuous trickling of bright red blood from her vagina. Her uterus is firm and her vital signs are within normal limits. The nurse determines that this sign may indicate which condition? A. Early postpartum hemorrhage. B. Laceration on the cervix C. Expected course in the fourth stage of labor. D. A full urinary bladder.

B. Laceration on the cervix

During a 26 week gestation prenatal exam, a client reports occasional dizziness. What intervention is best for the nurse to recommend the client? A. Elevate the head with two pillows while sleeping B. Lie on the left or right side when sleeping or resting C. Increase intake of foods that are high in iron D. Decrease the amount of carbohydrates in the diet

B. Lie on the left or right side when sleeping or resting

During a prenatal visit, the nurse discusses the effects of smoking on the fetus with a client. Which statement is most characteristic of an infant whose mother smoked during pregnancy compared with the infant of a nonsmoking mother? A. Lower Apgar score recorded at delivery B. Lower initial weight documented at birth C. Higher oxygen use to stimulate breathing D. Higher prevalence of congenital anomalies

B. Lower initial weight documented at birth

16. During a preconception counseling session for women trying to get pregnant in 3 to 6 months, what information should the nurse provide? A. Discontinue all forms of contraception. B. Make sure to include adequate folic acid in the diet. C. Continue to take any medications that are taken regularly. D. Lose weight so more weight is gained during pregnancy.

B. Make sure to include adequate folic acid in the diet.

A pregnant client tells the nurse that the first day of her last menstrual period was August 2, 2006. Based on Nagele's rule, what is the estimated date of delivery? A. April 25, 2007 B. May 9, 2007 C. May 29, 2007 D. June 2, 2007

B. May 9, 2007

52. An infant with hyperbilirubinaemia is receiving phototherapy. What intervention should the nurse implement? A. Maintain NPO status. B. Monitor temperature. C. Apply skin lotion as prescribed. D. Change T-shirt every 3 hours.

B. Monitor temperature.

A 36-week primigravida is admitted to labor and delivery with severe abdominal pain and bright red vaginal bleeding. Her abdomen is rigid and tender to touch. The fetal heart rate FHR) is 90 beats/minute, and the maternal heart rate is 120 beats/minute. What action should the nurse implement first? A. Alert the neonatal team and prepare for neonatal resuscitation B. Notify the healthcare provider from the client's bedside C. Obtain written consent for an emergency cesarean section D. Draw a blood sample for stat hemoglobin and hematocrit

B. Notify the healthcare provider from the client's bedside

A client who is 32 weeks gestation arrives at the clinic reporting nausea and vomiting for the past 24 hours. The nurse reviews the records and observes there has been a rapid weight gain over 6 weeks. Which action should the nurse implement next? A. Ask for a 24 hour diet recall. B. Obtain a blood pressure. C. Inspect for pedal edema. D. Listen to fetal heart rate.

B. Obtain a blood pressure.

45. Which finding for a client in labor at 41-weeks gestation requires additional assessment by the nurse? A. Score of eight on the biophysical profile. B. One fetal movement noted in an hour. C. Cervix dilated 2 cm and 50% effaced. D. Fetal heart rate of 116 bpm.

B. One fetal movement noted in an hour.

The nurse is assessing a patient who is 36 hours post delivery. Which finding should the nurse report to the healthcare provider? A. White blood cell count 19,000 B. Oral temperature of 100.6 C. Fundus deviated to the right D. Breast are firm when palpated

B. Oral temperature of 100.6

27. During an assessment of a multiparous client who delivered an 8-lb 7-oz infant 4 hours ago, the nurse notes the client's perineal pad is completely saturated within 15 minutes. What action should the nurse implement next? A. Encourage the client to avoid. B. Perform fundal massage. C. Notify the healthcare provider. D. Assess blood pressure.

B. Perform fundal massage.

The nurse is caring for a postpartum client who is exhibiting symptoms of a spinal headache 24 hours following delivery of a normal newborn. Prior to the anesthesiologists arrival on the unit, which action should the nurse perform? A. Cleanse the spinal injection site. B. Place procedure equipment at bedside. C. Apply an abdominal binder. D. Insert an indwelling Foley catheter.

B. Place procedure equipment at bedside.

The nurse calls a client who is 4 days postpartum to follow up about her transition with her newborn son at home. The woman tells the nurse, "I don't know what is wrong. I love my son, but I feel so let down. I seem to cry for no reason!" Which adjustment phase should the nurse determine the client is experiencing? A. Taking-in phase B. Postpartum blues C. Attachment difficulty D. Letting-go phase

B. Postpartum blues

58. What action should the nurse implement with the family when an infant is born with anencephaly? A. Ensure that measures to facilitate the attachment process are offered. B. Prepare the family to explore ways to cope with the imminent death of the infant. C. Provide emotional support to facilitate the consideration of fetal organ donation. D. Inform the family about multiple corrective surgical procedures that will be needed.

B. Prepare the family to explore ways to cope with the imminent death of the infant.

The nurse is caring for a client following an emergency cesarean delivery under general anesthesia. Which assessment finding occurring in the first 8 hours after deliver is most critical and requires immediate intervention? A. Mild nausea and anorexia. B. Respiratory rate of 12bpm. C. A positive Homan's sign. D. Uterine atony.

B. Respiratory rate of 12bpm

A woman who is 38 weeks gestation is receiving magnesium sulfate for severe preeclampsia. Which assessment finding warrants immediate intervention by the nurse? a. Dizziness when standing. b. Sinus tachycardia. c. Absent patellar reflexes. d. Lower back pain.

B. Sinus tachycardia

A nurse receives a shift change report for a newborn who is 12 hours post-vaginal delivery. In developing a plan of care, the nurse should give the highest priority to which finding? A. Cyanosis of the hands and feet B. Skin color that is slightly jaundiced C. Tiny white papules on the nose or chin D. Red patches on the cheeks and trunk

B. Skin color that is slightly jaundiced

A nurse receives a shift change report for a newborn who is 12 hours post-vaginal delivery. In developing a plan of care, the nurse should give the highest priority to which finding? A. Cyanosis of the hands and feet B. Skin color that is slightly jaundiced C. Tiny white papules on the nose or chin D. Red patches on the cheeks and trunk

B. Skin color that is slightly jaundiced

A nurse receives a shift change report for a newborn who is 12 hours post-vaginal delivery. In developing a plan of care, the nurse should give the highest priority to which finding? A.Cyanosis of the hands and feet B.Skin color that is slightly jaundiced C.Tiny white papules on the nose or chin D.Red patches on the cheeks and trunk

B. Skin color that is slightly jaundiced Rationale: Jaundice, a yellow skin coloration, is caused by elevated levels of bilirubin, which should be further evaluated in a newborn <24 hours old. Acrocyanosis (blue color of the hands and feet) is a common finding in newborns; it occurs because the capillary system is immature. Milia are small white papules present on the nose and chin that are caused by sebaceous gland blockage and disappear in a few weeks. Small red patches on the cheeks and trunk are called erythema toxicum neonatorum, a common finding in newborns.

21. The nurse is discussing the stages of labor with a group of women in the last month of pregnancy and provide examples of different positional techniques used during the second stage of labor. Which position should the nurse address that provides the best advantage of gravity during delivery? A. Walking. B. Squatting. C. Kneeling. D. Lithotomy.

B. Squatting.

11. A client at 25-weeks gestation tells the nurse that she dropped a cooking utensil last week and her baby jumped in response to the noise. What information should the nurse provide? A. Report the fetus's behavior to the healthcare provider. B. The fetus can respond to sound by 24-weeks gestation. C. This is a demonstration of the fetus's acoustical reflex. D. It is a coincidence the fetus responded at the same time.

B. The fetus can respond to sound by 24-weeks gestation.

9. A client at 8-weeks gestation ask the nurse about the risk for congenital heart defect (CHD) in her baby. Which response best explains when a CHD may occur? A. They usually occur in the first trimester pregnancy. B. The heart develops in the third to fifth weeks after conception. C. It depends on what the causative factors are for a CHD. D. We don't really know what or when CHDs occur.

B. The heart develops in the third to fifth weeks after conception.

Following a minor motor vehicle collision, a client at 36-weeks gestation is brought to the emergency center. She is lying supine on a backboard, is awake, and denies any complaints. Her blood pressure is 80/50 mmHg and heart rate is 130 bpm. Which action should the nurse implement first? A. Palpate the abdomen for contractions. B. Tilt the backboard sideways to displace the uterus laterally. C. Obtain a blood sample for complete blood count. D. Infuse 1,000 mL normal saline using a large bare IV.

B. Tilt the backboard sideways to displace the uterus laterally.

The nurse is caring for a 35-week gestation infant delivered by cesarean section 2 hours ago. The nurse observes the infant's respiratory rate is 72 breaths/minute with nasal flaring, grunting, and retractions. The nurse should recognize these findings indicate which complication? A. Persistent pulmonary hypertension of the newborn. B. Transient tachypnea of the newborn. C. Meconium aspiration syndrome. D. Bronchopulmonary dysplasia.

B. Transient tachypnea of the newborn.

14. A 31-year-old woman uses an over-the-counter (OTC) pregnancy test that is positive one week after a missed period. At the clinic, the client tells the nurse she takes phenytoin (Dilantin) for epilepsy, has a history of irregular periods, is under stress at work, and has not been sleeping well. The clients physical examination and ultrasound do not indicate that she is pregnant. How should the nurse explain the most likely cause for obtaining false-positive pregnancy test results? A. Being under too much stress at work. B. Using an anticonvulsant for epilepsy. C. Having an irregular menstrual cycle. D. Taking the pregnancy test too early.

B. Using an anticonvulsant for epilepsy.

A client with gestational hypertension is in active labor and receiving an infusion of magnesium sulfate. Which drug should the nurse have available for signs of potential toxicity? A. oxytocin (pitocin) B. calcium gluconate C. terbutaline (Brethine) D. naloxone (narcan)

B. calcium gluconate

A client who has an autosomal dominant inherited disorder is exploring family planning options and the risk of transmission of the disorder to the infant. The nurse's response should be based on what information? A. males inherit the disorder with a greater frequency than females B. each pregnancy carries a 50% chance of inheriting the disorder C. the disorder occurs in 25% of pregnancies D. all children will be carriers of the disorder

B. each pregnancy carries a 50% chance of inheriting the disorder

A client in active labor complains of cramps in her leg. What intervention should the nurse implement? A. ask if she takes a daily calcium tablet B. extend the leg and dorsiflex the foot C. lower the leg off the side of the bed D. elevate the leg above the heart

B. extend the leg and dorsiflex the foot. "Toes to the nose"

A primigravida at 40 weeks gestation is receiving oxytocin (Pitocin) to augment labor. Which adverse effect should the nurse monitor for during the infusion of Pitocin? A. dehydration B. hyperstimulation C. galactorrhea D. fetal tachycardia

B. hyperstimulation. Pitocin causes the uterine myofibril to contract, so unless the infusion is closely monitored, the client is at risk for hyperstimulation which can lead to tetanic contractions, uterine rupture, and fetal distress or demise.

A full term infant is transferred to the nursery from labor and delivery. Which information is most important for the nurse to receive when planning immediate care for the newborn? A. length of labor and method of delivery B. infant's condition at birth and treatment received C. feeding method chosen by the parents D. history of drugs given to the mother during labor

B. infant's condition at birth and treatment received

A client who is attending antepartum classes asks the nurse why her healthcare provider has prescribed iron tablets. The nurse's response is based on what knowledge? A. supplementary iron is more efficiently utilized during pregnancy B. it is difficult to consume 18 mg of additional iron by diet alone. C. iron absorption is decreased in the GI tract during pregnancy D. iron is needed to prevent megaloblastic anemia in the last trimester

B. it is difficult to consume 18 mg of additional iron by diet alone.

During a prenatal visit, the nurse discusses with a client the effects of smoking on the fetus. When compared with nonsmokers, mothers who smoke during pregnancy tend to produce infants who have A. lower Apgar scores B. lower birth weights C. respiratory distress D. a higher rate of congenital anomalies

B. lower birth weights

A 30 year old gravida 2, para 1 client is admitted to the hospital at 26 weeks gestation in preterm labor. She is given a dose of terbutaline sulfate (Brethine) 0.25 mg SQ. Which assessment is the highest priority for the nurse to monitor during the adminstration of this drug? A. maternal blood pressure and respirations B. maternal and fetal heart rates C. hourly urinary output D. deep tendon reflexes

B. maternal and fetal heart rates

A 30 year old gravida, 2 para 1 client is admitted to the hospital at 26 weeks gestation in preterm labor. She is started on an IV solution of terbutaline (Brethine). Which assessment is the highest priority for the nurse to monitor during the administration of this drug? A. maternal blood pressure and respirations B. maternal and fetal heart rates C. hourly urinary output D. deep tendon reflexes

B. maternal and fetal heart rates

The nurse identifies crepitus when examining the chest of the newborn who was delivered vaginally. Which further assessment should the nurse perform? A. elicit a positive scarf sign on the affected side B. observe for an asymmetrical Moro (startle) reflex C. Watch for swelling of fingers on the affected side D. Note paralysis of affected extremity and muscles

B. observe for an asymmetrical Moro (startle) reflex. The most common neonatal birth trauma due to a vaginal delivery is fracture of the clavicle. Although an infant may be asymptomatic, a fractured clavicle should be suspected if the infant has limited use of the affected arm, malposition of the arm, an asymmetric Moro reflex, crepitus over the clavicle, focal swelling or tenderness, or cries when the arm is moved.

A primipara has delivered a stillborn fetus at 30 weeks gestation. To asses the parents in the grieving process which intervention is most for the nurse to implement ? A. explain the possible cause of the fetal demise B. Provide a time for the parents to hold their infant in privacy C. Encourage the parents to seek counseling within the next few weeks D. Assist the couple to request autopsy

B. provide a time for the parents to hold their infant in privacy

The nurse teaching a preconception preparation class is discussing ways to improve dietary folic acid intake. Which evening snack contains the most folic acid? A. Fresh strawberries B. Roasted peanuts in shell C. Unflavored yogurt D. Vanilla milkshake with protein supplement

B. roasted peanut in shell

The nurse should explain to a 30 year old gravid client that alpha fetoprotein testing is recommended for which purpose? A. detect cardiovascular disorders B. screen for neural tube defects C. monitor for placental functioning D. assess for maternal pre-eclampsia

B. screen for neural tube defects

A full-term infant is transferred to the nursery from labor and delivery. Which information is most important for the nurse to receive when planning immediate care for the newborn? A. the length of labor and method of delivery B. the infant's condition at birth and treatment received C. the feeding method chosen by the parents D. the history of drugs given to the mother during labor

B. the infant's condition at birth and treatment received

A new mother asks the nurse, "How do I know that my daughter is getting enough breast milk?" Which explanation is appropriate? A."Weigh the baby daily, and if she is gaining weight, she is getting enough to eat." B."Your milk is sufficient if the baby is voiding pale, straw-colored urine six to ten times a day." C."Offer the baby extra bottled milk after her feeding and see if she still seems hungry." D."If you're concerned, you might consider bottle feeding so that you can monitor intake."

B."Your milk is sufficient if the baby is voiding pale, straw-colored urine six to ten times a day." Rationale: The urine will be dilute (straw-colored) and frequent (>6 to 10 times/day), if the infant is adequately hydrated. Although a weight gain of 30 g/day is indicative of adequate nutrition, most home scales do not measure this accurately, and the suggestion will likely make the mother anxious. Option C causes nipple confusion and diminishes the mother's milk production. Option D does not address the client's question.

A client who is 3 days postpartum and breastfeeding asks the nurse how to reduce breast engorgement. Which instruction should the nurse provide? A.Avoid using the breast pump. B.Breastfeed the infant every 2 hours. C.Reduce fluid intake for 24 hours. D.Skip feedings to let the sore breasts rest.

B.Breastfeed the infant every 2 hours. Rationale: The mother should be instructed to attempt feeding her infant every 2 hours while massaging the breasts as the infant is feeding. If the infant does not feed adequately and empty the breast, using a breast pump helps extract the milk and relieve some of the discomfort. Dehydration irritates swollen breast tissue. Skipping feedings may cause further engorgement and discomfort.

Which maternal behavior is the nurse most likely to see when a new mother receives her infant for the first time? A.She eagerly reaches for the infant, undresses the infant, and examines the infant completely. B.Her arms and hands receive the infant and she then traces the infant's profile with her fingertips. C.Her arms and hands receive the infant and she then cuddles the infant to her own body. D.She eagerly reaches for the infant and then holds the infant close to her own body.

B.Her arms and hands receive the infant and she then traces the infant's profile with her fingertips. Rationale:Attachment and bonding theory indicates that most mothers will demonstrate behaviors described in option B during the first visit with the newborn, which may be at delivery or later. After the first visit, the mother may exhibit different touching behaviors such as eagerly reaching for the infant and cuddling the infant close to her

A primigravida, when returning for the results of her multiple marker screening (triple screen), asks the nurse how problems with her baby can be detected by the test. What information will the nurse give to the client to describe best how the test is interpreted? A.If MSAFP (maternal serum alpha-fetoprotein) and estriol levels are high and the human chorionic gonadotropin (hCG) level is low, results are positive for a possible chromosomal defect. B.If MSAFP and estriol levels are low and the hCG level is high, results are positive for a possible chromosomal defect. C.If MSAFP and estriol levels are within normal limits, there is a guarantee that the baby is free of all structural anomalies. D.If MSAFP, estriol, and hCG are absent in the blood, the results are interpreted as normal findings.

B.If MSAFP and estriol levels are low and the hCG level is high, results are positive for a possible chromosomal defect. Rationale:Low levels of MSAFP and estriol and elevated levels of hCG found in the maternal blood sample are indications of possible chromosomal defects. High levels of MSAFP and estriol in the blood sample after 15 weeks of gestation can indicate a neural tube defect, such as spina bifida and anencephaly, not chromosomal defects. One of the limitations of the multiple marker screening is that any defects covered by skin will not be evident in the blood sampling. After 15 weeks of gestation, there will be traces of MSAFP, estriol, and hCG in the blood sample.

The nurse calls a client who is 4 days postpartum to follow up about her transition with her newborn son at home. The woman tells the nurse, "I don't know what is wrong. I love my son, but I feel so let down. I seem to cry for no reason!" Which adjustment phase should the nurse determine the client is experiencing? A.Taking-in phase B.Postpartum blues C.Attachment difficulty D.Letting-go phase

B.Postpartum blues Rationale: During the postpartum period, when serum hormone levels fall, women are emotionally labile, often crying easily for no apparent reason. This phase is commonly called postpartum blues, which peaks around the fifth postpartum day. The taking-in phase is the period following birth when the mother focuses on her own psychological needs; typically, this period lasts for 24 hours. Crying is not a maladaptive attachment response. It indicates a normal physical and emotional response. The letting-go phase is when the mother sees the child as a separate individual.

A primigravida is admitted to the birthing unit in early labor. A pelvic examination reveals that her cervix is 100% effaced and dilated 3 cm. The fetal head is at +1 station. In which area of the client's pelvis is the fetal occiput?

Below the ischial spine

The nurse places one hand above the symphysis while massaging the fundus of a multiparous client whose uterine tone is boggy 15 minutes after delivering a 7 pound 10 ounce infant. Which information should the nurse provide the client about this finding?

Both the lower uterine segment and the fundus must be massaged.

The nurse prepares to administer an injection of Vitamin K to a newborn infant. The mother tells the nurse, "Wait! I don't want my baby to have a shot" Which response would be best for the nurse to make? A) Explain that the Vit K shot is required by state law and compliance is mandatory B) inform the mother that the injection was prescribed by the Dr. C) Explore the mother's concerns about the infant receiving the shot. D) Remind the mother that all babies receive this shot and it is relatively painless.

C

What action should the nurse implement with the family when an infant is born with anencephaly? A) Ensure that measures to facilitate the attachment process are offered B) Inform the family about multiple corrective surgical procedures that will be needed C) Prepare the family to explore ways to cope with the imminent death of the infant D) Provide emotional support to facilitate the consideration of fetal organ donataion

C

When assessing the integument of a 24 hour newborn, the nurse notes a pink papular rash with superimposed vesicles on the thorax, back, and abdomen. What action should the nurse implement next? A) Move the newborn to isolation B) Obtain a culture from one of the vesticles C) Document the finding as erythema toxicum D) Notify the Dr. immediately

C

Which cardiovascular findings should the nurse assess further in a client at 20 weeks? A) A decrease in BP B) Increase in RBC production C) Decrease in pulse D) increase in heart sounds

C

Which nonpharmacological interventions should the nurse implement to provide the most effective response in decreasing procedural pain to the neonate? A) Skin to skin contact with parent B) Tactile stimualtion C) Oral sucrose and nonnutritive sucking D) Commercial warm pack

C

A primigravida at 12 weeks gestation who just moved to the USA indicates she has not received any immunizations. Which immunizations should the nurse administer at this time? (Select all) A) Chickenpox B) Rubella C) Hep B D) Diphtheria E) Tetanus

C D E

A new mother who has just had her first baby says to the nurse, "I saw the baby in the recovery room. She sure has a funny looking head" Which response by the nurse is best? A. "This is not an unusual shaped head, especially for a first baby." B. "It may look funny to you, but newborn babies are often born with heads like your baby's." C. "That is normal, the head will return to a round shape within 7 to 10 days." D. "Your pelvis was too small, so the baby's head had to adjust to the birth canal."

C. "That is normal, the head will return to a round shape within 7 to 10 days."

A new mother who has just had her first baby says to the nurse, "I saw the baby in the recovery room. She sure has a funny-looking head." Which response by the nurse is best? A. "This is not an unusually shaped head, especially for a first baby." B. "It may look odd, but newborn babies are often born with heads like that." C. "That is normal. The head will return to a round shape within 7 to 10 days." D. "Your pelvis was too small, so the head had to adjust to the birth canal."

C. "That is normal. The head will return to a round shape within 7 to 10 days."

A new mother who has just had her first baby says to the nurse, "I saw the baby in the recovery room. She sure has a funny-looking head." Which response by the nurse is best? A. "This is not an unusually shaped head, especially for a first baby." B. "It may look odd, but newborn babies are often born with heads like that." C. "That is normal. The head will return to a round shape within 7 to 10 days." D. "Your pelvis was too small, so the head had to adjust to the birth canal."

C. "That is normal. The head will return to a round shape within 7 to 10 days."

When assessing a pregnant woman who is 39 weeks gestation who is admitted to labor and delivery, which finding is most important to report the health care provider? A. + 1 proteinuria B. 130/70 blood pressure C. 102 oral temperature D. +1 pedal edema

C. 102 oral temperature

The nurse is using the Silverman-Anderson index to assess an infant with respiratory distress and determines that the infant is demonstrating marked nasal flaring, an audible expiratory grunt, and just visible intercostal and xiphoid retractions. Using this scale, which score should the nurse assign? A. 3 B. 4 C. 5 D. 8

C. 5

In determining the one minute Apgar score of a male infant the nurse asses a heart rate of 120 per min....respiration.. He has a loud cry with stimualtion, good muscle tone, color is acrocyanotic . What should the nurse assign? A. 7 B. 8 C. 9 D. 10

C. 9

49. The mother of a neonate asks the nurse why it is so important to keep the infant warm. What information should the nurse provide? A. The thick layer of subcutaneous fat is inadequate for insulation. B. Warmth promotes sleep so that the infant will grow quickly. C. A large body surface area favors heat loss to the environment. D. The kidneys and renal function are not fully developed.

C. A large body surface area favors heat loss to the environment.

32. A client at 28-weeks gestation arrives at the labor and delivery unit with a complaint of bright red, painless vaginal bleeding. For which diagnostic procedure should the nurse prepare the client? A. Contraction stress test. B. Lecithin-sphingomyelin ratio. C. Abdominal ultrasound. D. Internal fetal monitoring.

C. Abdominal ultrasound.

Prior to discharge, what instructions should the nurse give to parents regarding the newborn's umbilical cord care at home? A. Wash the cord frequently with mild soap and water. B. Cover the cord with a sterile dressing. C. Allow the cord to air-dry as much as possible. D. Apply baby lotion after the baby's daily bath.

C. Allow the cord to air-dry as much as possible.

Prior to discharge, what instructions should the nurse give to parents regarding the newborn's umbilical cord care at home? A. Wash the cord frequently with mild soap and water. B. Cover the cord with a sterile dressing. C. Allow the cord to air-dry as much as possible. D. Apply baby lotion after the baby's daily bath.

C. Allow the cord to air-dry as much as possible.

When planning care for a laboring client , the nurse identifies the need to withhold solid foods while the client is in labor . What is the most important reason for this nursing intervention? A. Gastric emptying time decreases during labor B. Nausea occurs from analgesic used during labor C. An increased risk for aspiration can occur if general analgesic is needed D. Autonomic nervous system stimulation during labor decreases peristalsis

C. An increased risk for aspiration can occur if general analgesic is needed

65. A nulliparous client telephones the labor and delivery unit to report that she is in labor. What action should the nurse implement? A. Suggest the client to come to the hospital for labor evaluation. B. Tell the woman to stay home until her membranes rupture. C. Ask the client to describe why she thinks she is in labor. D. Emphasize that food and fluid intake should stop.

C. Ask the client to describe why she thinks she is in labor.

A client in active labor is becoming increasingly fearful because her contractions are occurring more often than she had expected. Her partner is also becoming anxious. Which of the following should be the focus of the nurse's response? A. Telling the client and her partner that the labor process is often unpredictable B. Informing the client that this means she will give birth sooner than expected C. Asking the client and her partner if they would like the nurse to stay in the room D. Affirming that the fetal heart rate is remaining within normal limits

C. Asking the client and her partner if they would like the nurse to stay in the room

A client in active labor is becoming increasingly fearful because her contractions are occurring more often than she had expected. Her partner is also becoming anxious. Which of the following should be the focus of the nurse's response? A. Telling the client and her partner that the labor process is often unpredictable B. Informing the client that this means she will give birth sooner than expected C. Asking the client and her partner if they would like the nurse to stay in the room D. Affirming that the fetal heart rate is remaining within normal limits

C. Asking the client and her partner if they would like the nurse to stay in the room

A laboring client's membranes rupture spontaneously. The nurse notices that the amniotic fluid is greenish-brown. What intervention should the nurse implement first? A. Turn the client to her left side B. Contact the healthcare provider C. Assess the fetal heart rate D. Check the cervical dilation

C. Assess the fetal heart rate

3. The nurse assesses a male newborn and determines that he has the following vital signs: axillary temperature 95.1 F, heart rate 136 beats/minute and a respiratory rate of 48 breaths/minute. Based on these findings, which action should the nurse take first? A. Notify the pediatrician of the infant's vital signs. B. Encourage the infant to take the breast or sugar water. C. Assess the infant's blood glucose level. D. Check the infant's arterial blood gases.

C. Assess the infant's blood glucose level.

The nurse is preparing to administer methylergonovine maleate (Methergine) to a postpartum client. Based on what assessment finding should the nurse withhold the drug? A. Respiratory rate of 22 breaths/min B. A large amount of lochia rubra C. Blood pressure 149/90 D. Positive Homan's sign

C. Blood pressure 149/90

A newborn's head circumference is 12inches and his chest measurement is 13 inches. The nurse notes that this infant has no molding, and was a breech presentation delivered by Cesarean section. What action should the nurse take based on these data? A. No action need be taken. It is normal for an infant born by Cesarean section to have a small head circumference. B. Notify the pediatrician immediately. These findings support the possibility of hydrocephalus. C. Call these findings to the attention of the pediatrician. The head/chest ratio is abnormal. D. Record the findings on the chart. They are within normal limits.

C. Call these findings to the attention of the pediatrician. The head/chest ratio is abnormal.

The nurse notes on the fetal monitor that a laboring client has a variable deceleration. Which action should the nurse implement first? A. Assess cervical dilation. B. Administer oxygen via facemask. C. Change the client's position. D. Turn off the oxytocin infusion.

C. Change the client's position.

A 3-month-old with myelomeningocele and atonic bladder is catheterized every 4 hours to prevent urinary retention. The home health nurse notes that the child has developed episodes of sneezing, urticaria, watery eyes, and a rash in the diaper area. What action is most important for the nurse to take? A. Auscultate the lungs for respiratory pneumonia. B. Draw blood to analyze for streptococcal infection C. Change to latex-free gloves when handling infant D. Apply zinc oxide to perineum with each diaper change

C. Change to latex-free gloves when handling infant

13. Which cardiovascular findings should the nurse assess further in a client who is at 20-weeks gestation? A. Decrease in blood pressure. B. Increase in red blood cell production. C. Decrease in pulse rate. D. Increase in heart sounds (S1, S2).

C. Decrease in pulse rate.

69. The nurse is giving discharge instructions for a client following a suction curettage for hydatidiform mole. The client asks why oral contraceptives are being recommended for the next 12 months. What information should the nurse provide? A. Molar reoccurrences are higher if conception occurs within 1 year after initial mutation. B. Pregnancy within 1 year decreases the chances of a future successful pregnancy. C. Diagnostic testing for human chorionic gonadotropin (hCG) levels are elevated by pregnancy. D. Oral contraceptives prevent a reoccurrence of a molar pregnancy.

C. Diagnostic testing for human chorionic gonadotropin (hCG) levels are elevated by pregnancy.

A multiparous women at 38 weeks gestation with a history of rapid progression of labor is admitted for induction due to signs and symptoms of pregnancy induced hypertension (PIH). One hour after the oxytocin infusion is initiated she complains of a headache. Her contractions are occurring every 1-2 mins , lasting 60-75 seconds and a vaginal exam reveals that her cervix is 90% and dilated 6 cm.What intervention is most important for the nurse to implement? A. Prepare for immediate delivery B. Measure deep tendon reflexes C. Discontinue the Pitocin infusion D. Turn the client to her left side

C. Discontinue the Pitocin infusion

24. A client in early labor is having uterine contractions every 3 to 4 minutes, lasting an average of 55 to 60 seconds. An internal uterine pressure catheter (IUPC) is inserted. The intrauterine pressure is 65 to 70 mmHg at the peak of the contraction and the resting tone is 6 to 10 mmHg. Based on this information, what action should the nurse implement? A. Bring the delivery table to the room. B. Prepare to administer an oxytocic. C. Document the findings in the client record. D. Notify the client's healthcare provider.

C. Document the findings in the client record.

A new mother is having trouble breastfeeding her newborn. The child is making frantic rooting motions and will not grasp the nipple. Which intervention should the nurse implement? A. Encourage frequent use of a pacifier so that the infant becomes accustomed to sucking. B. Hold the infant's head firmly against the breast until he latches onto the nipple. C. Encourage the mother to stop feeding for a few minutes and comfort the infant. D. Provide formula for the infant until he becomes calm, and then offer the breast again.

C. Encourage the mother to stop feeding for a few minutes and comfort the infant.

A new mother is having trouble breastfeeding her newborn. The child is making frantic rooting motions and will not grasp the nipple. Which intervention should the nurse implement? A. Encourage frequent use of a pacifier so that the infant becomes accustomed to sucking. B. Hold the infant's head firmly against the breast until he latches onto the nipple. C. Encourage the mother to stop feeding for a few minutes and comfort the infant. D. Provide formula for the infant until he becomes calm, and then offer the breast again.

C. Encourage the mother to stop feeding for a few minutes and comfort the infant.

A mother who is breastfeeding her baby receives instructions from the nurse. Which instruction is most effective in preventing nipple soreness? A. Wear a cotton bra with nonbinding support. B. Increase nursing time gradually over several days. C. Ensure that the baby is positioned correctly for latching on. D. Manually express a small amount of milk before nursing.

C. Ensure that the baby is positioned correctly for latching on.

A mother who is breastfeeding her baby receives instructions from the nurse. Which instruction is most effective in preventing nipple soreness? A. Wear a cotton bra with nonbinding support. B. Increase nursing time gradually over several days. C. Ensure that the baby is positioned correctly for latching on. D. Manually express a small amount of milk before nursing.

C. Ensure that the baby is positioned correctly for latching on.

A client who delivered a healthy infant 5 days ago calls the clinic nurse and reports that her lochia is getting lighter in color. Which action should the nurse take? A. Instruct the client to go to the emergency room. B. Recommend vaginal douching. C. Explain this is a normal finding. D. Determine if ovulation has occurred.

C. Explain this is a normal finding.

The nurse is preparing a laboring client for an amniotomy. Immediately after the procedure is completed, it is most important for the nurse to obtain which information? A. Maternal blood pressure B. Maternal temperature C. Fetal heart rate (FHR) D. White blood cell count (WBC)

C. Fetal heart rate (FHR)

The nurse is preparing a laboring client for an amniotomy. Immediately after the procedure is completed, it is most important for the nurse to obtain which information? A.Maternal blood pressure B.Maternal temperature C.Fetal heart rate (FHR) D.White blood cell count (WBC)

C. Fetal heart rate (FHR) Rationale: The FHR should be assessed before and after the procedure to detect changes that may indicate the presence of cord compression or prolapse. An amniotomy (artificial rupture of membranes [AROM]) is used to stimulate labor when the condition of the cervix is favorable. The fluid should be assessed for color, odor, and consistency. Option A should be assessed every 15 to 20 minutes during labor but is not specific for AROM. Option B is monitored hourly after the membranes are ruptured to detect the development of amnionitis. Option D should be determined for all clients in labor.

71. Which client finding should the nurse document as a positive sign of pregnancy? A. A urine sample with a positive pregnancy test. B. Presence of Braxton Hicks contractions. C. Fetal heart tones (FHT) heard with a doppler. D. Last menstrual cycle occurred 2 months ago.

C. Fetal heart tones (FHT) heard with a doppler.

The nurse is teaching care of the newborn to a childbirth preparation class and describes the need for administering antibiotic ointment into the eyes of the newborn. An expectant father asks, "What type of disease causes infections in babies that can be prevented by using this ointment?" Which response by the nurse is accurate? A. Herpes B. Trichomonas C. Gonorrhea D. Syphilis

C. Gonorrhea

The nurse is teaching care of the newborn to a childbirth preparation class and describes the need for administering antibiotic ointment into the eyes of the newborn. An expectant father asks, "What type of disease causes infections in babies that can be prevented by using this ointment?" Which response by the nurse is accurate? A. Herpes B. Trichomonas C. Gonorrhea D. Syphilis

C. Gonorrhea

The nurse is teaching care of the newborn to a group of prospective parents and describes the need for administering antibiotic ointment into the eyes of the newborn. Which infectious organism will this treatment prevent from harming the infant? A. Herpes B. Staphylococcus C. Gonorrhea D. Syphilis

C. Gonorrhea. Erythromycin ointment is instilled into the lower conjunctiva of each eye within 2 hours after birth to prevent ophthalmica neonatorum, and infection caused by gonorrhea, and including conjunctivitis,, an infection caused by chlamydia. The infant may be exposed to these bacteria when passing through the birth canal.

A 25-year-old client has a positive pregnancy test. One year ago she had a spontaneous abortion at 3 months of gestation. Which is the correct description of this client that should be documented in the medical record? A. Gravida 1, para 0 B. Gravida 1, para 1 C. Gravida 2, para 0 D. Gravida 2, para 1

C. Gravida 2, para 0

The nurse instructs a laboring client to use accelerated blow breathing. The client begins to complain of tingling fingers and dizziness. Which action should the nurse take? A. Administer oxygen by face mask. B. Notify the health care provider of the client's symptoms. C. Have the client breathe into her cupped hands. D. Check the client's blood pressure and fetal heart rate.

C. Have the client breathe into her cupped hands.

The nurse instructs a laboring client to use accelerated blow breathing. The client begins to complain of tingling fingers and dizziness. Which action should the nurse take? A. Administer oxygen by facemask. B. Notify the health care provider of the client's symptoms. C. Have the client breathe into her cupped hands. D. Check the client's blood pressure and fetal heart rate.

C. Have the client breathe into her cupped hands.

The nurse instructs a laboring client to use accelerated-blow breathing. The client begins to complain of tingling fingers and dizziness. What action should the nurse take? A. administer oxygen by face mask B. notify the healthcare provider of the client's symptoms C. have the client breathe into her cupped hands D. check the client's blood pressure and fetal heart rate

C. Have the client breathe into her cupped hands.

22. A client in the first stage of active labor is using a shallow pattern of rapid breaths that is twice the normal adult breathing rate. The client complains of feeling light-headed, dizzy, and states that her fingers are tingling. What action should the nurse implement? A. Notify the healthcare provider. B. Administer oxygen via nasal cannula. C. Help her breathe into a paper bag. D. Tell the client to slow her breathing.

C. Help her breathe into a paper bag.

A client who delivered by cesarean section 24 hours ago is using a patient-controlled analgesia (PCA) pump for pain control. Her oral intake has been ice chips only since surgery. She is now complaining of nausea and bloating and states that because she has had nothing to eat, she is too weak to breastfeed her infant. Which nursing diagnosis has the highest priority? A. Altered nutrition, less than body requirements for lactation B. Alteration in comfort related to nausea and abdominal distention C. Impaired bowel motility related to pain medication and immobility D. Fatigue related to cesarean delivery and physical care demands of infant

C. Impaired bowel motility related to pain medication and immobility

A client who delivered by cesarean section 24 hours ago is using a patient-controlled analgesia (PCA) pump for pain control. Her oral intake has been ice chips only since surgery. She is now complaining of nausea and bloating, and states that because she had nothing to eat, she is too weak to breastfeed her infant. Which nursing diagnosis has the highest priority? A. Altered nutrition, less than body requirements for lactation B. Alteration in comfort related to nausea and abdominal distention C. Impaired bowel motility related to pain medication and immobility D. Fatigue related to cesarean delivery and physical care demands of infant

C. Impaired bowel motility related to pain medication and immobility

At 0600 while admitting a woman for a scheduled repeat c-section the client tells the nurse that she had coffee at 0400 because she wanted avoid getting a headache. What action should the nurse take first? A. Ensure preoperative lab results are available B. Start IV presecribed Lactated ringers C. Inform the anesthesia care provider D. Contact the client obstetrician

C. Inform the anesthesia care provider

An infant born to a heorin addict mother is admitted to the neonatal care unit. . What behaviors can the baby exhibit? A. Lethargy and a poor suck B. Facial abnormalities and microcephaly C. Irritability and high pitched cry D. Low birth weight and intrauterine growth retardation

C. Irritability and high pitched cry

The nurse is counseling a client who wants to become pregnant. She tells the nurse that she has a 36-day menstrual cycle and the first day of her last menstrual period was January 8. When will the client's next fertile period occur? A. January 14 to 15 B. January 22 to 23 C. January 29 to 30 D. February 6 to 7

C. January 29 to 30

The nurse is counseling a client who wants to become pregnant. She tells the nurse that she has a 36-day menstrual cycle and the first day of her last menstrual period was January 8. When will the client's next fertile period occur? A. January 14 to 15 B. January 22 to 23 C. January 29 to 30 D. February 6 to 7

C. January 29 to 30

A 16 year old gravida 1, para 0 client has just been admitted to the hospital with a diagnosis of eclampsia. She is not presently convulsing. Which intervention should the nurse plan to include in this client's nursing care plan? A. Assess temperature every hour. B. Monitor blood pressure, pulse, and respirations every 4 hours. C. Keep an airway at the bedside. D. Allow family visitation

C. Keep an airway at the bedside.

35. The nurse is assisting with the insertion of a pulmonary artery catheter (PAC)for a client at 32-weeks gestation who has severe preeclampsia with edema. As the PAC enters the right ventricle, what is the priority nursing assessment? A. Observe for maternal blood pressure changes. B. Assess fetal response to the procedure. C. Monitor for premature ventricular contractions. D. Note to any complaint of sudden chest pain.

C. Monitor for premature ventricular contractions.

A 38-week primagravida who works as a secretary and sits at a computer 8 hours each day tells the nurse that her feet have begun to swell. Which instruction would be most effective in preventing pooling of blood in the lower extremities? A. Wear support stockings B. Reduce salt in her diet C. Move about every hour D. Avoid constrictive clothing

C. Move about every hour

A 38-week primigravida who works as a secretary and sits at a computer 8 hours each day tells the nurse that her feet have begun to swell. Which instruction will aid in the prevention of pooling of blood in the lower extremities? A. Wear support stockings. B. Reduce salt in the diet. C. Move about every hour. D. Avoid constrictive clothing.

C. Move about every hour.

43. A macrosomic infant is in stable condition after a difficult forceps-assisted delivery. After obtaining the infant's weight at 4550 grams (9 lbs., 6 oz.), what is the priority nursing action? A. Assess newborn reflexes for signs of neurological impairment. B. Leave the infant in the room with the mother to foster attachment. C. Obtain serum glucose levels frequently while observing for signs of hypoglycemia. D. Perform a gestational age assessment to determine if the infant is large-for-gestational-age.

C. Obtain serum glucose levels frequently while observing for signs of hypoglycemia.

A neonate who has congenital adrenal hypoplasia (CAH) presents with ambiguous genitalia. What is the primary nursing consideration when supporting the parents of a child with this anomaly? A. Discuss the need for cortisol and aldosterone replacement therapy after discharge B. Support the parents in their decision to assign sex of their child according to their preference C. Offer information about ultrasonography and genotyping to determine sex assignment D. Explain that corrective surgical procedures consistent with sex assignment can be delayed

C. Offer information about ultrasonography and genotyping to determine sex assignment

56. Which nonpharmacological interventions should the nurse implement to provide the most effective response in decreasing procedural pain in a neonate? A. Commercial warm packs. B. Tactile stimulation. C. Oral sucrose and nonnutritive sucking. D. Skin-to-skin contact with parent.

C. Oral sucrose and nonnutritive sucking.

Following the vaginal delivery of a 10-pound infant, the nurse assesses a new mother's vaginal bleeding and finds that she has saturated two pads in 30 minutes and has a boggy uterus. What action should the nurse take first? A. Increase oxytocin IV infusion. B. Have the client empty her bladder. C. Perform fundal massage until firm. D. Inspect the perineum for lacerations.

C. Perform fundal massage until firm.

A mother spontaneously delivers her infant in a taxi cab on the way to the hospital. The emergency room nurse reports that the mother has active herpes (HSVII) lesion on the vulva. What intervention should the nurse implement first when admitting the neonate in the nursery? A. Obtain blood specimen for serum glucose level B. Document the temperature on the flow sheet C. Place newborn in the isolation area of the nursery D. Administer Vitamin K injection

C. Place newborn in the isolation area of the nursery

The parents of a male newborn have signed an informed consent for circumcision. What priority intervention should the nurse implement upon completion of the circumcision? A. Offer a pacifier dipped in glucose water. B. Give PRN dose of liquid acetaminophen. C. Place petrolatum gauze dressing on the site. D. Wrap the infant in warm receiving blankets.

C. Place petrolatum gauze dressing on the site.

A mother expresses fear about changing the infant's diaper after circumcision. What information should the nurse include in the teaching plan? A. Cleanse the penis with prepackaged diaper wipes every 3 to 4 hours. B. Wash off the yellow exudate on the glans once every day to prevent infection. C. Place petroleum ointment around the glans with each diaper change and cleansing. D. Apply pressure by squeezing the penis with the fingers for 5 minutes if bleeding occurs.

C. Place petroleum ointment around the glans with each diaper change and cleansing.

A mother expresses fear about changing the infant's diaper after circumcision. What information should the nurse include in the teaching plan? A. Cleanse the penis with prepackaged diaper wipes every 3 to 4 hours. B. Wash off the yellow exudate on the glans once every day to prevent infection. C. Place petroleum ointment around the glans with each diaper change and cleansing. D. Apply pressure by squeezing the penis with the fingers for 5 minutes if bleeding occurs.

C. Place petroleum ointment around the glans with each diaper change and cleansing.

Upon admission to the nursery, the nurse places a newborn supine under radiant warmer , an external heat source. What should the nurse implement first to ensure safe thermoregulation? A. Dry the newborns scalp and place a stockinet cap on the head B. Move the temperature probe over the ribs when turning to a lateral position C. Place temperature probe on the abdomen in the line with the radiant heat source D. Wrap the infant in two blankets and place the radiant warmer on low

C. Place temperature probe on the abdomen in the line with the radiant heat source

Twenty minutes after a continuous epidural anesthetic is administered, a laboring client's blood pressure drops from 120/80 mm Hg to 90/60 mm Hg. Which action should the nurse take immediately? A. Notify the health care provider or anesthesiologist. B. Continue to assess the blood pressure every 5 minutes. C. Place the client in a lateral position. D. Turn off the continuous epidural.

C. Place the client in a lateral position.

Twenty minutes after a continuous epidural anesthetic is administered, a laboring client's blood pressure drops from 120/80 to 90/60 mm Hg. Which action should the nurse take immediately? A. Notify the health care provider or anesthesiologist. B. Continue to assess the blood pressure every 5 minutes. C. Place the client in a lateral position. D. Turn off the continuous epidural.

C. Place the client in a lateral position.

Six hours after an oxytocin (Pitocin) induction was begun and 2 hours after spontaneous rupture of the membranes, the nurse notes several sudden decreases in the fetal heart rate with quick return to baseline, with and without contractions. Based on this fetal heart rate pattern, which intervention is best for the nurse to implement? A. Turn the client to her side. B. Begin oxygen by nasal cannula at 2 L/min. C. Place the client in a slight Trendelenburg position. D. Assess for cervical dilation.

C. Place the client in a slight Trendelenburg position.

10 A client at 8-months gestation tells the nurse that she knows her baby listens to her, but her husband thinks she is imagining things. What information should the nurse provide? A. The interaction between the mother's voice and the fetus's response ensures bonding. B. The healthcare provider should address her concerns about her baby's hearing function. C. The fetus in utero is capable of hearing and does respond to the mother's voice. D. Many women imagine what their baby is like by interpreting fetal movements.

C. The fetus in utero is capable of hearing and does respond to the mother's voice.

Six hours after an oxytocin (Pitocin) induction was begun and 2 hours after spontaneous rupture of the membranes, the nurse notes several sudden decreases in the fetal heart rate with quick return to baseline, with and without contractions. Based on this fetal heart rate pattern, which intervention is best for the nurse to implement? A.Turn the client to her side. B.Begin oxygen by nasal cannula at 2 L/min. C.Place the client in a slight Trendelenburg position. D.Assess for cervical dilation.

C. Place the client in a slight Trendelenburg position. Rationale:The goal is to relieve pressure on the umbilical cord, and placing the client in a slight Trendelenburg position is most likely to relieve that pressure. The FHR pattern is indicative of a variable fetal heart rate deceleration, which is typically caused by cord compression and can occur with or without contractions. Option A may be helpful but is not as likely to relieve the pressure as the Trendelenburg position. Option B is not helpful with cord compression. Option D is not the priority intervention at this time. After repositioning the client, a vaginal examination is indicated to rule out cord prolapse and assess for cervical change.

An infant with tetralogy of Fallot becomes acutely cyanotic and hyper apneic. Which action should the nurse implement first?A. Administer morphine sulphate. B. Start IV fluids. C. Place the infant in a knee-chest position D. Provide 100% oxygen by face mask.

C. Place the infant in a knee-chest position

The nurse is preparing to administer phytonadione to a newborn. Which statement made by the parents indicates understanding why the nurse is administering this medication? A. Improve insufficient dietary intake. B. Stimulate the immune system. C. Prevent hemorrhagic disorders. D. Help an immature liver.

C. Prevent hemorrhagic disorders.

31. The nurse is providing discharge teaching for a gravid client who is being released from the hospital after placement of cerclage. Which instruction is the most important for the client to understand? A. Arrange for home uterine monitoring. B. Plan for a possible cesarean birth. C. Report uterine cramping or low backache. D. Make arrangements for care at home.

C. Report uterine cramping or low backache.

The nurse is assessing a 2-hour-old infant born by cesarean delivery at 39-weeks gestation. Which assessment finding should receive the highest priority when planning this infants care? A. Blood pressure 76/42 mm/Hg B. Faint heart murmur C. Respiratory rate 76 breaths/min D. Blood glucose 45 mg/dl

C. Respiratory rate 76 breaths/min

30. Which finding indicates to the nurse that a 4-day-old infant is receiving adequate breast milk? A. Gains 1 to 2 ounces per week. B. Defecates at least once per 24 hours. C. Saturates 6 to 8 diapers per day. D. Rests for 6 hours between feedings.

C. Saturates 6 to 8 diapers per day.

12. A woman, whose pregnancy is confirm, asks the nurse what the function of the placenta is in early pregnancy. What information supports the explanation that the nurse should provide? A. Produces nutrients for fetal nutrition. B. Forms a protective, impenetrable barrier. C. Secretes both estrogen and progesterone. D. Excretes prolactin and insulin.

C. Secretes both estrogen and progesterone.

A full-term, 24-hour-old infant in the nursery regurgitates and suddenly turns cyanotic. What should the nurse do first? A. Suction the oral and nasal passages B. Give oxygen by positive pressure C. Stimulate the infant to cry D. Turn the infant onto the right side

C. Stimulate the infant to cry

53. A neonate who is receiving an exchange transfusion for hemolytic disease develops respiratory distress, tachycardia, and a cutaneous rash. What nursing intervention should be implemented first? A. Inform the healthcare provider. B. Monitor vital signs electronically. C. Stop the infusion. D. Administer calcium gluconate.

C. Stop the infusion.

41. What nursing action should be included in the plan of care for a newborn experiencing symptoms of drug withdrawal? A. Play soft music and talk to soothe the infant. B. Feed every 4 to 6 hours to allow extra rest. C. Swaddle the infant snuggly and hold tightly. D. Administer chloral hydrate for sedation.

C. Swaddle the infant snuggly and hold tightly.

A 26-year-old gravida 2, para 1, client is admitted to the hospital at 28 weeks of gestation in preterm labor. She is given three doses of terbutaline sulfate (Brethine), 0.25 mg subcutaneously, to stop her labor contractions. What are the primary side effects of terbutaline sulfate? A. Drowsiness and paroxysmal bradycardia B. Depressed reflexes and increased respirations C. Tachycardia and a feeling of nervousness D. A flushed warm feeling and dry mouth

C. Tachycardia and a feeling of nervousness

A client at 30 weeks of gestation is on bed rest at home because of increased blood pressure. The home health nurse has taught her how to take her own blood pressure and gave her parameters to judge a significant increase in blood pressure. When the client calls the clinic complaining of indigestion, which instruction should the nurse provide? A. Lie on your left side and call 911 for emergency assistance. B. Take an antacid and call back if the pain has not subsided. C. Take your blood pressure now and if it is seriously elevated, go to the hospital. D. See your health care provider to obtain a prescription for a histamine blocking agent.

C. Take your blood pressure now and if it is seriously elevated, go to the hospital.

A client at 30 weeks of gestation is on bed rest at home because of increased blood pressure. The home health nurse has taught her how to take her own blood pressure and gave her parameters to judge a significant increase in blood pressure. When the client calls the clinic complaining of indigestion, which instruction should the nurse provide? A. Lie on your left side and call 911 for emergency assistance. B. Take an antacid and call back if the pain has not subsided. C. Take your blood pressure now, and if it is seriously elevated, go to the hospital. D. See your health care provider to obtain a prescription for a histamine blocking agent.

C. Take your blood pressure now, and if it is seriously elevated, go to the hospital.

A community health nurse visits a family in which a 16-year-old unmarried daughter is pregnant with her first child and is at 32-weeks gestation. The client tells the nurse that she has been having intermittent back pain since the night before. What is the priority nursing intervention? A. Ask the clients mother to call an ambulance for transport to the hospital immediately. B. Determine what physical activities the client has performed for the past 24 hours C. Teach the client if she has experienced any recent changes in vaginal discharge.

C. Teach the client if she has experienced any recent changes in vaginal discharge.

44. An infant who weighs 3.8 kg is delivered vaginally at 39-weeks gestation with a nuchal cord after a 30-minute second stage. The nurse identifies petechiae over the face and upper back of the newborn. What information should the nurse provide to parents about this finding? A. Further assessment is indicated. B. An increased blood volume causes broken blood vessels. C. The pinpoint spots are benign and disappear within 48 hours. D. Petechiae occurs with forceps delivery.

C. The pinpoint spots are benign and disappear within 48 hours.

A client with gestational diabetes is undergoing a non-stress test at 34 weeks gestation. Fetal heart beat is 144 beats / min. The client is instructed to mark the fetal monitor paper by pressing each time the baby moves. After 20 mins the nurse evaluates the fetal monitor strip A. The mother perceives and marks at least four fetal movements B. Fetal movements must be elicited with a vibroacoustic stimulator C. Two fetal heart accelerations of 15 beats/ min x 15 seconds are recorded D. No FHR late decelerations occur in response to fetal movement

C. Two FHR accelerations of 15 beats/minute x 15 seconds are recorded.

When preparing a class on newborn care for expectant parents, which is correct for the nurse to teach concerning the newborn infant born at term gestation? A. Milia are red marks made by forceps and will disappear within 7 to 10 days. B. Meconium is the first stool and is usually yellow gold in color. C. Vernix is a white cheesy substance, predominantly located in the skin folds. D. Pseudostrabismus found in newborns is treated by minor surgery.

C. Vernix is a white cheesy substance, predominantly located in the skin folds.

When preparing a class on newborn care for expectant parents, which is correct for the nurse to teach concerning the newborn infant born at term gestation? A. Milia are red marks made by forceps and will disappear within 7 to 10 days. B. Meconium is the first stool and is usually yellow gold in color. C. Vernix is a white cheesy substance, predominantly located in the skin folds. D. Pseudostrabismus found in newborns is treated by minor surgery.

C. Vernix is a white cheesy substance, predominantly located in the skin folds.

When preparing a class on newborn care for expectant parents, what content should the nurse teach concerning the newborn infant born at term gestation? A. Milia are red marks made by forceps and will disappear within 7-10 days B. Meconium is the first stool and is usually yellow gold in color C. Vernix is a white, cheesy substance, predominantly located in the skin folds D. Pseudostrabismus found in newborns is treated by minor surgery.

C. Vernix is a white, cheesy substance, predominantly located in the skin fold

A client who delivered a healthy infant 5 days ago calls the clinic nurse and reports that her lochia is getting lighter in color and asks when the flow will stop. How should the nurse respond? A. 2 weeks B. 10 days C. When the placental site has healed D.After the first time ovulation occurs

C. When the placental site has healed

A client at 32 weeks gestation is hospitalized with severe pregnancy-induced hypertension (PIH), and magnesium sulfate is prescribed to control symptoms. Which assessment finding would indicate that therapeutic drug level has been achieved? A. 4+ reflexes B. urinary output of 50 mL per hour C. a decrease in respiratory rate from 24 to 16 D. a decreased body temperature

C. a decrease in respiratory rate from 24 to 16

A newborn, whose mother is HIV positive, is scheduled for follow-up assessments. The nurse knows that the most likely presenting symptom for a pediatric client with AIDS is A. shortness of breath B. joint pain C. a persistent cold D. organomegaly

C. a persistent cold

The nurse is preparing a client with a term pregnancy who is in active labor for an amniotomy. What equipment should the nurse have available at the client's bedside? (select all that apply) A. litmus paper B. fetal scalp electrode C. a sterile glove D. an amnihook E. sterile vaginal speculum F. lubricant

C. a sterile glove D. an amnihook F. lubricant

The nurse is preparing a client with a term pregnancy who is in active labor for an amniotomy. What equipment should the nurse have available at the client's bedside? (select all that apply) A. Litmus paper B. fetal scalp electrode C. a sterile glove D. an amniotic hook E. sterile vaginal speculum F. a Doppler

C. a sterile glove D. an amniotic hook F. a Doppler

A newborn infant is brought to the nursery from the bathing suite. The nurse notices that the infant is breathing satisfactorily but appears dusky. What action should the nurse take first? A. notify the pediatrician B. suction the infant's nares, then the oral cavity C. check the infant's oxygen saturation rate D. position the infant on the right side.

C. check the infant's oxygen saturation rate

A client at 40-weeks' gestation presents to the obstetrical floor and indicates that the amniotic membranes ruptured spontaneously at home. She is in active labor and feels the need to bear down and push. What information is most important for the nurse to obtain first? A. the estimated amount of fluid B. time the membranes ruptured C. color and consistency of the fluid D. any odor noted when membranes ruptured.

C. color and consistency of the fluid

A mother who is breastfeeding her baby receives instructions from the nurse. Which instruction is most effective to prevent nipple soreness? A. wear a cotton bra B. increase nursing time gradually C. correctly place the infant on the breast D. manually express a small amount of milk before nursing

C. correctly place the infant on the breast

A client with no prenatal care arrives at the labor unit screaming, "the baby is coming!" The nurse performs a vaginal examination that reveals the cervix is 3 cm dilated and 75% effaced. What additional information is most important for the nurse to obtain? A. gravidity and parity B. time and amount of last oral intake C. date of last normal menstrual period D. frequency and intensity of contractions

C. date of last normal menstrual period

A 24-hour old newborn has a pink papular rash with vesicles superimposed on the thorax, back and abdomen. What action should the nurse implement? A. notify the healthcare provider B. move the newborn to an isolation nursery C. document the finding in the infant's record D. obtain a culture of the vesicles

C. document the finding in the infant's record. Erythema Toxicum (or erythema neonatorium) is a newborn rash that is commonly referred to as "flea bites." but is a normal finding that is documented in the infant's record and requires no further action.

The total bilirubin level of a 36 hour, breastfeeding newborn is 14 mg/dl. Based on this finding, which intervention should the nurse implement? A. provide phototherapy for 30 minutes q8h B. feed the newborn sterile water hourly C. encourage the mother to breastfeed frequently D. assess the newborn's blood glucose level

C. encourage the mother to breastfeed frequently

A client at 32 weeks gestation is diagnosed with preeclampsia. Which assessment is most indicative of an impending convulsion? A. 3+ deep tendon reflexes B. periorbital edema C. epigastric pain D. decreased urine output

C. epigastric pain. Epigastric pain is indicative of an edematous liver or pancreas which is an early warning sign of an impending convulsion (eclampsia) and requires immediate attention.

The nurse is teaching breastfeeding to prospective parents in a childbirth education class. Which instruction should the nurse include as content in the class? A. begin as soon as your baby is born to establish a four-hour feeding schedule B. resting helps with milk production. Ask that your baby be fed at night in the nursery C. feed your baby every 2 to 3 hours or on demand, whichever comes first. D. do not allow your baby to nurse any longer than the prescribed number of minutes

C. feed your baby every 2-3 hours or on demand, whichever comes first

At 14 weeks gestation, a client arrives at the emergency center complaining of a dull pain in the right lower quadrant of her abdomen. The nurse obtains a blood sample and initiates an IV. Thirty minutes after admission, the client reports feeling a sharp abdominal pain and a shoulder pain. Assessment findings include diaphoresis, a heart rate of 120 bpm, and a blood pressure of 86/48. Which action should the nurse implement next? A. check the hematocrit results B. administer pain medication C. increase the rate of IV fluids D. monitor client for contractions

C. increase the rate of IV fluids. The client is demonstrating symptoms of blood loss, probably the result of an ectopic pregnancy, which occurs at approximately 14 weeks gestation when embryonic growth expands the fallopian tube causing its rupture and can result in hemorrhage and hypovolemic shock. Increasing the IV infusion rate provides intravascular fluid to maintain blood pressure.

The nurse assesses a client admitted to the labor and delivery unit and obtains the following data: dark red vaginal bleeding, uterus slightly tense between contractions, BP 110/68, FHR 110 bpm, cervix 1 cm dilated and uneffaced. Based on these assessment findings, what intervention should the nurse implement? A. insert an internal fetal monitor B. assess for cervical changes q1h C. monitor for bleeding from IV sites D. perform Leopold's maneuvers

C. monitor for bleeding from IV sites. This client is presenting with signs of placental abruption. Disseminated intravascular coagulation (DIC) is a complication of placental abruptio, characterized by abnormal bleeding.

A client at 30-weeks gestation, complaining of pressure over the pubic area, is admitted for observation. She is contracting irregularly and demonstrates underlying uterine irritability. Vaginal examination reveals that her cervix is closed, thick and high. Based on this data, which intervention should the nurse implement first? A. provide oral hydration B. have a complete blood count (CBC) drawn C. obtain a specimen for urine analysis D. place the client on strict bedrest

C. obtain a specimen for urine analysis. This should be done first because preterm clients with uterine irritability and contractions are often suffering from a UTI, and this should be ruled out first.

A client is admitted with the diagnosis of total placenta previa. Which finding is most important for the nurse to report to the healthcare provider immediately? A. heart rate of 100 bpm B. variable fetal heart rate C. onset of uterine contractions D. burning on urination

C. onset of uterine contractions. Total (complete) placenta previa involves the placenta covering the entire cervical os (opening). The onset of uterine contractions places the client at risk for dilation and placental separation, which causes painless hemorrhaging.

Twenty minutes after a continuous epidural anesthetic is administered, a laboring client's blood pressure drops from 120/80 to 90/60. What action will the nurse take? A. notify the healthcare provider or anesthesiologist immediately. B. continue to assess the blood pressure q5 minutes C. place the woman in a lateral position D. turn off the continuous epidural

C. place the woman in a lateral position. The nurse should immediately turn the woman to a lateral position, place a pillow or wedge under the right hip to deflect the uterus, increase the rate of the main line IV infusion, and administer oxygen by face mask at 10-12 L/min. If the blood pressure remains low, especially if it further decreases ,the anesthesiologist/healthcare provider should be notified immediately. Turning off the continuous epidural may also be warranted, but such action is based on hospital protocol.

The nurse observes a new mother is rooming-in and caring for her newborn infant. Which observation indicates the need for further teaching? A. cuddles the baby close to her B. rocks and soothes the infant in her arms C. places the infant prone in the bassinet D. wraps the baby in a warm blanket after bathing

C. places the infant prone in the bassinet. This is associated with an increased incidence of sudden infant death syndrome (SIDS)

The nurse who is working at a prenatal clinic notes a woman that is at 18 weeks of gestation has two elevated maternal alpha feto-protein (MSAFP) values. What action should the nurse implement? A. Instruct the client to increase intake of folic acid supplements B. Request a consultation with genetic counselor C. Schedule a sonogram in the radiology department D. Send the client to the laboratory for repeat MSAFP

C. schedule a sonogram in the radiology department

A couple, concerned because the woman has not been able to conceive, is referred to a healthcare provider for a fertility workup and a hysterosalpingography is scheduled. Which complaint would indicate to the nurse that that woman's fallopian tubes are patient? A. back pain B. abdominal pain C. shoulder pain D. leg cramps

C. shoulder pain

A 26-year old, gravida 2, para 1 client is admitted to the hospital at 28 weeks gestation in preterm labor. She is given 3 doses of terbutaline sulfate (Brethine) 0.25mg SQ to stop her labor contractions. The nurse plans to monitor for which primary side effect of terbutaline sulfate? A. drowsiness and bradycardia B. depressed reflexes and increased respirations C. tachycardia and a feeling of nervousness D. a flushed, warm feeling and a dry mouth

C. tachycardia and a feeling of nervousness

A 23 year old client who is receiving Medicaid is pregnant with her first child. Based on knowledge of the statistics related to infant mortality, which plan should the nurse implement with this client? A. refer the client to a social worker to arrange for home care B. recommend prenatal care from an obstetrician, not a nurse midwife C. teach the client why keeping prenatal care appointments is important D. Advise the client that neonatal intensive care may be needed

C. teach the client why keeping prenatal care appointments is important

The nurse should encourage the laboring client to begin pushing when A. there is only an anterior or posterior lip of cervix left. B. the client describes the need to have a bowel movement. C. the cervix is completely dilated D. the cervix is completely effaced

C. the cervix is completely dilated

The nurse is assessing the umbilical cord of a newborn. Which finding constitutes a normal finding? A. two vessels: one artery and one vein B. two vessels: two arteries and no veins C. three vessels: two arteries and one vein D. three vessels: Two veins and one artery

C. three vessels: two arteries and one vein

A new mother who has just had her first baby says to the nurse, "I saw the baby in the recovery room. She sure has a funny-looking head." Which response by the nurse is best? A."This is not an unusually shaped head, especially for a first baby." B."It may look odd, but newborn babies are often born with heads like that." C."That is normal. The head will return to a round shape within 7 to 10 days." D."Your pelvis was too small, so the head had to adjust to the birth canal."

C."That is normal. The head will return to a round shape within 7 to 10 days." Rationale: Option C reassures the mother that this is normal in the newborn and provides correct information regarding the return to a normal shape. Although option A is correct, it implies that the client should not worry. Any implied or spoken "don't worry" is usually the wrong answer. Option B is condescending and dismissing; the mother is seeking reassurance and information. Option D is a negative statement and implies that molding is the mother's fault.

The nurse is using the Silverman-Anderson index to assess an infant with respiratory distress and determines that the infant is demonstrating marked nasal flaring, an audible expiratory grunt, and just visible intercostal and xiphoid retractions. Using this scale, which score should the nurse assign? A.3 B.4 C.5 D.8

C.5 Rationale: The Silverman-Anderson index is an assessment scale that scores a newborn's respiratory status as grade 0, 1, or 2 for each component; it includes synchrony of the chest and abdomen, retractions, nasal flaring, and expiratory grunt. No respiratory distress is graded 0, and a total of 10 indicates maximum respiratory distress. This infant is demonstrating respiratory distress with maximal effort, so a grade 2 is assigned for marked nasal flaring, grade 2 for an audible expiratory grunting, plus grade 1 for just visible retractions, which is a total score of 5. Options A, B, and D are not accurate.

Prior to discharge, what instructions should the nurse give to parents regarding the newborn's umbilical cord care at home? A.Wash the cord frequently with mild soap and water. B.Cover the cord with a sterile dressing. C.Allow the cord to air-dry as much as possible. D.Apply baby lotion after the baby's daily bath

C.Allow the cord to air-dry as much as possible. Rationale:Recent studies have indicated that air drying or plain water application may be equal to or more effective than alcohol in the cord healing process. Options A, B, and D are incorrect because they promote moisture and increase the potential for infection.

A client in active labor is becoming increasingly fearful because her contractions are occurring more often than she had expected. Her partner is also becoming anxious. Which of the following should be the focus of the nurse's response? A.Telling the client and her partner that the labor process is often unpredictable B.Informing the client that this means she will give birth sooner than expected C.Asking the client and her partner if they would like the nurse to stay in the room D.Affirming that the fetal heart rate is remaining within normal limits

C.Asking the client and her partner if they would like the nurse to stay in the room Rationale: Offering to remain with the client and her partner offers support without providing false reassurance. The length of labor is not always predictable, but options A and B do not offer the client the support that is needed at this time. Option D may be reassuring regarding the fetal heart rate but does not provide the client the emotional support she needs at this time during the labor process.

A new mother is having trouble breastfeeding her newborn. The child is making frantic rooting motions and will not grasp the nipple. Which intervention should the nurse implement? A.Encourage frequent use of a pacifier so that the infant becomes accustomed to sucking. B.Hold the infant's head firmly against the breast until he latches onto the nipple. C.Encourage the mother to stop feeding for a few minutes and comfort the infant. D.Provide formula for the infant until he becomes calm, and then offer the breast again.

C.Encourage the mother to stop feeding for a few minutes and comfort the infant. Rationale: The infant is becoming frustrated and so is the mother; both need a time out. The mother should be encouraged to comfort the infant and to relax herself. After such a time out, breastfeeding is often more successful. Options A and D would cause nipple confusion. Option B would only cause the infant to be more resistant, resulting in the mother and infant becoming more frustrated.

A mother who is breastfeeding her baby receives instructions from the nurse. Which instruction is most effective in preventing nipple soreness? A.Wear a cotton bra with nonbinding support. B.Increase nursing time gradually over several days. C.Ensure that the baby is positioned correctly for latching on. D.Manually express a small amount of milk before nursing.

C.Ensure that the baby is positioned correctly for latching on. Rationale: The most common cause of nipple soreness is incorrect positioning of the infant on the breast for latching on. The baby's body is in alignment with the ears, shoulders, and hips in a straight line, with the nose, cheeks, and chin touching the breast. Option A helps prevent chafing, and nonbinding support aids in prevention of discomfort from the stretching of the Cooper ligament. Option B is important but is not necessary for all women. Option D helps soften an engorged breast and encourages correct infant latching on but is not the best answer.

The nurse is teaching care of the newborn to a childbirth preparation class and describes the need for administering antibiotic ointment into the eyes of the newborn. An expectant father asks, "What type of disease causes infections in babies that can be prevented by using this ointment?" Which response by the nurse is accurate? A.Herpes B.Trichomonas C.Gonorrhea D.Syphilis

C.Gonorrhea Rationale: Erythromycin ointment is instilled into the lower conjunctiva of each eye within 2 hours after birth to prevent ophthalmia neonatorum, an infection caused by gonorrhea, and inclusion conjunctivitis, an infection caused by Chlamydia. The infant may be exposed to these bacteria when passing through the birth canal. Ophthalmic ointment is not effective against option A, B, or D.

A 25-year-old client has a positive pregnancy test. One year ago she had a spontaneous abortion at 3 months of gestation. Which is the correct description of this client that should be documented in the medical record? A.Gravida 1, para 0 B.Gravida 1, para 1 C.Gravida 2, para 0 D.Gravida 2, para 1

C.Gravida 2, para 0 Rationale: This is the client's second pregnancy or second gravid event, so option C is correct. The spontaneous abortion (miscarriage) occurred at 3 months of gestation (12 weeks), so she is a para 0. Parity cannot be increased unless delivery occurs at 20 weeks of gestation or beyond. Option A does not take into account the current pregnancy, nor does option B, which also counts the miscarriage as a "para," an incorrect recording. Although option D is correct concerning gravidity, para 1 is incorrect.

A client who delivered by cesarean section 24 hours ago is using a patient-controlled analgesia (PCA) pump for pain control. Her oral intake has been ice chips only since surgery. She is now complaining of nausea and bloating and states that because she has had nothing to eat, she is too weak to breastfeed her infant. Which nursing diagnosis has the highest priority? A.Altered nutrition, less than body requirements for lactation B.Alteration in comfort related to nausea and abdominal distention C.Impaired bowel motility related to pain medication and immobility D.Fatigue related to cesarean delivery and physical care demands of infant

C.Impaired bowel motility related to pain medication and immobility Rationale: Impaired bowel motility caused by surgical anesthesia, pain medication, and immobility is the priority nursing diagnosis and addresses the potential problem of a paralytic ileus. Options A and B are both caused by impaired bowel motility. Option D is not as important as impaired motility.

The nurse is counseling a client who wants to become pregnant. She tells the nurse that she has a 36-day menstrual cycle and the first day of her last menstrual period was January 8. When will the client's next fertile period occur? A.January 14 to 15 B.January 22 to 23 C.January 29 to 30 D.February 6 to 7

C.January 29 to 30 Rationale: This client can expect her next period to begin 36 days from the first day of her last menstrual period. Her next period would begin on February 12. Ovulation occurs 14 days before the first day of the menstrual period. The client can expect ovulation to occur January 29 to 30. Options A, B, and D are incorrect.

A 38-week primigravida who works as a secretary and sits at a computer 8 hours each day tells the nurse that her feet have begun to swell. Which instruction will aid in the prevention of pooling of blood in the lower extremities? A.Wear support stockings. B.Reduce salt in the diet. C.Move about every hour. D.Avoid constrictive clothing.

C.Move about every hour. Rationale: Pooling of blood in the lower extremities results from the enlarged uterus exerting pressure on the pelvic veins. Moving about every hour will relieve pressure on the pelvic veins and increase venous return. Option A would increase venous return from varicose veins in the lower extremities but would be of little help with swelling. Option B might be helpful with generalized edema but is not specific for edematous lower extremities. Option D does not address venous return, and there is no indication in the question that constrictive clothing is a problem.

Twenty minutes after a continuous epidural anesthetic is administered, a laboring client's blood pressure drops from 120/80 to 90/60 mm Hg. Which action should the nurse take immediately? A.Notify the health care provider or anesthesiologist. B.Continue to assess the blood pressure every 5 minutes. C.Place the client in a lateral position. D.Turn off the continuous epidural.

C.Place the client in a lateral position. Rationale: The nurse should immediately turn the client to a lateral position or place a pillow or wedge under one hip to deflect the uterus. Other immediate interventions include increasing the rate of the main line IV infusion and administering oxygen by facemask. If the blood pressure remains low after these interventions or decreases further, the anesthesiologist or health care provider should be notified immediately. To continue to monitor blood pressure without taking further action could constitute malpractice. Option D may also be warranted, but such action is based on hospital protocol.

A 26-year-old gravida 2, para 1, client is admitted to the hospital at 28 weeks of gestation in preterm labor. She is given three doses of terbutaline sulfate (Brethine), 0.25 mg subcutaneously, to stop her labor contractions. What are the primary side effects of terbutaline sulfate? A.Drowsiness and paroxysmal bradycardia B.Depressed reflexes and increased respirations C.Tachycardia and a feeling of nervousness D.A flushed warm feeling and dry mouth

C.Tachycardia and a feeling of nervousness Rationale: Terbutaline sulfate (Brethine), a beta-sympathomimetic drug, stimulates beta-adrenergic receptors in the uterine muscle to stop contractions. The beta-adrenergic agonist properties of the drug may cause tachycardia, increased cardiac output, restlessness, headache, and a feeling of nervousness. Option A is not a side effect. Options B and D are side effects of magnesium sulfate.

A client at 30 weeks of gestation is on bed rest at home because of increased blood pressure. The home health nurse has taught her how to take her own blood pressure and gave her parameters to judge a significant increase in blood pressure. When the client calls the clinic complaining of indigestion, which instruction should the nurse provide? A.Lie on your left side and call 911 for emergency assistance. B.Take an antacid and call back if the pain has not subsided. C.Take your blood pressure now, and if it is seriously elevated, go to the hospital. D.See your health care provider to obtain a prescription for a histamine blocking agent.

C.Take your blood pressure now, and if it is seriously elevated, go to the hospital. Rationale: Checking the blood pressure for an elevation is the best instruction to give at this time. A blood pressure exceeding 140/90 mm Hg is indicative of preeclampsia. Epigastric pain can be a sign of an impending seizure (eclampsia), a life-threatening complication of gestational hypertension. Additional data are needed to confirm an emergency situation as described in option A. Options B and D ignore the threat to client safety posed by a significant increase in blood pressure.

A pregnant woman reports night sweats, sleep disturbances, and weight gain. Upon diagnosis, the woman has low levels of estrogen and is treated with hormonal therapy. Which teratogenic effect is likely to occur in the newborn?

Congenital defects of the female reproductive system

A nurse assesses a male newborn and determines that he has the following vital signs: Axillary temp of 95.1 F, HR of 136, RR of 48. Based on these findings, which action should the nurse take first? A) Check the infants ABG B) Notify the pediatrician of the vital signs C) Encourage the infant to breastfeed or drink sugar water D) Assess the infant's glucose level

D

What action should the nurse implement when caring for a newborn receiving phototherapy? A) Reposition every 6 hours B) Limit the intake of forumla C) Apply an oil based lotion to the skin D) Place an eyeshield over the eyes

D

Which action should the nurse implement when caring for a newborn immediately after birth? A) Foster parent-newborn attachment B) administer eye prophylaxis at vitamin K C) dry the newborn and wrapping in a blanket D) Keep the newborn's airway clear

D

Which client should the nurse report to the health care provider as needing a prescription for RhoGAM? A) Newborn with rising serum billirubin B) Woman whose blood group is AB+ C) Newborn whose COOMBS is negative D) A primigravida mother who is rH negative.

D

Which finding indicates to the nurse that a 4 day old infant is receiving adequate breast milk? A) Gains 1-2 ounces per week B) Rests for 6 hours between feedings C) Defecates at least once per 24 hours D) Saturates 6-8 diapers per day

D

Which procedure evaluates the effect of fetal movement on the fetal heart activity? A) Sonography B) Biophysical profile C) Contraction test D) Non-stress test

D

While assessing a newborn the nurse observes diffuse edema of the soft tissues of the scalp that cross the suture lines. How should the nurse document this finding? A) Molding B) Cephalohematoma C) Hemangioma D) Caput succedaneum

D

An expectant father tells the nurse he fears that his wife is "losing her mind." He states that she is constantly rubbing her abdomen and talking to the baby and that she actually reprimands the baby when it moves too much. Which recommendation should the nurse make to this expectant father? A.Suggest that his wife seek professional counseling to deal with her symptoms. B.Explain that his wife is exhibiting ambivalence about the pregnancy. C. Ask him to report similar abnormal behaviors at the next prenatal visit. D.Reassure him that normal maternal-fetal bonding is occurring.

D) Reassure him that normal maternal-fetal bonding is occurring. Rationale: These behaviors are positive signs of maternal-fetal bonding and do not reflect ambivalence. No intervention is needed. Quickening, the first perception of fetal movement, occurs at 17 to 20 weeks of gestation and begins a new phase of prenatal bonding during the second trimester. Options A and C are not necessary because the behaviors displayed are normal.

A client at 32 weeks gestation comes to the prenatal clinic with complaints of pedal edema, dyspnea, fatigue and a moist cough. Which question is most important for the nurse to ask this client? A. "Which symptom did you experience first?" B. "Are you eating large amounts of salty foods?" C. "Have you visited a foreign country recently?" D. "Do you have a history of rheumatic fever?"

D. "Do you have a history of rheumatic fever?" Clients with a history of rheumatic fever may develop mitral valve prolapse, which increases the risk for cardiac decompensation due to the increased blood volume that occurs during pregnancy, so obtaining information about this client's health history is a priority.

A client who is in the second trimester tells the nurse that she wants to use herbal therapy. Which response is best for the nurse to provide? A. "Herbs are a cornerstone of good health to include in your treatment." B. "Touch is also therapeutic in relieving discomfort and anxiety." C. "Your healthcare provider should direct treatment options for herbal therapy." D. "It is important that you want to take part in your care."

D. "It is important that you want to take part in your care."

A 30-year old multiparous woman who has a 3 year old boy and a newborn girl tells the nurse, "My son is so jealous of my daughter. I don't know who I'll ever manage both children when I get home." How should the nurse respond? A. "Tell the older child that he is a big boy now and should love his new sister." B. "Ask friends and relatives not to bring gifts to the older sibling because you do not want to spoil him." C. "Let the older child stay with his grandparents for the first 6 weeks to allow him to adjust to the newborn." D. "Regression in behaviors in the older child is a typical reaction so he needs attention at this time."

D. "Regression in behaviors in the older child is a typical reaction so he needs attention at this time." Preschool-aged children frequently regress in habits or behaviors, such as toileting and sleep habits, as a method of seeking attention so the parents should distribute their attention between the children and include the preschooler during infant care.

A new mother is afraid to touch her baby's head for fear of hurting the "large soft spot". Which explanation should the nurse give to this anxious client? A. "Some care is required when touching the large soft area on top of your baby's head until the bones fuse together." B. "That's just an 'old wives' tale' so don't worry, you can't harm your baby's head by touching the soft spot. C. "The soft spot will disappear within 6 weeks and is very unlikely to cause any problems for your baby." D. "There's a strong, tough membrane there to protect the baby so you need not be afraid to wash or comb his/her hair."

D. "There's a strong, tough membrane there to protect the baby so you need not be afraid to wash or comb his/her hair." The anterior fontanel or "large soft spot" normally closes at 12-18 months of age.

A pregnant woman comes to the prenatal clinic for an initial visit. In reviewing her childbearing history, the client indicates that she has delivered premature twins, one full-term baby, and has had no abortions. Which GTPAL should the nurse document in the client's record? A. 31203 B. 41203 C. 21212 D. 31103

D. 31103. The client has been pregnant 3 times including the current pregnancy (G3); She had one full-term infant (T1); She also had a preterm (P1) twin pregnancy (a multifetal gestation is considered one birth when calculating parity); There were no abortions (A0), so this client has a total of 3 living children.

A 34-week primigravida with pregnancy induced hypertension (PIH) is receiving Ringer's Lactate 500 ml with magnesium sulfate 20 grams at the rate of 3 grams/hour. How many ml/hour should the nurse program the infusion pump? (Enter numeric value only) A. 120 B. 70 C. 65 D. 75

D. 75

The nurse is preparing to give an enema to a laboring client. Which client would require the most caution when carrying out this procedure? A. a gravida 6, para 5 who is 38 years of age an in early labor B. a 37 week primigravida who presents at 100% effacement, 3 cm cervical dilation and a -1 station. C. A gravida 2, para 1 who is at 1 cm cervical dilation and a 0 station admitted for induction of labor due to post dates D. A 40-wk primigravida who is at 6 cm dilation and the presenting part is not engaged

D. A 40-wk primigravida who is at 6 cm dilation and the presenting part is not engaged. When the presenting part is ballottable, it is floating out of the pelvis. In such a situation, the cord can descent before the fetus causing a prolapsed cord, which is an emergency situation.

A 38-week primigravida is admitted to labor and delivery after a non-reactive stress test (NST). The nurse begins a contraction stress test (CST) with an oxytocin (Pitocin) infusion. Which finding is most important for the nurse to report to the healthcare provider? A. Spontaneous rupture of membranes. B. Fetal heart rate accelerations with fetal movement. C. Absences of uterine contraction of 20 minutes. D. A pattern of fetal late decelerations.

D. A pattern of fetal late decelerations.

A 38 week primigravida is admitted to labor and delivery after a non-reactive result on a non-stress test (NST) .The nurse begins contraction stress test (CST) with an oxytocin ( Pitocin ) infusion. Which finding is most important for the nurse to report to the health care provider ? A. Spontaneous rupture of membrane B. Fetal heart rate accelerations with fetal movement C. Absence of uterine contractions within 20 mins D. A pattern of late fetal decelerations

D. A pattern of late fetal decelerations

18. A client in her second trimester of pregnancy asks if it is safe for her to have a drink with dinner. How should the nurse respond to the client? A. Only one drink with the evening meal is not harmful to the fetus. B. Wine can be consumed several times a week after the first trimester. C. During second trimester beer can be consumed without harm to the fetus. D. Abstinence is strongly recommended throughout the pregnancy.

D. Abstinence is strongly recommended throughout the pregnancy.

55. What nursing action should be implemented when intermittently gavage-feeding a preterm infant? A. Insert feeding tube through nares. B. Apply steady pressure to syringe. C. Avoid letting infant suck on tube. D. Allow formula to flow by gravity.

D. Allow formula to flow by gravity.

64. A client is experiencing "back" labor and complains of intense pain in the lower lumbar-sacral area. What action should the nurse implement? A. Assist the client in guided imagery. B. Encourage pant-blow breathing techniques. C. Perform effleurage on the abdomen. D. Apply counter pressure against the sacrum.

D. Apply counter pressure against the sacrum.

25. A multiparous client has been in labor for 8 hours when her membranes ruptured. What action should the nurse implement first? A. Prepare the client for imminent birth. B. Document the characteristics of the fluid. C. Notify the client's primary healthcare provider. D. Assess the fetal heart rate and pattern.

D. Assess the fetal heart rate and pattern.

When assessing a client at 12 weeks of gestation, the nurse recommends that she and her husband consider attending childbirth preparation classes. When is the best time for the couple to attend these classes? A. At 16 weeks of gestation B. At 20 weeks of gestation C. At 24 weeks of gestation D. At 30 weeks of gestation

D. At 30 weeks of gestation

When assessing a client at 12 weeks of gestation, the nurse recommends that she and her husband consider attending childbirth preparation classes. When is the best time for the couple to attend these classes? A. At 16 weeks of gestation B. At 20 weeks of gestation C. At 24 weeks of gestation D. At 30 weeks of gestation

D. At 30 weeks of gestation

7. The nurse is teaching a new mother about diet and breastfeeding. Which instruction is most important to include in the teaching plan? A. Double prenatal milk intake to improve vitamin D transfer to the infant. B. Increase caloric intake by approximately 500 calories/day. C. Avoid spicy foods to prevent infant colic. D. Avoid alcohol because it is excreted in breast milk.

D. Avoid alcohol because it is excreted in breast milk.

A primigravida client with gestational hypertension and a Bishop score of 3 is scheduled for induction of labor. The nurse administers misoprostol at 0700, then observes regular contractions with cervical changes at 0900. Which action should the nurse take? A. Administer misoprostol every 2hrs. B. Ambulate the client after administration of misoprostol. C. Start oxytocin infusion immediately. D. Begin oxytocin 4hrs after misoprostol is given.

D. Begin oxytocin 4hrs after misoprostol is given.

While caring for a laboring client on continuous fetal monitoring, the nurse notes a fetal heart rate pattern that falls and rises abruptly with a "V" shaped appearance. What action should the nurse take first? A. Prepare for a potential cesarean B. Allow the client to begin pushing C. Administer oxygen at 10/L by mask D. Change the maternal position

D. Change the maternal position

A client at 35 weeks gestation complains of a "pain whenever the baby moves". On assessment, the nurse notes the client's temperature to be 101.2F with severe abdominal or uterine tenderness on palpation. The nurse knows that these findings are indicative of which condition? A. Round ligament strain. B. Viral infection C. Abruptio placenta D. Chorioamnionitis

D. Chorioamnionitis

A client at 28 weeks gestation is admitted to the obstetrical unit following her involvement in a motor vehicle collision. While stabilizing the patient , the nurse obtains fetal monitor reading. Which action should the nurse take if the fetus is tachycardic is on the monitor? A. Recount the heart rate manually to confirm a monitor malfunction B. Explain that there is no indication the fetal heart rate is due to trauma C. Evaluate the presence of preterm labor by performing a vaginal D. Contact the healthcare provider after initiating oxygen per face mask

D. Contact the healthcare provider after initiating oxygen per face mask

The nurse is evaluating a full-term multigravida who was induced 3 hours ago. The nurse determines that the client is dilated 7 cm and is 100% effaced at 0 station, with intact membranes. The monitor indicates that the FHR decelerates at the onset of several contractions and returns to baseline before each contraction ends. Which action should the nurse take? A. Reapply the external transducer. B. Insert the intrauterine pressure catheter. C. Discontinue the oxytocin infusion. D. Continue to monitor labor progress.

D. Continue to monitor labor progress.

The nurse is evaluating a full-term multigravida who was induced 3 hours ago. The nurse determines that the client is dilated 7 cm and is 100% effaced at 0 station, with intact membranes. The monitor indicates that the FHR decelerates at the onset of several contractions and returns to baseline before each contraction ends. Which action should the nurse take? A. Reapply the external transducer. B. Insert intrauterine pressure catheter. C. Discontinue the oxytocin infusion. D. Continue to monitor labor progress.

D. Continue to monitor labor progress.

A client receiving oxytocin (Pitocin) to augment early labor. Which assessment is most important for the nurse to obtain each time the infusion rate is increased? A. Pain level B. Blood pressure C. Infusion site D. Contraction pattern

D. Contraction pattern.

An infant is placed in a radiant warmer immediately after birth. At one hour of age, the nurse finds the infant to be jittery, tachypneic, and hypotonic. What is the first action that the nurse should take? A. Notify the health care provider immediately B. Increase the temperature of the radiant warmer C. Assess infant heart rateD. Determine the infants blood sugar level

D. Determine the infant's blood sugar level.

8. Which nursing intervention best enhances maternal-infant bonding during the fourth stage of labor? A. Brighten the lighting so the mother can view the infant. B. Provide positive reinforcement for maternal care of infant. C. Complete a newborn assessment as quickly as possible. D. Encourage early initiation of breast or formula feeding.

D. Encourage early initiation of breast or formula feeding.

The nurse is preparing a young couple and their 24-hour-old infant for discharge from the hospital. In conducting discharge ... A. Ensure that they have the pediatric clinic's phone number. B. Provide the results of the infant's hearing test to the parents. C. Request a return demonstration of a diaper change. D. Evaluate infant feeding technique prior to discharge.

D. Evaluate infant feeding technique prior to discharge.

67. Which behavior should the nurse anticipate for a new mother with an uncomplicated vaginal birth on the third postpartum day? A. Request help with ambulation and perineal care. B. Be very excited and talkative about the birth experience. C. Sleep most of the time when the baby is not present. D. Exhibit interest in learning more about infant care.

D. Exhibit interest in learning more about infant care.

The parents of a newborn tell the nurse that their baby is already trying to walk. How should the nurse respond? A. Encourage the parents to report this to the healthcare provider B. Acknowledge the parents' observation. C. Schedule the newborn for further neurological testing. D. Explain the newborn's normal stepping reflex.

D. Explain the newborn's normal stepping reflex.

A mother who is HIV-positive delivers a full-term newborn and asks the nurse if her baby will become HIV-infected. Which explanation should the nurse provide? A. Most infants of HIV-positive women will continue to test positive for HIV antibodies. B. Infants who have HIV-positive mothers carry the virus and will eventually develop the disease. C. Medication taken during pregnancy to reduce the mother's viral load ensures that the infant is HIV-negative. D. HIV infection is determined at 18 months of age, when maternal HIV antibodies are no longer present.

D. HIV infection is determined at 18 months of age, when maternal HIV antibodies are no longer present.

A mother who is HIV-positive delivers a full-term newborn and asks the nurse if her baby will become HIV-infected. Which explanation should the nurse provide? A. Most infants of HIV-positive women will continue to test positive for HIV antibodies. B. Infants who have HIV-positive mothers carry the virus and will eventually develop the disease. C. Medication taken during pregnancy to reduce the mother's viral load ensures that the infant is HIV-negative. D. HIV infection is determined at 18 months of age, when maternal HIV antibodies are no longer present.

D. HIV infection is determined at 18 months of age, when maternal HIV antibodies are no longer present.

During the transition phase of labor, a client complains of tingling and numbness in her fingers and tells the nurse that she feels like she is going to pass out. What action should the nurse take? A. Encourage her to pant between contractions and blow with contractions. B. Coach her to take a deep cleansing breath and then refocus. C. Instruct her to pant three times and then exhale through pursed lips. D. Have her cup both hands over her nose and mouth while breathing.

D. Have her cup both hands over her nose and mouth while breathing.

Artifical rupture of the membrane of laboring client reveals meconium stained fluid. What is the priority? A. Clean perineal area to prevent infection B. Assess the mothers blood pressure to check for signs of preclampsia C. Assess mothers temperature to check for development of sepsis D. Have meconium aspirator available at delivery

D. Have meconium aspirator available at delivery

The nurse is caring for a newborn who is 18 inches long, weighs 4 pounds, 14 ounces, has a head circumference of 13 inches, and a chest circumference of 10 inches. Based on these physical findings, assessment for which condition has the highest priority? A. Hyperthermia B. Hyperbilirubinemia C. Polycythemia D. Hypoglycemia

D. Hypoglycemia

The nurse is caring for a newborn who is 18 inches long, weighs 4 pounds, 14 ounces, has a head circumference of 13 inches, and a chest circumference of 10 inches. Based on these physical findings, assessments for which condition has the highest priority? A. Hyperbilirubinemia B. Polycythemia C. Hyperthermia D. Hypoglycemia

D. Hypoglycemia

The nurse is caring for an newborn who is 18 inches long, weighs 4 pounds , 14 ounces, has a head circumference of 13 inches and chest circumference is of 10 inches. Based on these physical findings , assessment for which condition has the highest priority? A. Hyperthermia B. Hyperbillirubinemia C. Polycythemia D. Hypoglycemia

D. Hypoglycemia

A 41-week multigravida is receiving oxytocin (Pitocin) to augment labor. Contractions are firm and occurring every 5 minutes, with a 30- to 40-second duration. The fetal heart rate increases with each contraction and returns to baseline after the contraction. Which action should the nurse implement? A. Place a wedge under the client's left side. B. Determine cervical dilation and effacement. C. Administer 10 L of oxygen via facemask. D. Increase the rate of the oxytocin (Pitocin) infusion.

D. Increase the rate of the oxytocin (Pitocin) infusion.

A new mother who is breastfeeding her 4 week old infant has type 1 diabetes , reports that her insulin needs have decreased after the birth of her child. What action should the nurse implement ? A. Schedule an appointment with diabetic nurse educator B. Advise the client to breastfeed more frequently C. Counsel her to increase calorie intake D. Inform her that a decreased need for insulin occurs while breastfeeding

D. Inform her that a decreased need for insulin occurs while breastfeeding

37. A multigravida client at 40+ weeks gestation is induced by using oxytocin (Pitocin). An intrauterine pressure catheter (IUPC) is in place when the client's membranes rupture after 5 hours of active labor. Which finding should require the nurse to implement further action? A. Oxytocin is infusing at a rate of 30 mU/min. B. Labor has progressed at 1 cm/hr dilation. C. Contractions are lasting 60 to 80 seconds. D. Intensity of contractions is 138 mmHg.

D. Intensity of contractions is 138 mmHg.

A woman in her third trimester of pregnancy has been in active labor for the past 8 hours and has dilated 3cm. The nurse's assessment findings and electronic fetal monitoring(EFM) are consistent with hypotonic dystocia, and the healthcare provider prescribed and oxytocin drip. Which data is most important for the nurse to monitor? A. Preparation for emergency cesarean birth. B. Client's hourly blood pressure. C. Checking the perineum for bulging. D. Intensity, interval, and length of contractions.

D. Intensity, interval, and length of contractions.

An expectant father tells the nurse he fears that his wife "is losing her mind." He states she is constantly rubbing her abdomen and talking to the baby, and that she actually reprimands the baby when it moves too much. What recommendation should the nurse make to this expectant father? A. Reassure him that these are normal reactions to pregnancy and suggest that he discuss his concerns with the childbirth education nurse. B. Help him to understand that his wife is experiencing normal symptoms of ambivalence about the pregnancy and no action is needed. C. Ask him to observe his wife's behavior carefully for the next few weeks and report any similar behavior to the nurse at the next prenatal visit. D. Let him know that these behaviors are part of normal maternal/fetal bonding which occur once the mother feels fetal movement.

D. Let him know that these behaviors are part of normal maternal/fetal bonding which occur once the mother feels fetal movement.

The nurse is caring for a female client, a primigravida, with preeclampsia. Findings include +2 proteinuria, BP 172/112 mmHg, facial and hand swelling, complaints of blurry vision and a severe frontal headache. Which medication should the nurse anticipate for this client? A. Clonidine hydrochloride B. Carbamazepine C. Furosemide D. Magnesium sulfate

D. Magnesium sulfate

What is the most important assessment for the nurse to conduct following the administration of epidural anesthesia to a client who is at 40-weeks gestation? A. Level of pain sensation B. Station of presenting part C. Variability of fetal heart rate D. Maternal blood pressure

D. Maternal blood pressure

A client who gave birth to a healthy 8 pound infant 3 hours ago is admitted to the postpartum unit. Which nursing plan is best in assisting this mother to bond with her newborn infant? A. encourage the mother to provide total care for her infant B. provide privacy so the mother can develop a relationship with the infant. C. encourage the father to provide most of the infant's care during hospitalization. D. Meet the mother's physical needs and demonstrate warmth toward the infant.

D. Meet the mother's physical needs and demonstrate warmth toward the infant.

51. The nurse notes an irregular bluish hue on the sacral area of a 1-day old Hispanic infant. How should the nurse document this finding? A. Harlequin sign. B. Acrocyanosis. C. Erythema toxicum. D. Mongolian spots.

D. Mongolian spots.

63. A client in labor receives an epidural block. What intervention should the nurse implement first? A. Assess contractions. B. Encourage oral fluids. C. Obtain a radial pulse. D. Monitor blood pressure.

D. Monitor blood pressure.

The nurse observes that an antepartum client who is on bed rest for preterm labor is eating ice rather than the food on her breakfast tray. The client states that she has a craving for ice and then feels too full to eat anything else. Which is the best response by the nurse? A. Remove all ice from the client's room. B. Ask the client what foods she might consider eating. C. Remind the client that what she eats affects her baby. D. Notify the health care provider.

D. Notify the health care provider.

The nurse observes that an antepartum client who is on bed rest for preterm labor is eating ice rather than the food on her breakfast tray. The client states that she has a craving for ice and then feels too full to eat anything else. Which is the best response by the nurse? A. Remove all ice from the client's room. B. Ask the client what foods she might consider eating. C. Remind the client that what she eats affects her baby. D. Notify the health care provider.

D. Notify the health care provider.

59. A client who is stable has family members present when the nurse enters the birthing suite to access the mother and newborn. What action should the nurse implement at this time? A. Do a brief assessment for only the infant while the family members are present. B. Reschedule the visit so that the mother and infant can be assessed privately. C. Ask to meet with the client and infant without family members present. D. Observe interactions of family members with the newborn and each other.

D. Observe interactions of family members with the newborn and each other.

During a routine prenatal vital a client 32 weeks gestation complains of urinary frequency has increased during the day as well at night. The nurse determines the client is having irregular uterine contractions. What should the nurse implement ? A. Ask the client if she had sexual intercourse yesterday B. Determine if she has change in vaginal discharge C. Collect urine sample from dipstick analysis D. Obtain a midstream urine specimen for culture

D. Obtain a midstream urine specimen for culture

One hour following a normal vaginal delivery, a newborn infant boy's axillary temperature is 96° F, his lower lip is shaking and, when the nurse assesses for a Moro reflex, the boy's hands shake. Which intervention should the nurse implement first? A. Stimulate the infant to cry. B. Wrap the infant in warm blankets. C. Feed the infant formula. D. Obtain a serum glucose level.

D. Obtain a serum glucose level.

One hour following a normal vaginal delivery, a newborn infant boy's axillary temperature is 96° F, his lower lip is shaking, and when the nurse assesses for a Moro reflex, the boy's hands shake. Which intervention should the nurse implement first? A. Stimulate the infant to cry. B. Wrap the infant in warm blankets. C. Feed the infant formula. D. Obtain a serum glucose level.

D. Obtain a serum glucose level.

The nurse is caring for a client whos is 10 weeks gestation and palpates the fundus at 2 fingerbreadths above the pubic symphysis. The client reports nausea, vomiting, and scant dark brown vaginal discharge. Which action should the nurse take? A. Measure vital signs. B. Recommend bed rest. C. Collect urine sample urinalysis. D. Obtain human chronic gonadotropin levels.

D. Obtain human chronic gonadotropin levels.

23. A client in active labor at 39-weeks gestation tells the nurse she feels a wet sensation on the perineum. The nurse notices pale, straw-colored fluid with small white particles. After reviewing the fetal monitor strip for fetal distress, what action should the nurse implement? A. Clean the perineal area. B. Offer the client a bed pan. C. Escort the client to the bathroom. D. Perform a nitrazine test.

D. Perform a nitrazine test.

15. Which gastrointestinal findings should the nurse be concerned about any client at 28-weeks gestation? A. Decrease peristalsis. B. Ptyalism. C. Pyrosis. D. Pica.

D. Pica.

At 39-weeks gestation, a multigravida is having a non-stress test (NST). The fetal heart rate (FHR) has remained nonreactive during the 30 minutes of evaluation. Based on this finding, which action should the nurse implement? A. Initiate an intravenous infusion B. Observe the FHR pattern for 30 more minutes C. Schedule a biophysical profile D. Place an acoustic stimulator on the abdomen

D. Place an acoustic stimulator on the abdomen

29. What action should the nurse implement when caring for a newborn receiving phototherapy? A. Reposition every 6 hours. B. Apply an oil-based lotion to the skin. C. Limit the intake of formula. D. Place an eyeshield over the eyes.

D. Place an eyeshield over the eyes.

Upon admission to the nursery, the nurse places a newborn supine under a radiant warmer, an external heat source. What intervention should the nurse implement to ensure safe thermoregulation? A. Wrap the infant in two blankets and place the radiant warmer on low. B. Dry the newborn's scalp and place a stockinet cap on the head. C. Move temperature probe over the ribs when turning to a lateral position. D. Place temperature probe on the abdomen in line with the radiant heat source.

D. Place temperature probe on the abdomen in line with the radiant heat source.

38. A primigravida at 37-weeks gestation tells the nurse that her "bag-of-water" has broken. While inspecting the client's perineum, the nurse notes that the umbilical cord protruding from the vagina. What action should the nurse implement first? A. Give the healthcare provider a status report. B. Administer 10 L of oxygen via face mask. C. Wrap the cord with gauze soaked in saline. D. Place the client in the knee-chest position.

D. Place the client in the knee-chest position.

The nurse is examining an infant for possible cryptorchidism. Which exam technique should be used? A. Place the infant in side-lying to facilitate the exam B. Hold the penis and retract the foreskin gently C. Cleanse the penis with an antiseptic-soaked pad D. Place the infant in warm room and use a calm approach

D. Place the infant in warm room and use a calm approach

A multiparous client with active herpes lesion is admitted to the unit with spontaneous rupture of membranes. Which action should the nurse do first? A. Obtain blood cultures. B. Cover the lesion with a dressing. C. Administer penicillin. D. Prepare for a cesarean section.

D. Prepare for a cesarean section.

Just after delivery, a new mother tells the nurse, "I was unsuccessful breastfeeding my first child, but I would like to try with this baby." Which intervention should the nurse implement first? A. Assess the husband's feelings about his wife's decision to breastfeed their baby. B. Ask the woman to describe why she was unsuccessful with breastfeeding her last child. C. Encourage the woman to develop a positive attitude about breastfeeding to help ensure success. D. Provide assistance to the mother to begin breastfeeding as soon as possible after delivery.

D. Provide assistance to the mother to begin breastfeeding as soon as possible after delivery.

Just after delivery, a new mother tells the nurse, "I was unsuccessful breastfeeding my first child, but I would like to try with this baby." Which intervention is best for the nurse to implement first? A. Assess the husband's feelings about his wife's decision to breastfeed their baby B. Ask the client to describe why she was unsuccessful with breastfeeding her last child C. Encourage the client to develop a positive attitude about breastfeeding to help ensure success D. Provide assistance to the mother to begin breastfeeding as soon as possible after delivery.

D. Provide assistance to the mother to begin breastfeeding as soon as possible after delivery. Infants respond to breastfeeding best when feeding is initiated in the active phase soon after delivery

An off-duty nurse finds a woman in a supermarket parking lot delivering an infant while her husband is screaming for someone to help his wife. Which intervention has the highest priority? A. Use thread to tie off the umbilical cord. B. Provide privacy for the woman. C. Reassure the husband and keep him calm. D. Put the newborn to the breast immediately.

D. Put the newborn to the breast immediately.

An expectant father tells the nurse he fears that his wife "is losing her mind." He states that she is constantly rubbing her abdomen and talking to the baby and that she actually reprimands the baby when it moves too much. Which recommendation should the nurse make to this expectant father? A. Suggest that his wife seek professional counseling to deal with her symptoms. B. Explain that his wife is exhibiting ambivalence about the pregnancy. C. Ask him to report similar abnormal behaviors at the next prenatal visit. D. Reassure him that normal maternal-fetal bonding is occurring.

D. Reassure him that normal maternal-fetal bonding is occurring.

An expectant father tells the nurse he fears that his wife "is losing her mind." He states that she is constantly rubbing her abdomen and talking to the baby and that she actually reprimands the baby when it moves too much. Which recommendation should the nurse make to this expectant father? A. Suggest that his wife seek professional counseling to deal with her symptoms. B. Explain that his wife is exhibiting ambivalence about the pregnancy. C. Ask him to report similar abnormal behaviors at the next prenatal visit. D. Reassure him that normal maternal-fetal bonding is occurring.

D. Reassure him that normal maternal-fetal bonding is occurring.

Albumin 25% IV is prescribed for a child with nephrotic syndrome. Which assessment finding indicates to the nurse that the medication is having the desired effect? A. Weight gain B. Reduction of fever C. Improved caloric intake D. Reduction of edema

D. Reduction of edema

54. The nurse assesses a high-risk neonate under a radiant warmer who has an umbilical catheter and identifies that the neonate's feet are blanched. What action should be implemented? A. Elevate feet 15°. B. Place socks on infant. C. Wrap feet loosely in prewarmed blanket. D. Report findings to the healthcare provider.

D. Report findings to the healthcare provider.

A primipara client at 42 weeks gestation is admitted for induction. Within one hour after initiating an oxytocin infusion, her cervix is 100% effaced and 6 cm dilated, contractions are occurring every 1 minute with a 75 second duration. The nurse stops the oxytocin and starts oxygen. After 30 minutes of uterine rest, the contractions are occurring every 5 minutes with 20 second duration. What intervention should the nurse implement? A. Notify nursery about the client's response. B. Check for clonus in both feet. C. Stop oxygen per cannula. D. Restart oxytocin infusion rate per protocol.

D. Restart oxytocin infusion rate per protocol.

A postpartum client who is Rh-negative refuses to receive Rho (D) immune globulin (RhoGam) after delivery of an infant who is Rh-positive. Which information should the nure provide this client? A. RhoGam is not necessary unless all her pregnancies are Rh-positive B. The R-positive factor from the fetus threatens her blood cells C. The mother should receive RhoGam when the baby is Rh-negative D. RhoGam prevents maternal antibody formation for future Rh-positive babies

D. RhoGam prevents maternal antibody formation for future Rh-positive babies

The nurse is calculating the estimated date of confinement (EDC) using Ngele's rule for a client whose last menstrual period started on December 1. Which date is most accurate? A. August 1 B. August 10 C. September 3 D. September 8

D. September 8

After each feeding, a 3-day old newborn is spitting up large amounts of Enfamil Newborn Formula, a nonfat cow's milk formula. The pediatric healthcare provider changes the neonate's formula to Similac Soy Isomil Formula, a soy protein isolate based on infant formula. What information should the nurse provide to the mother about the newly prescribed formula? A. The new formula is a coconut milk formula used with babies with impaired fat absorption B. Enfamil Formula is a demineralized whey formula that is needed with diarrhea C. the new formula is a casein protein source that is low in pheynylalanine. D. Similac Soy Isomil Formula is a soy-based formula that contains sucrose.

D. Similac Soy Isomil Formula is a soy-based formula that contains sucrose. The nurse should explain that the newborn's feeding intolerance may be related to the lactose found in cow's milk formula and is being replaced with the soy-based formula that contains sucrose which is well-tolerated in infants with milk allergies and lactose intolerances

The nurse is reviewing the serum laboratory finding for a 5-day-old infant with congenital adrenal hyperplasia. Which laboratory results should be reported to the healthcare provider immediatly? A. Bilirubin of 1.5 mg/dl B. Glucose of 80 mg/dl C. Potassium of 4.5 mEq/L D. Sodium of 119 mEq/L

D. Sodium of 119 mEq/L

A breastfeeding infant, screened for congenital hypothyroidism, is found to have low levels of thyroxine (t4) and high levels of thyroid stimulating hormone (TSH)/ What is the best explanation for this finding? A. The thyroxine level is low because the TSH level is high. B. High thyroxine levels normally occur in breastfeeding infants. C. The thyroid gland does not produce normal levels of thyroxine for several weeks after birth D. The TSH is high because of the low production of T4 by the thyroid.

D. The TSH is high because of the low production of T4 by the thyroid.

In developing a teaching plan for expectant parents, the nurse decides to include information about when the parents can expect the infant's fontanels to close. Which statement is accurate regarding the timing of closure of an infant's fontanels that should be included in this teaching plan? A. The anterior fontanel closes at 2 to 4 months and the posterior fontanel by the end of the first week. B. The anterior fontanel closes at 5 to 7 months and the posterior fontanel by the end of the second week. C. The anterior fontanel closes at 8 to 11 months and the posterior fontanel by the end of the first month. D. The anterior fontanel closes at 12 to 18 months and the posterior fontanel by the end of the second month.

D. The anterior fontanel closes at 12 to 18 months and the posterior fontanel by the end of the second month.

In developing a teaching plan for expectant parents, the nurse decides to include information about when the parents can expect the infant's fontanels to close. Which statement is accurate regarding the timing of closure of an infant's fontanels that should be included in this teaching plan? A. The anterior fontanel closes at 2 to 4 months and the posterior fontanel by the end of the first week. B. The anterior fontanel closes at 5 to 7 months and the posterior fontanel by the end of the second week. C. The anterior fontanel closes at 8 to 11 months and the posterior fontanel by the end of the first month. D. The anterior fontanel closes at 12 to 18 months and the posterior fontanel by the end of the second month.

D. The anterior fontanel closes at 12 to 18 months and the posterior fontanel by the end of the second month.

The nurse is conducting a home health visit of a client who delivered 3 weeks ago and is formula feeding the infant. Which observations should the nurse find most concerning? A. The client notes infant feeds every 2-3 hours and voids 5-6 times per day. B. The client is in pajama's and infant is freshly bathed. C. Used bottles are in the kitchen and infant is in a swing. D. The clients eyes are red from crying and infant is fussing in the crib.

D. The clients eyes are red from crying and infant is fussing in the crib.

2. A client states, "During the three months I've been pregnant, it seems like I have had to go to the bathroom every five minutes." Which explanation should the nurse provide to this client? A. The client may have a bladder or kidney infection. B. Bladder capacity increases during pregnancy. C. During pregnancy a woman is especially sensitive to body functions. D. The growing uterus is putting pressure on the bladder.

D. The growing uterus is putting pressure on the bladder.

19. A female client who wants to delivery at home asks the nurse to explain the role of a nurse-midwife in providing obstetric care. What information should the nurse provide? A. Natural childbirth without analgesia is used to manage pain during labor. B. And obstetrician should also follow the client during pregnancy. C. Birth in the home setting is the preference for using a midwife for delivery. D. The pregnancy should progress normally and be considered low risk.

D. The pregnancy should progress normally and be considered low risk.

A couple has been trying to conceive for 9 months without success. Which information obtained from the clients is most likely to have an impact on the couple's ability to conceive a child? A. Exercise regimen of both partners includes running 4 miles each morning B. history of having sexual intercourse 2-3x/wk. C. The woman's menstrual period occurs every 35 days D. They use lubricants with each sexual encounter to decrease friction

D. They use lubricants with each sexual encounter to decrease friction. The use of lubricants has the potential to affect fertility because some lubricants interfere with sperm motility.

Vaginal prostiglandin gel is used to induce labor women who are 42 weeks of gestation. Thirty minutes after insertion of the gel , the client complains of vaginal warmth, and is experiencing 90 second contractions with fetal heart deceleration. What action should the nurse implement first A. Assess maternal vital signs B. Notify the healthcare provider C. Increase the IV infusion rate D. Turn to a side lying position

D. Turn to a side lying position

48. A client at 29-weeks gestation with possible placental insufficiency is being prepared for prenatal testing. Information about which diagnostic study should the nurse provide information to the client? A. Maternal serum alpha-fetoprotein. B. Amniocentesis. C. Chorionic villus sampling. D. Ultrasonography.

D. Ultrasonography.

A client who had her first baby 3 months ago & is breastfeeding her infant tells the nurse that she is currently using the same diaphragm that she used before becoming pregnant. Which information should the nurse provide this client? A. After ceasing breastfeeding, the diaphragm should be resized. B. Avoid intercourse during ovulation until the size of the diaphragm has been evaluated. C. If no more than 20 pounds was gained during pregnancy, the diaphragm is safe to use. D. Use an alternate form of contraceptive until a new diaphragm is obtained.

D. Use an alternate form of contraceptive until a new diaphragm is obtained.

A primigravida client who is 5 cm dilated, 90% effaced, and at 0 station is requesting an epidural for pain relief. Which assessment finding is most important for the nurse to report to the healthcare provider? A. cervical dilation of 5 cm with 90% effacement B. WBC of 12,000/mm3 C. hemoglobin of 12 mg/dl and hematocrit of 38%. D. a platelet count of 67,000/mm3

D. a platelet count of 67,000/mm3. This low amount places the client at risk for bleeding from an epidural

In developing a teaching plan for expectant parents, the nurse plans to including information about when the parents can expect the infant's fontanels to close. The nurse bases the explanation on knowledge that for the normal newborn, the A. anterior fontanel closes at 2-4 months and the posterior by the end of the first week B. anterior fontanel closes at 5-7 months and the posterior by the end of the second week C. anterior fontanel closes at 8-11 months and the posterior by the end of the first month D. anterior fontanel closes at 12-18 months and the posterior by the end of the second month.

D. anterior fontanel closes at 12-18 months and the posterior by the end of the second month.

A 40 week gestation primigravada client is being induced with an oxytocin (pitocin) secondary infusion and complains pain in her lower back. Which intervention should the nurse implement? A. discontinue the oxytocin (Pitocin) infusion B. place the client in a semi-Fowler's position C. inform the healthcare provider D. apply firm pressure on the sacral area.

D. apply firm pressure on the sacral area

The nurse is assessing a client who is having a non-stress test (NST) at 41 weeks gestation. The nurse determines that the client is not having contractions, the fetal heart rate (FHR) baseline is 144 bpm, and no FHR accelerations are occuring. What action should the nurse take? A. check the client for urinary bladder distension B. notify the healthcare provider of the nonreactive results C. have the mother stimulate the fetus to move D. ask the client if she has felt any fetal movement

D. ask the client if she has felt any fetal movement. An NST is used to determine fetal well-being and is often implemented when postmaturity is suspected. A 'reactive' NST occurs if the FHR accelerates 15 bpm for 15 seconds in response to the fetus' own movement, and is "nonreacctive" if no FHR acceleration occurs in response to fetal movement. The client should empty her bladder before starting the test, but bladder distention does not impede fetal movement. The client should be quizzed about fetal movement before determining that the NST is nonreactive. If no movement has occurred in the last 20-30 minutes, it is likely that the fetus is sleeping. Providing the mother with orange juice often wakes the infant, and then the NST should be conducted again.

When assessing a client who is at 12-weeks gestation, the nurse recommends that she and her husband consider attending childbirth preparation classes. When is the best time for the couple to attend these classes? A. at 16 weeks gestation B. at 20 weeks gestation C. at 24 weeks gestation D. at 30 weeks gestation

D. at 30 weeks gestation

One hour after giving birth to an 8 pound infant, a client's lochia rubra has increased from small to large and her fundus is boggy despite massage. The client's pulse is 84 bpm and blood pressure is 156/96. The healthcare provider prescribes Methergine 0.2 mg IM x 1. What action should the nurse take immediately? A. give the medication as prescribed and monitor for efficacy B. encourage the client to breast feed rather than bottle feed C. have the client empty her bladder and then massage the fundus D. call the healthcare provider to question the prescription

D. call the healthcare provider to question the prescription. Methergine is contraindicated for clients with elevated blood pressure, so the nurse should contact the healthcare provider and question the prescription.

Which assessment finding should the nursery nurse report to the pediatric healthcare provider? A. blood glucose level of 45 mg/dl B. blood pressure of 82/45 C. non-bulging anterior fontanel D. central cyanosis when crying

D. central cyanosis when crying. An infant who demonstrates central cyanosis when crying is manifesting poor adaption to extrauterine life which should be reported to the healthcare provider for determination of a possible underlying cardiovascular problem.

What action should the nurse implement to decrease the client's risk for hemorrhage after a cesarean section? A. monitor urinary output via an indwelling catheter B. assess the abdominal dressings for drainage C. give the Ringer's lactated infusion at 125 ml/hr. D. check the firmness of the uterus every 15 minutes

D. check the firmness of the uterus every 15 minutes. A client's risk for postpartal hemorrhage is decreased when the uterus is firm after delivery of the infant. Assessment of fundus consistency q15 min provides frequent intervals to stimulate the fundus to contract and prevent bleeding.

The nurse is planning preconception care for a new female client. Which information should the nurse provide to the client? A. discuss various contraceptive methods to use until pregnancy is desired B. provide written or verbal information about prenatal care C. ask the client about risk factors associated with complications of pregnancy D. encourage healthy lifestyles for families desiring pregancy

D. encourage healthy lifestyles for families desiring pregancy

An off-duty nurse finds a woman in a supermarket parking lot delivery an infant while her husband is screaming for someone to help his wife. Which intervention has the highest priority? A. use a thread to tie off the umbilical cord. B. provide as much privacy as possible for the woman. C. reassure the husband and try to keep him calm D. put the newborn to breast.

D. put the newborn to breast. Putting the newborn to breast will help contract the uterus and prevent a postpartum hemorrhage - this intervention has the highest priority.

A new mother calls the nurse stating that she wants to start feeding her 6-month-old child something besides breast milk, but is concerned that the infant is too young to start eating solid foods. How should the nurse respond? A. encourage the mother to schedule a developmental assessment of the infant B. advise the mother to wait at least another month before starting any solid foods C. instruct the mother to offer a few spoons of 2-3 pureed fruit at each meal D. reassure the mother that the infant is old enough to eat iron-fortified cereal

D. reassure the mother that the infant is old enough to eat iron-fortified cereal

A 6-month old child who had a cleft-lip repair has elbow restraints in place. What nursing intervention should the nurse plan to implement? A. remove restraints q4h for 30 minutes and place gloves on the child's hands B. record observations of the restraints q2h and ensure that they are in place at all times C. obtain the HCP advice as to when the restraints should be removed D. remove restraints one at a time to provide ROM exercises

D. remove restraints one at a time to provide ROM exercises

While breastfeeding, a new mother strokes the top of her baby's head and asks the nurse about the baby's swollen scalp. The nurse responds that the swelling is caput succadeaneum. Which additional information should the nurse provide this new mother? A. the infant should be positioned to reduce the swelling B. the swelling is a subperiosteal collection of blood C. the pediatrician will aspirate the blood if it gets larger D. the scalp edema will subside in a few days after birth.

D. the scalp edema will subside in a few days after birth. Caput succadeaneum is edema of the fetal scalp that crosses over the suture lines and is caused by pressure on the fetal head against the cervix during labor. It will subside in a few days after birth without treatment.

An off-duty nurse finds a woman in a supermarket parking lot delivering an infant while her husband is screaming for someone to help his wife. Which intervention has the highest priority? A.Use thread to tie off the umbilical cord. B.Provide privacy for the woman. C.Reassure the husband and keep him calm. D.Put the newborn to the breast immediately.

D.Put the newborn to the breast immediately. Rationale: Putting the newborn to the breast will help contract the uterus and prevent a postpartum hemorrhage. This intervention has the highest priority. Option A is not necessary; the infant can be transported attached to the placenta. Option B is an important psychosocial need but does not have the priority of option D. Although the husband is an important part of family-centered care, he is not the most important concern at this time.

When assessing a client at 12 weeks of gestation, the nurse recommends that she and her husband consider attending childbirth preparation classes. When is the best time for the couple to attend these classes? A.At 16 weeks of gestation B.At 20 weeks of gestation C.At 24 weeks of gestation D.At 30 weeks of gestation

D.At 30 weeks of gestation Rationale: Learning is facilitated by an interested pupil. The couple is most interested in childbirth toward the end of the pregnancy, when they are beginning to anticipate the onset of labor and the birth of their child. Option D is closest to the time when parents would be ready for such classes. Options A, B, and C are not the best times during a pregnancy for the couple to attend childbirth education classes. At these times they will have other teaching needs. Early pregnancy classes often include topics such as nutrition, physiologic changes, coping with normal discomforts of pregnancy, fetal development, maternal and fetal risk factors, and evolving roles of the mother and her significant others.

The nurse is evaluating a full-term multigravida who was induced 3 hours ago. The nurse determines that the client is dilated 7 cm and is 100% effaced at 0 station, with intact membranes. The monitor indicates that the FHR decelerates at the onset of several contractions and returns to baseline before each contraction ends. Which action should the nurse take? A.Reapply the external transducer. B.Insert the intrauterine pressure catheter. C.Discontinue the oxytocin infusion. D.Continue to monitor labor progress

D.Continue to monitor labor progress Rationale: The fetal heart rate indicates early decelerations, which are not an ominous sign, so the nurse should continue to monitor the labor progress and document the findings in the client's record. There is no reason to reapply the external transducer if the FHR tracings are being captured. Options B and C are not indicated at this time.

A mother who is HIV-positive delivers a full-term newborn and asks the nurse if her baby will become HIV-infected. Which explanation should the nurse provide? A.Most infants of HIV-positive women will continue to test positive for HIV antibodies. B.Infants who have HIV-positive mothers carry the virus and will eventually develop the disease. C.Medication taken during pregnancy to reduce the mother's viral load ensures that the infant is HIV-negative. D.HIV infection is determined at 18 months of age, when maternal HIV antibodies are no longer present.

D.HIV infection is determined at 18 months of age, when maternal HIV antibodies are no longer present. Rationale: All newborns of HIV-positive mothers receive passive HIV antibodies from the mother, so the evaluation of an infant for the HIV virus is determined at 18 months of age, when all the maternal antibodies are no longer in the infant's blood. Passive HIV antibodies disappear in the infant within 18 months of age. Option B is inaccurate. Although administration of HIV medication during pregnancy can significantly reduce the risk of vertical transmission, treatment does not ensure that the virus will not become manifest in the infant.

During the transition phase of labor, a client complains of tingling and numbness in her fingers and tells the nurse that she feels like she is going to pass out. What action should the nurse take? A.Encourage her to pant between contractions and blow with contractions. B.Coach her to take a deep cleansing breath and then refocus. C.Instruct her to pant three times and then exhale through pursed lips. D.Have her cup both hands over her nose and mouth while breathing.

D.Have her cup both hands over her nose and mouth while breathing. Rationale: Hyperventilation blows off carbon dioxide, depletes carbonic acid in the blood, and causes transient respiratory alkalosis, so the client should cup both her hands over her mouth and nose so that she can rebreathe carbon dioxide. Options A, B, and C do not help restore carbon dioxide levels as effectively as rebreathing air in the cupped hands or from a paper bag.

The nurse observes that an antepartum client who is on bed rest for preterm labor is eating ice rather than the food on her breakfast tray. The client states that she has a craving for ice and then feels too full to eat anything else. Which is the best response by the nurse? A.Remove all ice from the client's room. B.Ask the client what foods she might consider eating. C.Remind the client that what she eats affects her baby. D.Notify the health care provider.

D.Notify the health care provider. Rationale: The health care provider should be notified when a client practices pica (craving for and consumption of nonfood substances). The practice of pica may displace more nutritious foods from the diet, and the client should be evaluated for anemia. Option A is overreacting and may be perceived as punishment by the client. Option B allows the dietary department to customize the client's tray but fails to address physiologic problems associated with not consuming nutritious foods in pregnancy. Option C is judgmental and blocks further communication.

One hour following a normal vaginal delivery, a newborn infant boy's axillary temperature is 96° F, his lower lip is shaking, and when the nurse assesses for a Moro reflex, the boy's hands shake. Which intervention should the nurse implement first? A.Stimulate the infant to cry. B.Wrap the infant in warm blankets. C.Feed the infant formula. D.Obtain a serum glucose level.

D.Obtain a serum glucose level. Rationale: This infant is demonstrating signs of hypoglycemia, possibly secondary to a low body temperature. The nurse should first determine the serum glucose level. Option A is an intervention for a lethargic infant. Option B should be done based on the temperature, but first the glucose level should be obtained. Option C helps raise the blood sugar, but first the nurse should determine the glucose level.

Just after delivery, a new mother tells the nurse, "I was unsuccessful breastfeeding my first child, but I would like to try with this baby." Which intervention should the nurse implement first? A.Assess the husband's feelings about his wife's decision to breastfeed their baby. B.Ask the woman to describe why she was unsuccessful with breastfeeding her last child. C.Encourage the woman to develop a positive attitude about breastfeeding to help ensure success. D.Provide assistance to the mother to begin breastfeeding as soon as possible after delivery.

D.Provide assistance to the mother to begin breastfeeding as soon as possible after delivery. Rationale: Infants respond to breastfeeding best when feeding is initiated in the active phase soon after delivery. Options A and B might provide interesting data, but gathering this information is not as important as providing support and instructions to the new mother. Although option C is also true, this response by the nurse might seem judgmental to a new mother.

What complication should a nurse be alert for in a client receiving an oxytocin infusion to induce labor?

uterine tetany

A client with severe preeclampsia develops eclampsia. After the seizure, the client has a temperature of 102° F (38.9° C). What does the nurse suspect as the cause of the elevated temperature?

Irregularity in the cerebral thermal center

The nurse is caring for a woman with a previously diagnosed heart disease who is in the second stage of labor. Which assessment findings are of greatest concern? a. edema, basilar rales, and an irregular pulse b. Increased urinary output, and tachycardia c. Shortness of breath, bradycardia, and hypertension d. Regular heart rate, and hypertension

a. Edema, basilar rales, and an irregular pulse Edema, basilar rales, and an irregular pulse (A) indicate cardiac decompensation and require immediate intervention.

Which nursing intervention is most helpful in relieving postpartum uterine contractions or "afterpains?" a. Lying prone with a pillow on the abdomen b. Using a breast pump c. Massaging the abdomen d. Giving oxytocic medications

a. Lying prone with a pillow on the abdomen Lying prone (A) keeps the fundus contracted and is especially useful with multiparas, who commonly experience afterpains due to lack of uterine tone.

When explaining "postpartum blues" to a client who is 1 day postpartum, which symptoms should the nurse include in the teaching plan? (Select all that apply.) a. Mood swings b. Panic attacks c. Tearfulness d. Decreased need for sleep e. Disinterest in the infant

a. Mood swings c. Tearfulness "Postpartum blues" is a common emotional response related to the rapid decrease in placental hormones after delivery and include mood swings (A), tearfulness (C), feeling low, emotional, and fatigued.

A client receiving epidural anesthesia begins to experience nausea and becomes pale and clammy. What intervention should the nurse implement first? a. Raise the foot of the bed b. Assess for vaginal bleeding c. Evaluate the fetal heart rate d. Take the client's blood pressure

a. Raise the foot of the bed These symptoms are suggestive of hypotension which is a side effect of epidural anesthesia. Raising the foot of the bed (A) will increase venous return and provide blood to the vital areas. Increasing the IV fluid rate using a balanced non-dextrose solution and ensuring that the silent is in a lateral position are also appropriate interventions.

The nurse is providing discharge teaching for a client who is 24 hours postpartum. The nurse explains to the client that her vaginal discharge will change from red to pink and then to white. The client asks, "What if I start having red bleeding after it changes?" What should the nurse instruct the client to do? a. Reduce activity level and notify the healthcare provider b. Go to bed and assume a knee-chest position c. Massage the uterus and go to the emergency room d. Do not worry as this is a normal occurrence

a. Reduce activity level and notify the healthcare provider Lochia should progress in stages from rubra (red) to serosa (pinkish) to alba (whitish), and not return to red. The return to rubra usually indicates subinvolution of infection.

A 42-week gestational client is receiving an intravenous infusion of oxytocin (Pitocin) to augment early labor. The nurse should discontinue the oxytocin infusion for which pattern of contractions? a. Transition labor with contractions every 2 minutes, lasting 90 seconds each a. Early labor with contractions every 5 minutes, lasting 40 seconds each c. Active labor with contractions every 31 minutes, lasting 60 seconds each d. Active labor with contractions every 2 to 3 minutes, lasting 70 to 80 seconds each

a. Transition labor with contractions every 2 minutes, lasting 90 seconds each Contractions pattern (A) describes hyperstimulation and an inadequate resting time between contractions to allow for placental perfusion. The oxytocin infusion should be discontinued.

Client teaching is an important part of the maternity nurse's role. Which factor has the greatest influence on successful teaching on the gravid client? a. The client's readiness to learn b. The client's educational background c. The order in which the information is presented d. The extent to which the pregnancy was planned

a. the client's readiness to learn When teaching any client, readiness to learn (A) is the most important criterion. For example, the client with severe morning sickness in the first trimester may not be "ready to learn" about ways to relieve morning sickness.

Which maternal behavior is the nurse most likely to see when a new mother receives her infant for the first time? a. She eagerly reaches for the infant, undresses the infants, and examines the infant completely b. Her arms and hands receive the infant and she then traces the infant's profile with her fingertips c. Her arms and hands receive the infant and she then cuddles the infant to her own body d. She eagerly reaches for the infant and then holds the infant close to her own body

b. Her arms and hands receive the infant and she then traces the infant's profile with her fingertips Attachment/bonding theory indicates that most mothers will demonstrate behaviors described in (B) during the first visit with the newborn, which may be at delivery of later.

A client who is attending antepartum classes asks the nurse why her healthcare provider has prescribed iron tablets. The nurse's response is based on what knowledge? a. Supplementary iron is more efficiently utilized during pregnancy b. It it difficult to consume 18 mg of additional iron by diet alone c. Iron absorption is decreased in the GI tract during pregnancy d. Iron is needed to prevent megaloblastic anemia in the last trimester

b. It is difficult to consume 18 mg of additional iron by diet alone Consuming enough iron-containing foods to facilitate adequate fetal storage of iron and to meet the demands of pregnancy is difficult (B) so iron supplements are often recommended.

The nurse identifies crepitus when examining the chest of a newborn who was delivered vaginally. Which further assessment should the nurse perform? a. Elicit positive scarf sign on the affected side b. Observe for an asymmetrical Moro (startle) reflex c. Watch for swelling of fingers on the affected side d. Note paralysis of affected extremity and muscles

b. Observe for an asymmetrical Moro (startle) reflex The most common neonatal birth trauma due to vaginal delivery is fracture of the clavicle. Although an infant may be asymptomatic, a fracture clavicle should be suspected is an infant has limited use of the affected arm malposition of the arm, an asymmetric Moro reflex (B), crepitus over the clavicle, focal swelling or tenderness, or cries when the arm is moved.

A client at 31 weeks' gestation is admitted in preterm labor. What class of drugs might the nurse anticipate being prescribed?

beta-adrenergic

Oral contraceptives are prescribed for a client who smokes heavily. What side effect should the nurse warn the client might occur?

blood clots

A mother who is breastfeeding her baby receives instructions from the nurse. Which instruction is most effective to prevent nipple soreness? a. Wear a cotton bra b. Increase nursing time gradually c. Correctly place the infant on the breast d. Manually express a small amount of milk before nursing

c. Correctly place the infant on the breast The most common cause of nipple soreness is incorrect positioning (C) of the infant on the breast, e.g., grasping too little of the areola or grasping on the nipple.

The total bilirubin level of a 36-hour, breastfeeding newborns is 14 mg/dl. Based on this finding, which intervention should the nurse implement? a. Provide phototherapy for 30 minutes q8h b. Feed the newborn sterile water hourly c. Encourage the mother to breastfeed frequently d. Assess the newborn's blood glucose level

c. Encourage the mother to breastfeed frequently The normal total bilirubin level is 6 to 12 mg/dl after Day 1 of life. This infant's bilirubin is beginning to climb and the infant should be monitored to prevent further complications. Breast milk provides calories and enhances GI motility, which will assist the bowel in eliminating bilirubin (C).

The nurse is teaching care of the newborn to a group of prospective parents and describes the need for administering antibiotic ointment into the eyes of the newborn. Which infectious organism will this treatment prevent from harming the infant? a. Herpes b. Staphylococcus c. Gonorrhea d. Syphilis

c. Gonorrhea Erythromycin ointment is instilled into the lower conjunctive of each eye within 2 hours after birth to prevent ophthalmica neonatorum, an infection caused by gonorrhea, and inclusion conjunctivitis, an infection caused by chlamydia (C). The infant may be exposed to these bacteria when passing the birth canal.

The nurse is counseling a woman who wants to become pregnant. The woman tells the nurse that she has a 36-day menstrual cycle and the first day of her menstrual period was January *. The nurse correctly calculates that the woman's next fertile period is a. January 14-15 b. January 22-23 c. January 30-31 d. February 6-7

c. January 30-31 This woman can expect her next period to begin 36 days from the first day of her last menstrual period - the cycle begins at the first day of the cycle and continues to the first day of the next cycle. Her next period would, therefore, begin on February 13. Ovulation occurs 14 days before the first day of the menstrual period. Therefore, ovulation for this woman would occur January 31 (C).

The nurse assesses a client admitted to the labor and delivery unit and obtains the following data: dark red vaginal bleeding, uterus slightly tense between contractions, BP 110/68, FHR 110 beats/minute, cervix 1 cm dilated and uneffaced. Based on these assessment findings, what intervention should the nurse implement? a. Insert an internal fetal monitor b. Assess for cervical changes q1h c. Monitor bleeding from IV sites d. Perform Leopold's maneuvers

c. Monitor bleeding from IV sites Monitoring bleeding from peripheral sites (C) is the priority intervention. This client is presenting with signs of placental abruption. Disseminated intravascular coagulation (DIC) is a complication of placental abruption, characterized by abnormal bleeding.

A client at 30-weeks gestation, complaining of pressure over the pubic area, is admitted for observation. She is contracting irregularly and demonstrates underlying uterine irritability. Vaginal examination reveals that her cervix is closed, thick, and high. Based on these data, which intervention should the nurse implement first? a. Provide oral hydration b. Have a complete blood count (CBC) drawn c. Obtain a specimen for urine analysis d. Place the client on strict bedrest

c. Obtain a specimen for urine analysis Obtaining a urine analysis (C) should be done first because preterm clients with uterine irritability and contractions are often suffering from a urinary tract infection, and this should be ruled out first.

A client is admitted with the diagnosis of total placenta previa. Which finding is most important for the nurse to report to the healthcare provider immediately? a. Heart rate of 100 beats/minute b. Variable fetal heart rate c. Onset of uterine contractions d. Burning urination

c. Onset of uterine contractions Total (complete) placenta previa involves the placenta covering the entire cerviccal os (opening). The onset of uterine contractions (C) places the client at risk for dilation and placental separation, which causes painless hemorrhaging.

A newborn, whose mother is HIV positive, is scheduled for follow-up assessments. The nurse knows that the most likely presenting symptom for a pediatric client with AIDS is: a. shortness of breath b. joint pain c. a persistent cold d. organomegaly

c. a persistent cold Respiratory tract infections commonly occur in the pediatric population. However, the child iwth AIDS has a decreased ability to defend the body against these infections and often the presenting symptom of a child with AIDS is a persistent cold (C).

The nurse should encourage the laboring client to begin pushing when a. there is only an anterior or posterior lip of cervix left b. the client describes the need to have a bowel movement c. the cervix is completely dilated d. the cervix is completely effaced

c. the cervix is completely dilated Pushing begins with the second stage of labor, i.e., when the cervix is completely dilated (A, B, and D), the cervix can become edematous and may never completely dilate, necessitating an operative delivery. Many primigravidas begin active labor 100% effaced and then proceed to dilate.

A nurse on the high-risk unit is caring for a client with severe preeclampsia. Which intervention is the most effective in preventing a seizure?

controlling external stimuli

A pregnant woman comes to the prenatal clinic for an initial visit. In reviewing her childbearing history, the client indicated that she has delivered premature twins, one full-term baby, and has had no abortions. Which GTPAL should the nurse document in this client's record? a. 3-1-2-0-3 b. 4-1-2-0-3 c. 2-1-2-1-2 d. 3-1-1-0-3

d. 3-1-1-0-3 (D) describes the correct GTPAL. The client has been pregnant 3 times including the current pregnancy (G-3). She had one full-term infant (T-1). She also had a preterm (P-1) twin pregnancy (a multifetal gestation is considered one birth when calculating parity). There were no abortions (A-0), so this client has a total of 3 living children.

One hour after giving birth to an 8-pound infant, a client's lochia rubra has increased from small to large and her fundus is boggy despite massage. The client's pulse is 84 beats/minute and blood pressure is 156/96. The healthcare provider prescribes Methergine 0.2 mg IM x 1. What action should the nurse take immediately? a. Give the medication as prescribed and monitor for efficacy b. Encourage the client to breastfeed rather than bottle feed c. Have the client empty her bladder and massage the fundus d. Call the healthcare provider to question the prescription

d. Call the healthcare provider to question the prescription Methergine is contraindicated for clients with elevated blood pressure, so the nurse should contact the healthcare provider and question the prescription (D).

A client at 32-weeks gestation comes to the prenatal clinic with complaints of pedal edema, dyspnea, fatigue, and a moist cough. Which question is most important for the nurse to ask this client? a. Which symptom did you experience first? b. Are you eating large amounts of salty foods? c. Have you visited a foreign country recently? d. Do you have a history of rheumatic fever?

d. Do you have a history of rheumatic fever? Clients with a history of rheumatic fever (D) may develop mitral valve prolapse, which increases the risk for cardiac decompensation due to the increased blood volume that occurs during pregnancy, so obtaining information about the client's health history is priority.

A client who is in the second trimester of pregnancy tells the nurse that she wants to use herbal therapy. Which response is best for the nurse to provide? a. Herbs are a corner stone of good health to include in your treatment b. Touch is also therapeutic in relieving discomfort and anxiety c. Your healthcare provider should direct treatment options for herbal therapy d. It is important that you want to take part in your care

d. It is important that you want to take part in your care The emphasis of alternative and complementary therapies, such as herbal therapy, is that the client is viewed as a whole being, capable of decision-making and an integral part of the health care team, so (D) recognizes the client's request.

At 38 weeks' gestation a client is admitted to the birthing unit in active labor, and an external fetal monitor is applied. Late fetal heart rate decelerations begin to appear when her cervix is dilated 6 cm, and her contractions are occurring every 4 minutes and lasting 45 seconds. What is the likely cause of these late decelerations?

uteroplacental insufficiency

A client who gave birth to a healthy 8 pound infant 3 hours ago is admitted to the postpartum unit. Which nursing plan is best in assisting this mother to bond with her newborn infant? a. Encourage the mother to provide total care for her infant b. Provide privacy so the mother can develop a relationship with the infant c. Encourage the father to provide most of the infant's care during hospitalization d. Meet the mother's physical needs and demonstrate warmth toward the infant

d. Meet the mother's physical needs and demonstrate warmth toward the infant It is most important to meet the mother's requirement for attention to her needs so that she can begin infant care-taking (D). Nurse theorist Reva Rubin describes the initial postpartal period as the "taking-in phase," which is characterized by maternal reliance on others to satisfy the needs for comfort, rest, nourishment, and closeness to families and the newborn.

The nurse is calculating the estimated date of confinement (EDC) using Nagele's rule for a client whose last menstrual period started on December 1. Which date is most accurate? a. August 1 b. August 10 c. September 3 d. September 8

d. September 8 Calculation of a client's EDC provides baseline data to monitor fetal gestation. Nagele's rule uses the formula: subtract 3 months and add 7 days to the first day of the last normal menstrual period, so December 1 minus 3 months + 7 days is September 8 (D).

A woman who had a miscarriage 6 months ago becomes pregnant. Which instruction is most important is most important for the nurse to provide this client? a. Elevate lower legs while resting b. Increase caloric intake by 200 to 300 calories per day c. Increase water intake to 8 full glasses per day d. Take prescribed multivitamin and mineral supplements

d. Take prescribed multivitamin and mineral supplements A client who has had a spontaneous abortion or still birth in the last 1.5 years should take multivitamin and mineral supplements (D) and maintain a balanced diet because the previous pregnancy may have left her nutritionally depleted.

A 35-year-old primigravida client with severe preeclampsia is receiving magnesium sulfate via continuous IV infusion. Which assessment data indicates to the nurse that the client is experiencing magnesium sulfate toxicity? a. Deep tendon reflexes 2+ b. Blood pressure 140/90 c. Respiratory rate 18/minute d. Urine output 90 ml/4 hours

d. Urine output 90 ml/4 hours Urine outputs of less than 100 ml/4 hours (D), absent DTRs, and a respiratory rate of less than 12 breaths/minute are cardinal signs of magnesium sulfate toxicity

A nurse in the prenatal clinic is caring for a client with heart disease who is in her second trimester. Which hemodynamic change of pregnancy is likely to affect the client at this time?

heart rate acceleration in the last half of pregnancy

A pregnant couple is attending childbirth preparation classes. Which exercise should the nurse teach the mother to increase the tone of the muscles of the pelvic floor?

kegels

While auscultating the lungs of a client admitted with severe preeclampsia, the nurse identifies crackles. What inference does the nurse make when considering the presence of crackles in the lungs?

pulmonary edema

A woman who had a miscarriage 6 months ago became pregnant. Which instruction is most important for the nurse to provide this client? A. Elevate lower legs while resting B. increase caloric intake by 200-300 calories per day C. increase water intake to 8 full glasses per day D. take prescribed multivitamin and mineral supplements

take prescribed multivitamin and mineral supplements

In the eighth month of pregnancy a client tells the nurse that she is experiencing dyspareunia. Which information would be most helpful for the nurse to teach the client?

try other positions

A client is receiving an oxytocin infusion for induction of labor. The uterine graph on the electronic monitor indicates no rest period between contractions, and this is confirmed on palpation. What should the nurse do first?

turn oxytocin infusion off


Conjuntos de estudio relacionados

Chapter 12: Gender and Sexuality

View Set

new economic theories assignment

View Set

Chapter 6- Elasticity: The Responsiveness of Demand and Supply

View Set

VGP 3rd 6-Weeks Test Review 20/21

View Set

Accounting II exam 3 final exam review

View Set

Real Estate License Exam - Level 16: Property Insurance

View Set